You are on page 1of 174

Family  Medicine  Textbook  Notes     1  

 
 
  RISE  
Describe  the  approach  to   •   Identify  the  Risks  of  your  patient  
applying  preventative   •   Consider  recommended  Immunizations  (and  chemoprophylaxis)  
care  in  clinical  practice   •   Review  recommended  Screening  with  the  patient  
•   Address  appropriate  Education  or  counseling  
First  time  parents    
bringing  in  their  baby  for   It  is  important  to  establish  trust  early  on.  You  can  do  this  by  spending  a  short  time  interacting  with  
initial  well-­‐‑baby  exam.   the  infant  at  the  beginning  of  the  visit,  taking  time  to  elicit  any  concerns  or  questions  from  the  
What  is  the  approach  to   parent  early  in  the  visit,  and  remaining  aware  of  non-­‐‑verbal  cues.    
ease  anxiety  they  may  
have?  
What  are  the  common    
causes  of  lack  of   Inadequate  growth  may  be  the  presenting  feature  of  a  variety  of  disorders,  such  as  
development  in  an  infant   endocrinopathies,  cardiac  diseases,  and  renal  dysfunction.  More  commonly,  it  is  a  result  of  social  
seen  at  a  well-­‐‑child   stressors,  poor  bonding,  and  inadequate  nutrition.  
exam?  
   
What  measurements   First  2  years:  height,  weight,  and  head  circumferences  à  plotted  on  a  standardized  growth  chart  
should  be  used  to    
monitor  the  development   After  age  2:  height  and  weight  à  use  BMI  percentile  to  compare  to  other  children  
of  an  infant  over  time?  
Family  Medicine  Textbook  Notes     2  
 
  1)   Anaphylactic  reaction  to  vaccine  
  2)   Moderate  or  severe  acute  illness  following  a  dose  of  an  injectable  vaccine  
What  are  the   3)   Known  hypersensitivity  to  component  of  vaccine  
contraindications  for   4)   Moderate  or  severe  acute  illness  as  it  may  be  difficult  to  identify  subsequent  reactions  from  
vaccines  in  children?   immunization  
5)   Pregnancy  in  vaccine  recipient  (certain  live  vaccines  only)  
What  are  the  common    
causes  of  anemia  in  an  
infant  seen  at  a  well-­‐‑child  
exam?  
   
   
   
BMI  classifications  in   Overweight:  85th  –  95th  Percentile  
children?    
Obese:  >  95th  Percentile  
 
 
 
 
 
 
 
Family  Medicine  Textbook  Notes     3  
 
  Colic  is  a  term  often  used  to  describe  an  infant  who  is  difficult  to  manage  or  fussy  despite  being  
  otherwise  healthy.    
  •   May  be  defined  as  3  or  more  hours  of  uncontrollable  crying  or  fussing  at  least  3  times  per  
Describe  colic  as  seen   week  for  at  least  3  weeks  
during  well-­‐‑child   •   Other  symptoms  include  facial  expressions  of  pain  or  discomfort,  pulling  up  of  the  legs,  
examinations.   passing  flatus,  fussiness  with  eating,  and  difficulty  falling  or  staying  asleep  
•   Symptoms  classically  worsen  during  the  evening  
•   Usually  dissipates  once  the  child  is  3-­‐‑4  months  old  
•   Underlying  cause  is  unknown  à  organic  pathology  is  present  in  <  5%  
•   Possible  etiologies  include  an  immature  digestive  system  sensitive  to  certain  food  proteins,  
an  immature  nervous  system  sensitive  to  external  stimuli,  or  a  mismatch  between  the  
infant’s  temperament  and  those  of  the  caregivers  
•   No  long  term  consequences  of  colic  
No  definitive  treatment  can  be  offered  for  colic.  Little  evidence  supports  the  use  of  simethicone  or  
acetaminophen  drops.  May  be  helpful  to  reduce  the  amount  of  stimulation.  Frequent  burping,  
swaddling,  infant  massage,  or  white  noise  may  be  helpful.    
  Diagnoses  
  1)   Primary  Diaper  Dermatitis:  acute  skin  inflammation  in  the  diaper  area  with  multifactorial  
Describe  diaper  rash  as   etiology.  Main  cause  is  irritation  of  thin  skin  as  a  result  of  prolonged  contact  with  moisture  
seen  during  well-­‐‑child   including  feces  and  urine  
examinations.   2)   Irritant  Diaper  Dermatitis:  combination  of  intertrigo  (wet  skin  damaged  from  chafing)  and  
  miliaria  (heat  rash)  when  eccrine  glands  become  obstructed  from  excessive  hydration  à  
  typically  lasts  for  <  3  days  after  a  change  in  diaper  practices  
  3)   Candidal  Diaper  Dermatitis:  within  3  days,  45-­‐‑75%  of  diaper  rashes  are  colonized  with  
  Candida  albicans  of  fecal  origin  
  4)   Bacterial  Diaper  Dermatitis:  may  be  a  secondaryinfection  caused  by  S  aureus  or  Strep  
  pyogenes  
  5)   Perianal  Streptococcal  dermatitis:  caused  by  Group  A  ß-­‐‑hemolytic  streptococci  
Family  Medicine  Textbook  Notes     4  
 
  Risk  Factors  
  1)   Diarrhea  
  2)   Formula-­‐‑fed  infants  
  3)   Recent  antibiotic  use  
Describe  diaper  rash  as   4)   Urinary  Tract  Abnormalities  
seen  during  well-­‐‑child   5)   Poor  Skin  Care  
examinations,  continued   Treatment  
•   Parental  behavioral  change  to  keep  the  skin  as  exposed  and  dry  as  possible  
•   Frequent  diaper  changes  
•   Superabsorbant  diapers  that  pull  moisture  away  from  the  skin  are  helpful  
•   Apply  barrier  preparations,  including  zinc  oxide  paste,  petroleum  jelly,  vitamin  A  and  D  
ointment,  or  Burow  solution  (pastes  >  ointments  >  creams/lotions)  
•   For  moderate  to  severe  inflammation,  consider  a  nonfluorinated,  low-­‐‑potency  topical  
steroid  such  as  1%  hydrocortisone  ointment  
•   For  Candida,  use  topical  nonprescription  antifungal  creams:  clotrimazole,  miconazole  
•   No  follow  up  needed  unless  it  worsens  or  persists  
  By  9th  grade,  37%  of  males  and  29%  of  females  are  sexually  active.  These  youths  are  
  disproportionately  affected  by  chlamydia,  Neisseria  gonorrhea,  and  other  STIs.  Asymptomatic  
  carrier  states  are  common  and  associated  morbidity  is  high.    
   
Describe  STD  testing  of   All  sexually  active  female  adolescents  should  be  screened  for  chlamydia  and  gonorrhea.  There  is  
adolescents   insufficient  evidence  to  recommend  for  or  against  screening  asymptomatic  males.  High-­‐‑risk  
individuals  of  both  sexes  should  be  screened  for  syphilis.    
•   High  risk  =  men  who  have  sex  with  men,  men  and  women  who  have  unprotected  sex  with  
multiple  partners,  past  or  present  injection  drug  users,  men/women  who  exchange  sex  for  
money/drugs  or  have  sex  partners  who  do,  individuals  whose  past  or  current  sexual  
partners  were  HIV-­‐‑infected,  bisexual,  or  injection  drug  users,  persons  being  treated  for  
STIs,  and  persons  requesting  an  HIV  test  
Family  Medicine  Textbook  Notes     5  
 
  •   #1  cause  of  death  in  children/adolescents  =  unintentional  injuries  
What  are  the  most   •   Motor  vehicle  crashes  account  for  the  highest  number  of  deaths  among  children  older  than  
common  preventable   1-­‐‑year-­‐‑old.  
causes  of  death  in   •   Infants  who  sleep  on  their  stomachs  have  roughly  twice  the  incidence  of  SIDS  à  
children  and  adolescents?   recommended  that  infants  sleep  on  their  backs  (“Back  to  Sleep  Campaign”)  
•   Suicide  is  the  3rd  leading  cause  of  death  in  15  to  24  year  olds  

   
  In  the  Infant:  breastfeeding  is  associated  with  a  reduced  risk  of  otitis  media,  gastroenteritis,  
What  are  the  benefits  of   respiratory  illness,  SIDS,  necrotizing  enterocolitis,  obesity,  and  hypertension  
breastfeeding?    
In  the  Mother:  Reduced  risk  of  breast  and  ovarian  cancers,  type  2  diabetes,  and  postpartum  
depression  
 
  Home  
  Education  
Describe  the  HEADSSS   Activities  
mnemonic.   Drugs  
Sexuality  
Suicide  
Safety  
   
Describe  the  risk  for  
ovarian  cancer  in  women  
being  seen  at  well-­‐‑adult  
appointments.  
Family  Medicine  Textbook  Notes     6  
 
  USPSTF  Screening  Recommendations  for  aged  ≥  75  
  •   Tobacco  abuse:  recommended  (Level  A)  
  •   Alcohol  misuse:  recommended  (Level  A)  
  •   Nutrition  screening  and  counseling:  recommended  for  patients  with  CV  disease  risk  factors  
  (Level  B)  
Describe  routine   •   HTN:  recommended  (Level  A)  
preventative  care  for   •   Hyperlipidemia:  recommended  (Level  A)  
women  aged  75:   •   Aspirin  for  prevention  of  CV  Disease:  recommended  for  men  ≤  79  (Level  A)  
•   Aspirin  for  prevention  of  ischemic  stroke:  recommended  for  women  ≤  79  (Level  A)  
•   Diabetes:  recommended  for  BP  ≥  135/80  (Level  B)  
•   Obesity:  recommended  (Level  B)  
•   Depression:  recommended  if  supportive  care  is  available  (Level  B)  
•   Falls  (use  of  exercise,  physical  therapy,  vitamin  D  supplementation  if  high  risk,  community-­‐‑
dwelling):  recommended  (Level  B)  
USPSTF  Screening  Recommendation  Against  for  aged  ≥  75  
•   Prostate  Cancer  (PSA)  
•   Cervical  Cancer  
•   Colon  Cancer  if  aged  ≥  85  
Middle  aged  man  presenting   Screening  for  CV  conditions:  blood  pressure  (screen  for  HTN)  and  lipid  measurement  (screen  for  
for  wellness  exam.  No   dyslipidemia)  
complaints  on  history  and  has  
 
a  normal  physical  exam.  What  
screening  tests  should  be   Screening  for  Cancer:  Fecal  occult  blood  testing,  flexible  sigmoidoscopy  (w/  or  w/o  occult  blood  
considered?  Recommended   testing),  colonoscopy  or  double-­‐‑contrast  barium  enema  to  screen  for  colorectal  cancer  
immunizations?    
Immunizations:  Tetanus  toxoid,  reduced  diphtheria  toxoid,  and  acellular  pertussis  vaccine  (Tdap)  
if  he  has  never  had  one  or  if  it  has  been  10+  years  since  his  last  Td  vaccine;  influenza  vaccine  
annually  (in  the  fall/winter)  
Family  Medicine  Textbook  Notes     7  
 
   
What  are  the  components   The  purposes  of  the  health-­‐‑maintenance  visit  are  to  identify  the  individual  patient’s  health  
of  an  adult  health-­‐‑ concerns,  manage  the  patient’s  current  medical  conditions,  identify  the  patient’s  risks  for  future  
maintenance  visit?   health  problems,  perform  rational  and  cost-­‐‑effective  health  screening  tests,  and  promote  a  healthy  
lifestyle.  

  Screening  Tests  
  1)   CV  Disease:  USPSTF  strongly  recommends  (Level  A)  screening  of  adults  (aged  18+)  for  
  HTN  by  measuring  blood  pressure.  USPSTF  also  strongly  recommends  (Level  A)  screening  
  men  aged  35+  and  women  aged  45+  for  lipid  disorders.  USPSTF  recommends  (Level  B)  
  screening  adults  aged  20+  who  are  at  increased  risk  of  CV  diseases  for  lipid  disorders.  Men  
  aged  45-­‐‑79  are  recommended  (Level  A)  to  take  aspirin  daily  to  reduce  risk  of  MI  as  long  as  
  the  benefit  outweighs  the  risk  of  GI  hemorrhage.  Ultrasound  to  assess  for  abdominal  aortic  
  aneurysm  is  recommended  (Level  B)  for  men  65-­‐‑75  who  have  ever  smoked  
  2)   Cancer:  Men  and  women  aged  50+  are  strongly  advised  (Level  A)  to  have  screening  for  
What  are  the  screening   colorectal  cancer  à  fecal  occult  blood  testing  every  year,  sigmoidoscopy  every  3-­‐‑5  years,  
tests  and  immunizations   and  colonoscopy  every  10  years.  USPSTF  recommends  against  (Level  D)  routine  screening  
that  are  routinely   for  prostate  cancer  using  digital  examination  or  PSA.  Men  and  women  aged  50-­‐‑80  with  a  
recommended  for  adult   30+  pack  year  smoking  history  who  currently  smoke  or  quit  <  15  years  ago  are  
men?   recommended  (Level  B)  to  undergo  annual  low-­‐‑dose  CT  of  chest  to  screen  for  lung  cancer  
  3)   Other  Health  Conditions:  all  adults  (Level  B)  should  be  screened  for  obesity  (calculate  BMI).  
  Screening  for  diabetes  is  recommended  (Level  B)  for  adults  with  HTN  or  with  
  hyperlipidemia.  Screening  and  counseling  for  tobacco  use  is  strongly  recommended  (Level  
  A).  Screening  and  counseling  for  alcohol  use  is  also  recommended  (Level  B)  
   
   
   
Family  Medicine  Textbook  Notes     8  
 
  Immunizations  
  1)   Tdap:  all  adults  aged  19-­‐‑65  should  receive  booster  Tdap  in  place  of  a  scheduled  dose  of  Td  
What  are  the  screening   2)   Influenza:  routine  vaccination  recommended  for  everyone  aged  6  months  and  older  
tests  and  immunizations   3)   Hepatitis  B:  for  high  risk  individuals,  such  as  health-­‐‑care  workers,  those  exposed  to  blood  
that  are  routinely   or  blood  products,  dialysis  patients,  IV  drug  users,  persons  with  multiple  sexual  partners  or  
recommended  for  adult   recent  STDs  
men,  continued   4)   Hepatitis  A:  recommended  for  those  with  chronic  liver  disease,  those  who  use  clotting  
factors,  those  who  have  occupation  exposure  to  Hep  A  virus,  those  who  use  IV  drugs,  men  
who  have  sex  with  men,  or  those  who  travel  to  countries  where  Hep  A  is  endemic  
5)   Varicella:  for  those  who  have  no  reliable  history  of  immunization  or  disease,  those  who  are  
seronegative  on  testing  for  varicella  immunity,  and  for  those  exposed  to  the  virus  
6)   Meningococcal:  recommended  for  those  in  high  risk  groups,  such  as  college  dorm  residents  
or  military  recruits,  those  with  certain  complement  deficient,  those  with  function  or  
anatomical  asplenia,  and  those  who  travel  to  countries  where  it  is  endemic  
  The  primary  diagnostic  test  of  lung  function  is  spirometry.    
How  do  you  diagnose    
and  determine  the  stage   Normal  lung  function:  FEV1/FVC  >  0.7  
of  COPD  in  adults?   COPD:  FEV1  and  FVC  are  both  decreased;  FEV1/FVC  <  0.7  
Staging:  See  table  2-­‐‑1,  page  33  of  Case  Files  FM  
  •   All  patients  should  be  encouraged  to  quit  smoking  à  no  significant  improvements  in  lung  
  function  have  been  shown  to  occur,  but  this  does  prevent  the  rate  of  further  deterioration  
  •   Vaccination:  pneumococcal,  annual  influenza  
How  do  you  manage   •   Regular  efforts  +  efforts  to  maintain  normal  body  weight  
stable  COPD?   •   Avoid  secondhand  smoke,  aggravating  occupational  exposures,  and  indoor/outdoor  pollution  
•   Stage  I:  Short-­‐‑acting  bronchodilators  (albuterol,  ipratropium)  
•   Stage  II:  Long-­‐‑acting  bronchodilators  (salmeterol,  tiotroprium)  
•   Stage  III:  Inhaled  steroids  (fluticasone,  triamcinolone,  mometasone)  
•   Stage  IV:  Long-­‐‑term  oxygen  therapy  and  consider  surgical  interventions  
Family  Medicine  Textbook  Notes     9  
 
  Presents  with  change  in  sputum  color  or  amount,  cough,  wheezing,  and  increased  dyspnea.    
   
How  do  you  manage   MCC:  respiratory  tract  infections  (viral  or  bacterial).  Also  can  be  causes  by  air  pollutants  
acute  exacerbations  of    
COPD?   The  mainstays  of  medical  therapy  are  oxygen,  short-­‐‑acting  bronchodilators,  and  systemic  steroids  
(which  shorten  the  course  of  exacerbation  and  reduce  the  risk  of  relapse  à  40  mg  prednisolone  for  
10-­‐‑14  days).  Give  antibiotics  if  appropriate.  All  dyspneic  patients  should  have  an  assessment  of  
their  level  of  oxygenation.  
  Much  in  the  chest  can  hurt,  and  heart  disease  is  actually  one  of  the  less  common  causes  of  chest  
  pain  in  the  primary  care  setting.  
   
  1.    Acute  Coronary  Syndrome  
  •   Pain  is  due  to  hypoperfusion  of  the  myocardium,  usually  from  occlusion  of  a  coronary  
What  are  the  various   artery  by  thrombus  formed  on  the  disrupted  endothelium  of  a  ruptured  atherosclerotic  
causes  of  chest  pain?   plaque  
  •   Unstable  Angina  =  rest  pain  for  >  20  minutes  that  is  likely  associated  with  an  unstable  
  coronary  artery  occlusion  and  hypoperfusion  
  •   Variant  Angina  is  caused  by  spastic  narrowing  of  otherwise  normal  coronary  arteries  
  2.  Stable  Angina  
  •   “Chronic  effort-­‐‑dependent  angina”  
  •   Caused  by  lack  of  sufficient  oxygen  delivery  to  the  myocardium  during  exertion,  most  
  often  because  of  impaired  blood  flow  past  the  hallmark  atherosclerotic  plaques  of  CAD  
  •   Produced  characteristic  squeezing  or  dull  “pain”  of  stable  angina  as  well  as  characteristic  
  changes  on  EKG  (b/c  ischemic  muscle  conducts  electricity  differently)  and  wall  motion  
  abnormalities  on  echocardiography  (ischemic  muscle  does  not  contract  normally)  
  3.  Panic  Disorder  
  •   Mechanism  of  chest  pain  unknown  
   
Family  Medicine  Textbook  Notes     10  
 
  4.  Gastroesophageal  Reflux  Disease  and  Esophageal  Spasm  
  •   GERD  causes  irritation  of  the  esophageal  mucosa  
What  are  the  various   •   Esophageal  spasm  mimics  angina  
causes  of  chest  pain,   •   Reflux  can  be  exacerbated  by  triggers  that  relax  the  lower  esophageal  sphincter,  most  
continued   notably  caffeine,  alcohol,  and  fatty  foods  
5.  Pericarditis  
•   Pain  is  worse  when  recumbent  and  relieved  by  sitting  forward  
6.  Other  Causes  
•   Musculoskeletal  chest  pain  commonly  arises  from  ribs  or  thoracic  soft  tissue  
•   Pulmonary  embolism  may  produce  pleuritic  pain,  but  its  symptoms  are  notoriously  
variable  
•   Pleuritic  pain  can  also  be  produced  by  inflammation  from  an  infectious  process  or  by  
neoplasm  
•   Spontaneous  pneumothorax  is  uncommon  and  is  associated  with  vigorous  exercise,  
primarily  in  men  in  their  20s  
•   Rarely  can  be  due  to  thoracic  aortic  dissection  (almost  exclusively  found  among  
hypertensive  patients;  Marfan  Syndrome  and  syphilis  are  very  rare)  à  causes  a  tearing  
pain  that  may  be  felt  in  the  back  
  The  most  important  part  of  the  physical  exam  is  the  patient’s  overall  appearance.  If  pain  is  present  
What  is  the  most   at  the  time  of  examination,  is  the  patient  simply  describing  a  pain  that  is  annoying,  or  is  he/she  
important  aspect  of  the   pale  and  sweaty?  The  patient  presenting  with  chest  pain  should  have  vital  signs  assessed  
physical  exam  when  a   promptly  and  monitored  closely  for  changes  during  the  evaluation.  
patient  presents  with    
chest  pain?   Tachycardia  in  particular  is  characteristic  of  panic  and  pulmonary  embolism  (the  latter  especially  
when  accompanied  by  tachypnea).  Bradycardia,  especially  if  new  or  symptomatic,  may  be  
associated  with  inferior  myocardial  ischemia.  
Family  Medicine  Textbook  Notes     11  
 
  1)   Hypotension,  poor  tissue  perfusion,  pulmonary  edema,  or  oliguria  à  suggestive  of  decreased  
What  are  the  red  flags   cardiac  output,  possible  large  anterior  myocardial  infarction  
suggesting  life-­‐‑ 2)   Tachycardia,  tachypnea,  hypoxia  à  suggestive  of  pulmonary  embolism  
threatening  disease  in   3)   EKG  changes,  especially  ST  Elevation  or  new  left  bundle-­‐‑branch  block  à  suggestive  of  MI  
patients  with  chest  pain?   4)   New  systolic  mitral  murmur  à  suggestive  of  ruptured  papillary  muscle  
5)   Arrhythmia  and/or  chest  pain  in  younger  patient  à  suggestive  of  cocaine  abuse  
6)   Mediastinal  widening  on  chest  radiograph  à  suggestive  of  aortic  dissection  with  severe  tearing  
or  ripping  pain  
  1)   Infection  of  the  joint  à  this  is  the  first  differential  that  needs  to  be  excluded  because  
  cartilage  can  be  destroyed  within  the  first  24  hours  of  an  infection.    
What  is  the  differential   2)   Gout  à  most  commonly  occurs  in  men  aged  30-­‐‑50  and  in  women  aged  50-­‐‑70  (increased  
diagnosis  for  non-­‐‑ female  sex  hormones  in  premenopausal  women  protects  against  gout  by  increasing  the  
traumatic  joint  pain?   urinary  excretion  of  uric  acid)  
3)   Pseudogout  à  due  to  calcium  pyrophosphate  dehydrate  crystals  in  the  joints  
4)   Osteoarthritis  à  Most  commonly  in  people  >  65  with  repetitive  use,  trauma,  and  obesity  
(knee  OA)  à  presents  with  dull,  deep,  ache-­‐‑type  pain  
5)   Rheumatoid  Arthritis  à  usually  aged  30-­‐‑55.  Women  >  men  
  Physical  Exam  à  Range  of  motion  is  important  to  evaluate.    
  •   A  septic  joint  will  have  very  limited  ROM  due  to  pain  coupled  with  joint  effusion  and  
What  are  the  common   fever.  However,  a  nearby  cellulitis,  bursitis,  or  osteomyelitis  (also  in  the  differential)  will  
diagnostic  tests  used  for   usually  maintain  the  ROM  of  a  joint.    
the  most  common  causes   •   In  OA,  a  bony  crepitus  may  be  felt  on  passive  ROM  
of  non-­‐‑traumatic  joint   First  step  in  diagnostics:  examination  of  joint  aspirate  
pain?   1)   Septic  Arthritis  à  aspirate  will  have  an  average  of  100,000  WBC/microliter,  with  >  90%  
  neutrophils.  >  90%  of  aspirates  will  have  a  positive  culture.    
  2)   Gout  à  Polarizing  microscopy  of  joint  aspirate  will  show  monosodium  urate  crystals,  
  which  look  like  needles  and  have  a  strong  negative  birefringence  
 
Family  Medicine  Textbook  Notes     12  
 
  3)   Pseudogout  à  calcium  pyrophosphate  dehydrate  (CPPD)  crystals,  which  are  rod-­‐‑shaped,  
What  are  the  common   rhomboid,  and  have  a  weak  positive  birefringence  
diagnostic  tests  used  for   Imaging  
the  most  common  causes   •   In  OA  à  X  Rays  will  be  normal  initially,  but  there  will  be  gradual  development  of  bone  
of  non-­‐‑traumatic  joint   sclerosis,  subchondral  cysts,  and  osteophytes  
pain,  continued   Labs  
•   In  RA  à  Rheumatoid  Factor  (+),  Anti-­‐‑citrullinated  protein  antibody  (anti-­‐‑CCP)  (+),  elevated  
ESR,  elevated  C-­‐‑Reactive  Protein,  Anemia,  Thrombocytosis,  and  low  Albumin  
  Acute  Gout  Attack:  
  •   Low-­‐‑dose  Colchicine  
What  are  the  most   •   NSAIDs  (avoid  in  elderly,  heart  failure  patients,  those  with  peptic  ulcer  disease,  and  those  
common  treatment   with  liver  or  renal  disease)  
options  in  the  acute  onset   •   Intra-­‐‑articular  glucocorticoids  
of  gout?   •   Ice  Packs  
Chronic  Therapy  for  recurrent  attacks:  
•   Probenecid  à  increases  urinary  excretion  of  uric  acid  
•   Allopurinol  à  reduced  production  of  uric  acid  
   
What  are  the  most    
common  treatment   •   IV  antimicrobials  
options  in  the  acute  onset   •   Surgical  drainage  of  the  infected  joint  
of  infectious  arthritis?  
Family  Medicine  Textbook  Notes     13  
 
What  are  the  most    
common  treatment   •   Exercises  to  maintain  joint  mobility  and  muscle  strength  
options  in  the  chronic   •   Disease-­‐‑modifying  anti-­‐‑rheumatic  drugs  (DMARDs)  à  sulfasalazine,  methotrexate  
management  of   •   Can  also  use  NSAIDs,  glucocorticoids,  anti-­‐‑cytokines,  and  topical  analgesics  
rheumatoid  arthritis?  
What  are  the  most   •   Mobility  exercises  
common  treatment   •   Maintenance  of  adequate  ROM  
options  in  the  chronic   •   Weight  loss,  if  appropriate  
management  of   •   Intra-­‐‑articular  corticosteroid  injections  for  short-­‐‑term  relief  à  should  only  be  done  every  4-­‐‑
osteoarthritis?   6  months  to  avoid  cartilage  destruction  

  •   Women  who  intend  to  become  pregnant  should  be  advised  to  avoid,  whenever  possible,  
  potentially  harmful  agents  such  as  radiation,  drugs,  alcohol,  tobacco,  OTC  medications,  
What  are  the  components   herbs,  and  other  environmental  agents  
of  preconception   •   Radiation  exposure  greater  than  5  rad  is  associated  with  fetal  harm  à  most  routine  X  Rays  
counseling?   (dental,  etc)  only  expose  the  fetus  to  a  small  fraction  of  that  amount  
•   Recommended  that  all  women  of  childbearing  age  should  take  daily  folic  acid  supplements  
and  women  considering  conception  should  start  a  folic  acid  supplement  at  least  1  month  
prior  to  trying  to  conceive  à  low  risk  women  only  need  400-­‐‑800  micrograms  of  folic  acid  
daily  to  reduce  the  risk  of  neural  tube  defects;  women  who  have  had  a  previous  child  with  
a  neural  tube  defect  should  take  4  mg  of  folic  acid  daily  
•   Screening  for  genetic  diseases:  African  and  African-­‐‑American  women  may  be  offered  sickle  
cell  trait  screening;  French-­‐‑Canadian  or  Ashkenazi  Jewish  women  can  be  screened  for  Tay-­‐‑
Sachs  carrier  status;  Southeast  Asian  and  Middle  Eastern  women  may  be  screened  for  
thalassemia;  Ashkenazi  Jews  and  Caucasian  women  can  be  screened  for  cystic  fibrosis  
•   Women  aged  35+  at  time  of  delivery  should  be  educated  about  age-­‐‑related  risks,  
particularly  for  Down  Syndrome  
Family  Medicine  Textbook  Notes     14  
 
•   Women  with  multiple  medical  conditions  (diabetes,  asthma,  thyroid  disease,  HTN,  lupus,  
thromboembolism,  seizures)  should  be  referred  to  providers  with  experience  managing  
high-­‐‑risk  pregnancies  
•   Women  with  psychiatric  conditions  à  co-­‐‑manage  with  a  psychiatrist  
•   Screen  for  tobacco  use  and  alcohol  dependence  
•   Educate  about  proper  nutrition  and  exercise  during  pregnancy  
•   Discuss  social  issues,  such  as  financial  readiness,  social  support,  and  domestic  violence  
  •   Ideally,  the  first  visit  should  be  in  the  first  trimester.  Most  occur  at  week  8  or  later  
  History  
  •   Assess  last  menstrual  period  à  accurate  gestation  dating  is  one  of  the  most  crucial  pieces  of  
  information  needed  
  •   Pay  attention  to  medical  history,  prior  pregnancies,  delivery  outcomes,  pregnancy  
  complications,  neonatal  complications,  and  birth  weights  
  •   Gynecologic  history  should  focus  on  menstrual  history,  contraceptive  use,  and  history  of  
What  are  the  components   STDs  
of  the  initial  prenatal   •   Allergies,  current  medications,  and  substance  use  should  be  investigated  
visit?   •   Social  history:  Was  the  pregnancy  planned,  unplanned?  Social  support?  Genetic  history?  
  Physical  Exam  
  •   Height,  weight,  blood  pressure,  thyroid,  breast,  general  physical  and  pelvic  examinations  
  •   Estimation  of  gestational  age  by  uterine  size  or  fundal  height  measurement.  
  •   Attempt  to  hear  fetal  heart  tones  by  Doppler  fetoscope  
  Labs  
  •   Blood  type  
  •   Rh  status  antibody  screening  
  •   Rubella  status  
  •   HIV  
  •   HBsAg  
  •   RPR  
Family  Medicine  Textbook  Notes     15  
 
  •  Urinalysis  
  •  Urine  culture  
  •  Pap  smear  
  •  Cervical  swab  for  gonorrhea  and  Chlamydia  
What  are  the  components   •  CBC  
of  the  initial  prenatal   •  Triple  Screening  (Trisomy  21,  Trisomy  18,  Neural  Tube  Defects)  at  15-­‐‑20  weeks  gestation  
visit,  continued   •  USPSTF  recommends  screening  for  gestational  diabetes  (Level  B)  after  24  weeks  gestation  
à  if  positive,  perform  a  3-­‐‑hour  glucose  tolerance  test  following  an  overnight  fast  by  giving  
the  patient  a  100-­‐‑g  glucose  load  and  obtaining  fasting,  1-­‐‑hour,  2-­‐‑hour,  and  3-­‐‑hour  postload  
serum  glucose  à  2/4  positive  values  =  diagnosed  with  gestational  diabetes  
•   At  28  weeks,  perform  a  repeat  RPR  and  hemoglobin/HCT    
•   At  28  weeks,  give  RhoGAM  if  woman  is  Rh-­‐‑negative  
•   All  women  at  35-­‐‑37  weeks  gestation  should  be  offered  Group  B  Strep  screening  by  
vaginorectal  culture  
Immunizations  
•   Inactivated  influenza  should  be  offered  to  all  pregnant  women  during  flu  season  à  all  
pregnant  women  should  receive  the  influenza  vaccine  at  their  initial  prenatal  visit  
•   Tdap  should  be  given  between  27  and  36  weeks  gestation  of  each  pregnancy,  regardless  of  
prior  vaccination  status  
Logistical  Issues  
•   Follow-­‐‑up  visits  every  4  weeks  until  28  weeks  gestation,  every  2  weeks  from  28-­‐‑  to  36-­‐‑  
weeks,  and  every  week  from  36-­‐‑weeks  to  delivery  
Family  Medicine  Textbook  Notes     16  
 
  For  women  who  are  Rh  negative,  the  next  step  is  to  assess  the  antibody  screen  or  indirect  Coombs  
Describe  in  detail  the   test.    
recommended  actions  for   •   If  the  antibody  screen  is  negative,  there  is  no  isoimmunization,  and  RhoGAM  is  given  at  28  
RhoGAM  treatment  in   weeks  gestation  and  again  at  delivery  if  the  baby  is  confirmed  as  Rh  positive  (RhoGAM  is  
pregnant  women?   given  to  prevent  isoimmunization)  
•   If  the  antibody  screen  is  positive  and  the  identity  of  the  antibody  is  confirmed  as  Rh  (anti-­‐‑
D),  then  assessment  of  its  titer  will  assist  in  knowing  the  probability  of  fetal  effect  
  •   ACE-­‐‑Inhibitors  (Level  A)  à  improves  survival  post-­‐‑MI  in  anterior  infarcts  with  ejection  
  fraction  <  40%  
  •   Emergent  Reperfusion  (thrombolysis  or  percutaneous  coronary  intervention)  for  STEMI  
What  is  the  treatment   patients  (Level  A)  à  Target  time  to  thrombolysis  <  30  minutes;  Target  time  to  PCI  =  <  60  
strategy  for  acute  MI?   minutes  
•   All  patients  with  suspected  acute  coronary  syndromes  should  receive  aspirin,  325  mg  
swallowed  or  chewed,  immediately  and  then  continued  indefinitely  
•   In  addition  to  immediate  anti-­‐‑platelet  therapy,  patients  with  a  suspected  ACS  should  
receive  oxygen,  nitroglycerine,  and  morphine,  if  necessary  
•   ß-­‐‑blockers  can  improve  short-­‐‑term  and  long-­‐‑term  mortality  in  patients  with  ACS,  and  
should  be  administered  orally  within  the  first  24  hours  of  onset  
   
What  are  the  cholesterol   Patients  with  CAD:  
goals  for  a  patient  with   •   LDL  <  100  mg/dL  
chronic  cardiac  disease?   •   HDL  >  40  mg/dL  
•   Non-­‐‑HDL  cholesterol  (serum  triglycerides/5  +  LDL)  <  130  mg/dL  
Family  Medicine  Textbook  Notes     17  
 
  Percutaneous  transcutaneous  coronary  stenting  should  be  considered  instead  of  coronary  artery  
  bypass  in  patients  with  one-­‐‑,  two-­‐‑,  or  three-­‐‑vessel  disease  who  have  anatomy  suitable  for  stent  
What  are  the  common   therapy  and  who  have  normal  left  ventricular  function  
reasons  for  heart  bypass    
or  stent?   The  presence  of  significant  left  main  coronary  disease,  multi-­‐‑vessel  disease  not  amenable  to  
stenting,  or  significant  CAD  in  the  presence  of  left  ventricular  dysfunction  (ejection  fraction  <  50%)  
indicates  that  coronary  artery  bypass  graft  may  be  beneficial.  
  New-­‐‑Onset  AF  
  •   If  patient  is  hemodynamically  unstable  (ventricular  rate  >  140  bpm  and  with  acute  MI,  chest  
  pain,  dyspnea,  or  HF)  à  urgent  synchronized  cardioversion  
  •   If  patient  is  hemodynamically  stable  (identified  w/in  <  48  hours  of  onset)  à  start  heparin,  
  perform  transesophageal  echo  (TEE)  to  rule  out  atrial  thrombus,  and  cardioversion  
  •   If  new  onset  AF  identified  >  48  hours  after  onset  à  TEE  can  be  performed  to  rule  out  atrial  
How  do  you  treat  atrial   thrombus  and  cardioversion;  but  if  thrombus  is  present,  anticoagulation  with  warfarin  is  
fibrillation?   begun  and  cardioversion  is  delayed  3  weeks  
  •   HR  should  be  controlled  with  IV  diltiazem  or  ß-­‐‑blocker  to  maintain  ventricular  rate  b/w  60-­‐‑
  80  
  Paroxysmal  AF  (self-­‐‑limiting)  
  •   Most  should  NOT  be  placed  on  long-­‐‑term  rhythm  maintenance  therapy  
  •   If  recurrent  paroxysmal  episodes  occur,  and  if  the  patient  has  minimal  heart  disease,  first  
  line  therapy  is  flecainide  propafanone,  or  sotalol.  2nd  line  therapy  is  amiodarone  
  •   For  patients  with  AF  and  HF  à  first  line  therapy  is  amiodarone  
  •   For  patients  with  AF  and  CAD  à  Sotalol  is  first  line  therapy  
  •   For  patients  with  AF,  HTN,  and  LVH  à  amiodarone  
  •   For  patients  with  AF,  HTN,  and  no  LVH  à  flecainide  and  propafenone  are  first  line;  
  Amiodarone  and  sotalol  are  2nd  line  
   
   
Family  Medicine  Textbook  Notes     18  
 
   
  Persistent  AF  
How  do  you  treat  atrial   •   If  AF  doesn’t  terminate  spontaneously  or  with  initial  medications  à  either  have  to  accept  
fibrillation,  continued   progression  to  permanent  AF  or  attempt  cardioversion  to  normal  sinus  rhythm  (depends  
on  risk  factors  such  as  age,  symptoms,  and  risk  for  thromboembolic  events)  
Permanent  AF  
•   Control  HR  60-­‐‑80  bpm  à  use  drugs  that  block  the  AV  node  à  ß-­‐‑blockers  (first  line),  
nondihydropyridine  calcium  channel  blockers,  and  digoxin  
  Thiazide  Diuretics  
  •   Hypokalemia  
  •   Hyperuricemia  
  •   Hyponatremia  
•   Erectile  dysfunction  
 
ACE  Inhibitors  
 
•   Hyperkalemia  
What  are  the  side  effects   •   Cough  
of  the  major  HTN   •   Angioedema  
medications?   •   Increased  serum  creatinine  
  Calcium  Channel  Blockers  
  •   Peripheral  Edema  (amlodipine,  felodipine,  nifedipine)  
  •   Dizziness  (Diltiazem,  verapamil)  
  Angiotensin  Receptor  Blockers  
  •   Hyperkalemia  
  •   Angioedema  (less  than  ACE-­‐‑Is)  
ß-­‐‑Blockers  
 
•   Bradycardia  
 
•   Bronchoconstriction  (with  non-­‐‑cardioselective  ß-­‐‑blockers)  
  •   Erectile  dysfunction  
  •   Hypertriglyceridemia  
  •   Hyperglycemia  
Family  Medicine  Textbook  Notes     19  
 
  Loop  Diuretics  
What  are  the  side  effects   •   Hypokalemia  
of  the  major  HTN   •   Ototoxicity  
medications,  continued   Aldosterone  Antagonists  
•   Hyperkalemia  
•   Gynecomastia  (spironolactone)  
•   Erectile  dysfunction  

  ASCVD  Risk  Assessment  


  •   Blood  Pressure  à  longitudinal  studies  have  shown  that  systolic  BP  is  a  better  predictor  for  
  future  morbidity  and  mortality,  especially  for  middle-­‐‑aged  and  older  individuals    
  •   Smoking  status  
What  screening  tests  are   •   Total  cholesterol  and  HDL  cholesterol  
best  for  heart  disease  in  a   •   Diabetes  status  
40  year  old?   Recommendations  for  initial  evaluation  of  hypertension  (based  on  expert  opinion)  
•   Look  for  features  suggesting  secondary  disease,  such  as  young  age  (<25),  malignant  course,  and  
lack  of  response  to  therapy.  
•   Ask  about  symptoms  of  end-­‐‑organ  disease,  such  as  chest  pain,  orthopnea,  paroxysmal  
nocturnal  dyspnea,  lower  extremity  edema,  claudication,  and  left  ventricular  hypertrophy.  
•   Identify  other  CV  risk  factors,  such  as  diabetes,  smoking,  family  history  of  early  CAD,  and  
history  of  elevated  cholesterol  level  
•   On  physical  exam,  check  pulse  (for  arrhythmias),  BMI  (for  obesity),  cardiac  size,  rhythm,  S3,  
JVD,  and  edema  (for  LV  hypertrophy  and  heart  failure),  carotid  and  peripheral  pulses  (for  
carotid  artery  disease  and  peripheral  artery  disease),  renal  bruits  (for  renovascular  
hypertension),  fundus  (for  hypertensive  retinopathy),  and  neurologic  examination  (for  prior  
cerebrovascular  event)  
•   Labs  to  order:  urinalysis  (for  hypertensive  nephropathy),  urine  microalbumin  (for  
microalbuminuria  à  early  hypertensive  nephropathy),  BUN/Creatinine  (for  renal  dysfunction),  
potassium  and  sodium  levels  (for  aldosteronism),  glucose  (for  diabetes),  fasting  lipid  panel  (for  
hyperlipidemia),  and  electrocardiogram  (for  LV  hypertrophy,  prior  myocardial  infarction)  
Family  Medicine  Textbook  Notes     20  
 
  Patient  Presentation:  
  •   Acutely  painful  and  swollen  leg  
  Patient  history:  
  •   Historical  elements  with  the  greatest  positive  predictive  values  for  DVT  are  recent  surgery,  
How  do  you  manage  a   and  immobilization  for  more  than  3  days  in  the  past  4  weeks  
DVT?   Diagnostic  Tests  
•   Duplex  venous  ultrasound*  
•   MRI  
•   Helical  computed  tomography  
•   D-­‐‑Dimer*  
•   Contrast  venography  (invasive,  but  considered  the  reference  standard)  
Family  Medicine  Textbook  Notes     21  
 
 
 
 
 
 
How  do  you  manage  a  
pulmonary  embolism?  

 
Family  Medicine  Textbook  Notes     22  
 
 
 
 
 
 
 
 
 
 
 
What  is  the  treatment  
protocol  for  DVT?  
 
 
 
 
 
 
 
 
 
 
   
 
 
 
 
 
Family  Medicine  Textbook  Notes     23  
 
 
 
 
 
 
 
 
What  is  the  treatment  
protocol  for  DVT,  
continued  

 
Family  Medicine  Textbook  Notes     24  
 

 
  Heparin  
  •   MOA:  interacts  with  antithrombin  à  enhances  its  ability  to  inhibit  thrombosis  by  
  inactivating  clotting  factor  proteases,  especially  thrombin  (IIa),  IXa,  and  Xa  
Compare  heparin,   •   SE:  Hemorrhage,  Heparin-­‐‑Induced  Thrombocytopenia  
Coumadin,  and  low   Coumadin  (Warfarin)  
molecular  weight   •   MOA:  prevents  action  of  vitamin  K  epoxide  reductase,  which  activates  vitamin  K  à  blocks  
heparins  (LMWH)   gamma  carboxylation  of  factors  II,  IV,  IX,  and  X,  and  proteins  C  and  S  
•   SE:  bleeding,  teratogenic  (use  LMWH  when  patient  is  pregnant)  
LMWH  (Enoxaparin)  
•   MOA:  inhibits  clotting  factors  IIa  (thrombin)  and  Xa  
•   Advantages  include  fixed  dosing,  a  subcutaneous  route  of  administration  (makes  
outpatient  treatment  possible),  and  a  more  predictable  anticoagulant  response.  Also  have  
better  bioavailability,  longer  half-­‐‑life,  and  dose-­‐‑independent  clearance.    
Family  Medicine  Textbook  Notes     25  
 
  Mononucleosis  
  •   History/Physical  Exam:  sore  throat  follows  a  30-­‐‑50  day  incubation  period  and  a  3-­‐‑5  day  
  prodrome  characterized  by  fever,  malaise,  myalgias,  and  headache.  Other  common  physical  
  signs  include  pharyngeal  inflammation  (85%)  and  transient  palatal  petechial  (50%).  Cervical  
  adenopathy  (90%  posterior  cervical)  and  fever  are  present  in  over  99%.  Typically  in  patients  
  aged  15-­‐‑24  years  old.  
  •   Diagnostic  Testing:  Most  develop  lymphocytosis  (95%  have  >  60%  lymphocytes),  which  
  peaks  2  weeks  after  onset  of  symptoms.  Atypical  lymphocytes  are  common.  Monospot  test  
  (heterophil  antibodies  that  agglutinate  sheep  erythrocytes)  is  widely  used,  but  it  is  negative  
  in  the  first  week  of  infection.  Another  test  identifies  viral  capsid  antigen  immunoglobulin  M  
Compare  and  contrast   antibodies,  which  are  produced  early  in  the  infection  and  do  not  persist  once  the  acute  
mononucleosis  and  strep   infection  is  over.  
pharyngitis:   •   Management:  Treat  symptomatically  with  rest,  oral  fluids,  and  NSAIDs  or  acetaminophen  
for  fever  and  myalgias.  Aspirin  should  be  AVOIDED  because  of  its  association  with  Reye  
Syndrome.  Corticosteroids  are  recommended  in  patients  with  significant  pharyngeal  
edema.  Participation  in  contact  sports  should  be  limited  during  the  acute  phase  and  
continue  to  be  restricted  for  at  least  4  weeks  and  as  long  as  the  spleen  is  palpable.  
GBS  
•   History/Physical  Exam:  Fever,  absence  of  cough,  tonsillar  or  pharyngeal  exudate,  cervical  
adenopathy.  The  scarlatina  rash  (sandpapery)  is  rare,  but  highly  specific  for  strep.  
•   Diagnostic  Testing:  a  variety  of  rapid  antigen  tests  (enzyme  immunoassays,  liposomal  
assays,  and  immunochromatographic  assays)  and  cultures  are  available  
•   Management:  If  there  is  a  high  probability  that  the  patient  has  GBS,  give  them  
cephalosporin  for  7-­‐‑10  days,  penicillin  VK  250  mg  PO  QID  for  10  days,  or  amoxicillin  1  g  
PO  BID  for  6  days.  You  can  give  a  Dexamethasone  10  mg  IM  injection  for  severe  pain  and  
tonsillar  enlargement.  
 
Family  Medicine  Textbook  Notes     26  
 
   
  Palliative  care  is  a  cornerstone  of  both  end-­‐‑of-­‐‑life  care  as  well  as  chronic  disease  management.  The  
When  do  you  use   goal  of  palliative  treatment  is  not  just  to  extend  life,  but  to  maximize  comfort,  function,  and  quality  
palliative  care?   of  life.  The  biggest  difference  at  the  end  of  life  is  that  palliative  care  principles  become  paramount,  
and  the  prolongation  of  length  of  life  becomes  less  important  than  the  quality  of  life.    

  SET  up  the  interview  


  •   Arrange  to  give  potentially  bad  test  results  in  person  
  •   Arrange  for  privacy,  adequate  time,  and  no  interruptions  
  •   Involve  significant  others  
  •   Sit  down,  establish  rapport,  allow  for  silence/tears  
  •   Mentally  rehearse  and  emotionally  prepare  for  the  interview  
What  is  Buckman’s   Assess  the  patient’s  PERCEPTION  
Protocol  for  breaking  bad   •   “Ask  before  you  tell”  –  what  does  the  patient  know/understand  
news?   •   Ask  open-­‐‑ended  questions,  tailor  news  to  current  understand,  correct  misinformation,  
  identify  denial  
  Obtain  the  patient’s  INVITATION  
  •   Most  patients,  but  not  all,  want  full  disclosure  
  •   Discuss  information  disclosure  at  the  time  of  ordering  tests  and  before  giving  results  
  Give  KNOWLEDGE  (information)  to  the  patient  
  •   Warn  the  patient  bad  news  is  coming  à  “I’m  sorry,  but  I  have  bad  news”  or  “I’m  sorry  to  
  tell  you  that….”  
  •   Target  the  patient’s  vocabulary/comprehension  
  •   Avoid  euphemisms,  technical  jargon,  and  excess  bluntness  
  •   Ask  the  patient  to  repeat  back  what  you’ve  said  
  •   Regardless  of  prognosis,  identify  goals  –  e.g.,  cure,  pain  and  symptom  relief,  family  issues  
   
Family  Medicine  Textbook  Notes     27  
 
  Address  the  patient’s  EMOTIONS  with  empathic  responses  
  •   Physicians  are  generally  uncomfortable  with  patients’  emotional  reactions  to  bad  news  
What  is  Buckman’s   •   Four  components  of  an  empathic  response:  1)  observe  the  patient’s  emotions  2)  identify  the  
Protocol  for  breaking  bad   emotion  to  yourself  3)  identify  the  reason  for  that  emotion  4)  let  the  patient  know  that  you  
news,  continued   have  connected  with  that  emotion  
STRATEGIZE  and  SUMMARIZE  
•   A  clear  plan  lessens  the  patient’s  anxiety  and  fosters  patient  self-­‐‑determination  
•   Ask  if  the  patient  is  ready  to  discuss  a  plan  
•   Use  the  patient’s  knowledge,  expectations,  and  goals  as  a  starting  point;  discuss  fears;  
gently  work  past  denial  
•   Arrange  follow-­‐‑up  meetings    
  Pain  may  be  nociceptive,  which  presumes  normally  functioning  pain  receptors  and  nerves  or  
  neuropathic,  which  presumes  abnormal  function  of  the  peripheral  or  central  nervous  system.  
In  end-­‐‑of-­‐‑life  care,  when    
do  you  advance  to  the   Nociceptive  pain  responds  well  to  opioid  analgesics.  Neuropathic  pain  often  requires  adjuvant  
next  pain  medication  for   analgesics  in  addition  to  opioids.    
appropriate  pain  control?    
When  rotating  from  one  opiate  to  another,  a  relative  dose  reduction  to  allow  for  incomplete  cross-­‐‑
tolerance  is  often  advisable.    
Family  Medicine  Textbook  Notes     28  
 

 
Family  Medicine  Textbook  Notes     29  
 

 
  Constipation  remains  one  of  the  most  common  distressing  symptoms  for  patients  receiving  
                  palliative  care.  Contributing  factors  include  metabolic  derangements  from  the  underlying  illness  
                                                                                                                                                                                                                                                                                                                                                                                                                                             
itself,  immobility  and  decreased  fluid  intake  associated  with  declining  functional  status,  and  
In  end-­‐‑of-­‐‑life  care,  how   medical  treatments  (especially  opioids).    
do  you  control    
constipation?   When  patients  are  treated  with  chronic  opioids,  the  standard  practice  is  to  initiate  regular  
  administration  of  a  stimulant  laxative.    
 
 
 
 
 
   
Family  Medicine  Textbook  Notes     30  
 
 
 
 
 
 
 
In  end-­‐‑of-­‐‑life  care,  how  
do  you  control  
constipation,  continued  

 
Family  Medicine  Textbook  Notes     31  
 
  Diagnosis  of  depression  in  end-­‐‑of-­‐‑life  care  is  made  more  difficult  by  the  fact  that  many  of  the  
How  do  you  handle   somatic  symptoms  of  depression  in  physically  healthy  adults  (sleep  disturbance,  loss  of  appetite,  
depression  care  for   and  fatigue)  can  be  caused  by  the  underlying  disease  rather  than  depression.  Therefore,  
terminal  patients?   psychological  criteria  for  depression  (dysphoria,  anhedonia,  and  feelings  of  worthlessness  or  guilt)  
become  more  important  in  end-­‐‑of-­‐‑life  care.  

  •   Requires  that  the  attending  physician  certifies  a  life  expectancy  of  6  months  or  less  (Since  
  predicting  life  expectancy  is  an  inexact  science  and  many  physicians  are  uncomfortable  
What  are  the  eligibility   predicting  a  life  expectancy  of  less  than  6  months,  it  is  better  to  ask,  “would  you  be  
criteria  for  hospice?   surprised  if  this  patient  died  in  the  next  6  months?”)  
•   If  a  patient  lives  beyond  the  6  months  while  on  hospice,  they  may  remain  on  hospice  
service  as  long  as  there  is  adequate  documentation  that  they  met  hospice  eligibility  criteria  
upon  initiation  of  hospice  services.  
  •   Occurs  when  growth  is  interrupted  à  may  be  genetic,  medical,  nutritional,  behavioral,  
  psychological,  or  environmental  à  whatever  the  underlying  causes  are,  the  immediate  
  cause  of  FTT  is  malnutrition  
  •   There  is  no  single  definition,  measurement,  or  set  of  criteria  that  best  diagnoses  FTT  à  most  
  clinicians  will  consider  the  diagnosis  for  children  without  weight  gain  in  2  months  or  
Failure  to  Thrive   children  who  have  dropped  2  percentile  curves  in  <  6  months.  However,  the  most  common  
cause  of  an  abnormal  growth  curve  is  measurement  or  plotting  error,  so  this  should  be  
considered  before  other  action  is  taken.  
•   History  is  generally  the  most  important  diagnostic  tool  in  finding  the  underlying  cause  of  
FTT.  
•   Hospitalization  is  rarely  needed  for  a  child  with  FTT.  Hospitalization  should  be  considered  
in  cases  of  severe  malnutrition,  hypothermia,  bradycardia,  or  hypertension.    
•   FTT  in  the  first  year  of  life,  particularly  in  the  first  6  months,  is  more  likely  to  affect  brain  
development.  
Family  Medicine  Textbook  Notes     32  
 
  •   ASD  refers  to  a  continuum  of  disorders  of  brain  development  involving  impaired  
  communication  skills;  impaired  social  interactions;  and  restricted,  repetitive,  or  
  stereotypical  patterns  of  behavior  
  •   Concern  should  be  raised  when  a  child  exhibits  aberrant  social  skills,  abnormal  eye  contact,  
Autism  Spectrum   aloofness,  failure  to  orient  to  name,  failure  to  use  gestures  to  point  or  show,  lack  of  
Disorder   interactive  play,  or  lack  of  interest  in  peers.    
•   Several  screening  tools  include  the  Checklist  for  Autism  in  Toddlers  (CHAT)  for  18-­‐‑month  
olds,  and  the  Pervasive  Developmental  Disorder  Screening  Test.    
•   There  is  no  widely  accepted  guidelines  for  treatment,  but  there  is  agreement  that  early  and  
sustained  intervention  greatly    improves  outcomes.  
  •   CP  is  a  disorder  of  movement  and  posture  caused  by  injury  to  the  motor  areas  of  the  brain.  
  Motor  abnormalities  must  be  static,  not  progressive  over  time.  Due  to  rapid  development  in  
  the  first  year  of  life,  a  definitive  diagnosis  cannot  be  made  until  after  1  year  of  age  
Cerebral  Palsy   •   CP  has  many  etiologies,  all  involving  injury  to  the  developing  brain.    
•   Abnormalities  are  seen  in  posture,  oropharyngeal  function  (tongue  thrusts,  swallowing),  
strabismus,  increased  or  decreased  muscle  tone,  abnormal  evolution  of  primitive  reflexes,  
or  abnormal  deep  tendon  reflexes  
•   Other  conditions  that  can  occur  with  CP  include  seizures,  refractive  errors,  hearing  loss,  
mental  retardation,  failure  to  thrive,  and  behavioral  problems.  
•   Subtypes  of  CP:  spastic,  athetoid,  ataxic,  and  mixed  
•   The  prognosis  for  children  with  CP  is  extremely  variable  depending  on  the  type  and  
severity  of  the  neurologic  insult.  
Family  Medicine  Textbook  Notes     33  
 
  •   MR  refers  to  cognitive  ability  that  is  markedly  below  average  for  chronological  age  with  a  
  decreased  ability  to  adapt  to  the  environment.  The  diagnosis  of  decreased  cognitive  ability  
  must  be  made  via  standardized  testing.  
  •   Standardized  testing  is  less  predictive  for  young  children  à  the  term  “developmental  
  delay”  is  used  for  children  younger  than  3  
Mental  Retardation   •   Mild  MR:  IQ  50-­‐‑70.    
•   Moderate  MR:  IQ  35-­‐‑49    
•   Severe  MR:  IQ  20-­‐‑34.    
•   Profound  MR:  IQ  <  20  
•   Causes:  Due  to  near  drowning  in  children,  traumatic  brain  injury,  CNS  malignancy,  lead  
exposure,  infection,  first  trimester  maternal  fever,  intrauterine  alcohol  or  anticonvulsant  
exposure,  untreated  maternal  phenylketonuria,  metabolic  diseases  (i.e.,  hypothyroidism),  
single  gene  mutations  (i.e.,  fragile  X  syndrome,  neurofibromatosis),  or  chromosomal  
abnormalities  such  as  Down  syndrome,  Klinefelter  syndrome,  or  Prader-­‐‑Willi  syndrome  
  •   ADHD  is  characterized  by  inattention,  hyperactivity,  academic  underachievement,  
  behavior  problems  and  impulsivity  
  •   Recommended  to  initiate  an  evaluation  for  ADHD  in  all  children  between  6  and  12  
  presenting  with  symptoms    
  •   Symptoms  of  ADHD  must  be  present  before  age  7  and  must  be  present  in  two  or  more  
Attention  Deficit/   settings  (e.g.,  school  and  home).  There  must  be  clear  impairment  in  functioning  (social,  
Hyperactivity  Disorder   academic,  or  occupational)  
•   Many  disorders  co-­‐‑exist  with  ADHD,  such  as  oppositional  defiant  disorder  (35%),  conduct  
disorder  (26%),  anxiety  disorders  (26%),  and  depressive  disorders  (18%)  
•   Guidelines  recommend  initiating  treatment  with  a  stimulant  medication  (methylphenidate)  
and/or  behavioral  therapy  
•   After  stabilizing  treatment,  children  with  ADHD  should  be  seen  at  regular  intervals  to  
monitor  continued  response  to  therapy,  behavioral  goals,  academic  progress,  and  side  
effects.  
Family  Medicine  Textbook  Notes     34  
 
  •   Disruptive  behaviors  represent  a  spectrum  from  normal  disobedience  and  risk  taking  to  
  severe  conduct  disorder  
Disruptive  Behavior   •   Oppositional  Defiant  Disorder:  characterized  by  negativistic,  defiant,  and  hostile  behavior  
towards  authority  figures  
•   Conduct  Disorder:  the  persistent  violation  of  the  rights  of  others  and  societal,  age  
appropriate  norms  
•   The  most  important  risk  factors  for  CD  are  in  the  domain  of  the  family:  poor  family  
functioning,  substance  abuse,  psychiatric  disease  in  a  parent,  marital  discord,  child  abuse  
and  neglect,  and  poor  parenting.  Child  abuse  is  the  strongest  and  most  consistent  risk  
factor  for  CD.    
•   The  family  physician  should  consider  both  alternative  and  comorbid  disorders  including  
ADHD,  depression,  anxiety,  personality  disorders,  learning  disabilities,  and  substance  
abuse.    
•   Treatment  of  ODD:  parent  training  programs  
•   Treatment  of  CD:  family  and  parenting  interventions,  child  skill  training  
  •   LLDs  very  often  occur  with  ADHD  and  disorders  of  conduct  
  •   Disorders  of  expressive  or  receptive  language  require  delays  in  these  areas  not  due  to  
  sensory  or  motor  deficit  or  environmental  deprivation.  These  delays  must  be  in  excess  of  
Language  and  Learning   those  expected  by  nonverbal  intelligence  scores  (IQ)  
Disorders   •   75%  of  children  with  learning  disabilities  have  social  skill  deficits.  In  the  classroom,  they  
have  less  on-­‐‑task  behavior,  more  off-­‐‑task  behavior,  more  conduct  disorder,  more  
distractibility,  and  more  withdrawn  behavior  
•   More  than  other  disorders,  the  diagnosis  of  LLD  requires  specialized  testing  beyond  the  
scope  of  the  family  physician  
Family  Medicine  Textbook  Notes     35  
 
  •   Anorexia  Nervosa:  patients  refuse  to  maintain  even  a  minimally  normal  body  weight,  have  
  intense  fear  of  gaining  weight  or  becoming  fat  despite  being  underweight,  and  exhibit  
Eating  Disorders   disturbance  in  the  perception  of  the  shape  or  size  of  their  bodies.  Amenorrhea  will  be  
present  in  post-­‐‑menarchal  women.    
•   Bulimia  Nervosa:  patients  exhibit  recurrent  episodes  of  binge  eating  with  recurrent  
inappropriate  compensatory  behaviors  to  prevent  weight  gain  (vomiting,  use  of  laxatives,  
diuretics,  enemas,  excessive  exercise,  or  diet  pills).  To  meet  diagnostic  criteria,  binge  eating  
and  compensatory  behaviors  must  occur  at  least  twice  per  week  for  at  least  3  months  
•   Eating  Disorder  Not  Otherwise  Specified:  Patient  has  many  eating  disorder  symptoms  but  
does  not  meet  criteria  for  classic  AN  or  BN  
•   Binge  Eating  Disorder:  a  form  of  EDNOS  that  involves  binge  eating  without  regular  
compensatory  behaviors  
 
•   A  patient  with  disordered  eating  may  present  with  symptoms  related  to  almost  any  organ  
system.  More  often,  presenting  complaints  are  physical,  such  as  abdominal  pain  or  syncope,  
or  psychological,  such  as  irritability,  depression,  or  sleep  disturbance.  
•   History  and  physical  exam  is  the  cornerstone  to  making  the  diagnosis  of  an  eating  disorder.    
•   Treatment:  SSRIs  
Family  Medicine  Textbook  Notes     36  
 

 
Family  Medicine  Textbook  Notes     37  
 
  •   Classically,  severe  hyperglycemia  generates  symptoms  of  fatigue,  weight  loss,  polydipsia,  
  polyphagia,  and  polyuria.    
  •   More  subtle  signs  and  symptoms  include  obesity,  recurrent  infections  (especially  yeast  
What  is  the  most  common   vaginitis,  skin  infections,  and  periodontal  infections),  slow  healing  wounds,  neurological  
initial  presentation  for  a   syndromes  (especially  focal  limb  neuropathies  presenting  with  paresthesia,  burning,  and  
patient  with  diabetes?   tingling  in  the  extremities),  visual  changes  and  blurry  vision,  abdominal  pain  from  
nonalcoholic  fatty  liver  or  chronic  pancreatitis,  heart  disease  or  stroke,  and,  in  women,  
menstrual  irregularity  and  obesity,  polycystic  ovarian  syndrome,  history  of  gestational  
diabetes,  or  giving  birth  to  an  infant  weighing  more  than  9  pounds.  

 
 
 
What  are  the  clinical  red  
flags  suggesting  a  
hyperglycemic  crisis  in  
Type  I  and  Type  II  
diabetics?  

 
Family  Medicine  Textbook  Notes     38  
 
 
 
 
What  are  the  diagnostic  
criteria  for  diabetes?  
What  are  the  criteria  for  
diagnosing  someone  with  
an  increased  risk  of  
diabetes?  
 

 
 
 
What  are  the  components  
of  the  physical  exam  that  
should  be  performed  for  
a  newly  diagnosed  
patient  with  diabetes?  
 

 
Family  Medicine  Textbook  Notes     39  
 
  Treatment  of  Hypertension  
  •   Maintain  BP  below  130/80  mmHg  
  •   Advise  patients  to  follow  DASH  diet  
  •   If  microalbuminuria  is  present,  treat  with  ACE  inhibitors  or  ARBs  
Smoking  Cessation  
 
•   Counseling  on  smoking  cessation  
 
Aspirin  Therapy  to  Prevent  Stroke  
  •   For  women  with  increased  10-­‐‑year  risk  of  stroke;  take  81  mg  aspirin  daily  to  reduce  the  risk  of  
  stroke  
  •   For  men  with  increased  10-­‐‑year  risk  of  CHD:  take  81  mg  aspirin  daily  to  reduce  risk  of  heart  attack  
  Management  of  Dyslipidemia  
What  are  the   •   Use  statins  to  treat  to  LDL  goal  <  100  mg/dL  
recommendations  for  the   Management  of  Hyperglycemia  
management  of  diabetes?   •   A  team  approach  to  care  (e.g.,  provider,  nurse,  pharmacist,  diabetes  educator)  improves  outcomes  
and  patient  satisfaction  
•   Long-­‐‑term  A1C  below  or  around  7.0%  lowers  risk  of  microvascular  and  possibly  macrovacular  
complications  
•   Metformin  is  the  initial  drug  of  choice  for  patients  who  can  take  it  
•   When  insulin  is  started,  oral  agents  should  be  continued,  to  reduce  total  insulin  needs  and  weight  
gain  
•   Glucose  self-­‐‑monitoring  improves  overall  management  of  hyperglycemia  in  patients  treated  with  
insulin  
•   Monitor  A1C  every  6  months  and  more  frequently  if  target  levels  have  not  been  reached  
Screening  for  and  Management  of  Complications  
•   In  patients  with  retinopathy,  control  of  blood  pressure  and  glucose,  laser  photocoagulation,  
vitrectomy,  and  other  interventions  can  help  preserve  sight  
•   Screen  for  nephropathy  annually  using  spot  microalbumin/creatinine  ratio  (<30  =  normal)  
•   Screen  annually  for  peripheral  neuropathy  
•   Perform  foot  exam  annually  
Family  Medicine  Textbook  Notes     40  
 
  Rapid-­‐‑Acting  Insulin  à  used  with  meals  to  decrease  postprandial  rise  in  blood  glucose  
  1)   Lispro  Insulin  (“Humalog”)  
  2)   Insulin  Aspart  (“NovoLog”)  
  3)   Insulin  glulisine  (“Apidra”)  
  Short-­‐‑Acting  Insulin  à  Regular  human  insulin  used  with  meals  to  decrease  postprandial  rise  in  
  blood  glucose.  Less  expensive  than  rapid-­‐‑acting  insulin  analogues  
  1)   Regular  Insulin  (“Humalin  R”;  “Novolin  R”)  
  Intermediate-­‐‑Acting  Insulin  à  used  with  regular  insulin  to  manage  total  daily  insulin  
  requirements.  Usually  administered  twice  daily.  Infrequently  used.  
  1)   Isophane  insulin  suspension  (NPH)  (“Humulin  N;  Novolin  N”)  
  2)   Insulin  Zinc  Suspension  (lente)  (“Humulin  L”)  
  Long-­‐‑Acting  Insulin  à  Used  as  a  once  daily  long-­‐‑acting  insulin.  Infrequently  used.  
What  are  the  treatments   1)   Extended  insulin  zinc  suspension  (ultralente)  (“Humulin  U”)  
used  in  the  management   2)   Insulin  glargine  (analog)  (“Lantus”)  
of  diabetes?   3)   Insulin  determir  (“Levemir”)  
Biguanides  
1)   Metformin  (“Glucophage”)  à  watch  for  signs  and  symptoms  of  lactic  acidosis.  Renal  
excretion.  Start  at  a  low  dose  to  avoid  GI  side  effects.  
Sulfonylureas  à  Renal  excretion  
1)   Glimepiride  (“Amaryl”)  
2)   Glipizide  (“Glucotrol”)  
3)   Glyburide  (“DiaBeta”,  “Micronase”,  “Glynase”)  
Thiazolidinediones  à  hepatic  elimination  
1)   Pioglitazone  (“Actos”)  
2)   Rosiglitazone  (“Avandia”)  
Other  Newer  Agents  
1)   Sitagliptin  (“Januvia”)  à  expensive;  risk  of  pancreatitis  
2)   Exenatide  (“Byetta”)  à  very  expensive;  risk  of  pancreatitis  
Family  Medicine  Textbook  Notes     41  
 
  Starting  a  patient  on  nighttime  basal  insulin  
  1)   Before  initiating  insulin  à  review  behavioral  management;  teach  or  review  self  monitoring  
  blood  glucose  monitoring;  teach  adjustment  of  insulin  dose  based  on  SBGM  results;  Teach  
  injection  technique;  Review  signs,  symptoms,  and  treatment  of  hypoglycemia  
  2)   Visit  at  which  insulin  is  initiated  à  calculate  initial  insulin  dose  (initial  dose  =  Fasting  
  Plasma  Glucose  (mg/dL)/18);  review  patient’s  injection  technique;  review  SBGM,  
  adjustment  algorism  and  hypoglycemia  instructions  
  3)   Subsequent  visits  à  consider  non-­‐‑visit  review  of  SBGM  (fax,  e-­‐‑mail,  electronic  transmission  
What  are  the  therapy   from  glucose  meter)  
guidelines  for  insulin?   Initiating  and  adjusting  BID  Insulin  
•   Before  starting  BID  regimen  à  discontinue  sulfonylurea;  continue  Metformin,  TZD  agent;  
Review  SBGM,  new  insulin  adjustment  algorithm,  diet  and  exercise  plans;  if  not  already  
SBGM  BID,  begin  BID  monitoring  (fasting,  before  evening  meals)  
•   Starting  Dose  à  AM:  50%  of  previous  nighttime  insulin  dose;  PM:  50%  of  previous  
nighttime  insulin  dose  
•   Dose  Adjustment  à  AM  dose  adjustments  based  on  before-­‐‑evening-­‐‑meal  SBGM  readings;  
PM  dose  adjustments  based  on  fasting  before-­‐‑breakfast  SBGM  readings  
•   Dose  adjustment  frequency  à  Make  dose  reductions  based  on  a  reading  <  70  mg/dL  the  very  
next  day;  Make  dose  increases  based  on  the  average  glucose  reading  from  the  previous  3  
days  every  3  days  
•   Dose  decrease  details  à  if  any  before  supper  plasma  glucose  measurement  is  <  70  mg/dL,  
reduce  the  before-­‐‑breakfast  insulin  dose  by  2  units  beginning  the  next  morning;  If  any  
fasting  plasma  glucose  measurements  is  <  70  mg/dL,  reduce  the  before-­‐‑supper  insulin  dose  
by  2  units  beginning  that  afternoon.  
Family  Medicine  Textbook  Notes     42  
 
 
 
 
 
 
 
 
 
 
What  are  the  
recommendations  for  
monitoring  ongoing  
diabetes?  
 

 
  Daytime  Hypoglycemia  
What  is  the  dosing   •   Mild  hypoglycemia  during  the  day  à  reduce  sulfonylurea  dose  by  25%  
adjustment  strategy  for   •   Severe  hypoglycemia  (requires  assistance  from  someone  else)  during  the  dayà  reduce  
when  a  diabetic  patient   sulfonylurea  by  50%  
has  daytime  and   Nighttime  Hypoglycemia  
nighttime  hypoglycemia?   •   For  any  documented  hypoglycemia  episode  at  night,  reduce  insulin  dose  by  2  units,  wait  1  
week  before  any  dose  increases  
Family  Medicine  Textbook  Notes     43  
 
   
   
   
  A1c  <  7%  
  •   Self  management  medical  nutrition  therapy  
  •   Exercise  
  A1c  7-­‐‑8%  
What  is  the  treatment   •   Metformin  
strategy  for  a  patient  with   A1c  8-­‐‑11%  
diabetes?   •   Metformin  plus  one  other  drug:  
-­‐‑   If  insulin  deficient  à  add  sulfonylurea  (glipizide)  
-­‐‑   If  insulin  defect  à  add  DPP-­‐‑4  inhibitor  (sitagliptin)  and  GLP-­‐‑1  agonist  (exenatide)  
-­‐‑   If  insulin  resistant  à  add  TZD  (pioglitazone)  
A1c  >  11%  
•   Metformin  +  TZD  +  basal  insulin  
•   Add  multi-­‐‑daily  insulin  
 
 
 
 
 
 
 
 
 
Family  Medicine  Textbook  Notes     44  
 
  Causes  
  •   Congenital  hypothyroidism  is  most  commonly  caused  by  endemic  iodine  deficiency.  In  
  countries  with  sufficient  iodine  intake,  it  is  usually  caused  by  thyroid  gland  dysgenesis  or  
  defective  hormone  synthesis.  
  •   Acquired  hypothyroidism  is  usually  the  result  of  autoimmune  (Hashimoto’s)  thyroiditis.  
  Other  causes  include  surgical  removal  of  thyroid  tissue  or  destruction  of  the  thyroid  by  
How  do  you  diagnose   radioactive  iodine,  other  external  radiation,  or  toxin  exposure.  Also  caused  by  certain  
and  treat   drugs,  such  as  amiodarone  and  lithium.  Transient  hypothyroidism  can  be  caused  by  
hypothyroidism?   subacute  or  lymphocytic  thyroiditis.  
Signs  and  Symptoms  
•   Weakness,  lethargy,  cold  intolerance,  decreased  sweating,  forgetfulness,  constipation,  
coarse  or  dry  skin,  slow  speech,  eyelid  edema,  skin  cold  to  touch,  thick  tongue,  facial  
edema,  coarse  hair,  skin  pallor.    
•   Physical  exam  may  reveal  a  diffuse  or  nodular  goiter,  sluggish  movements,  bradycardia,  
pretibial  edema,  facial  puffiness,  coarse  skin,  brittle  nails,  carpel  tunnel  syndrome,  and  
prolongation  of  the  ankle  reflex.    
Diagnostic  Testing  
•   Elevated  TSH  
•   Low  free  T4  
Treatment  
•   Hormone  supplementation  with  levo-­‐‑thyroxine  à  ~100  mcg/day  for  women,  ~125  mcg/day  
for  men.  Dosages  are  titrated  by  25-­‐‑60  mcg  every  6  weeks  until  a  euthyroid  state  (normal  
TSH)  is  achieved  
•   Because  the  half-­‐‑life  of  thyroid  hormone  is  nearly  a  week,  it  takes  3-­‐‑6  weeks  after  initiating  
a  dosage  change  for  a  steady  state  to  be  achieved.    
Family  Medicine  Textbook  Notes     45  
 
  Causes  
  •   Grave’s  Disease  (60-­‐‑80%  of  cases)  
  •   Toxic  Nodular  Goiter  and  solitary  hyper-­‐‑functioning  nodules  
  •   Thyroiditis  
  •   Excess  iodine  ingestion,  either  from  diet,  radiographic  contrast,  or  medication  
  •   Factitious  hyperthyroidism  (intentional  or  accidental  ingestion  of  excess  thyroid  hormone)  
   
  Signs  and  Symptoms  
  •   Tachycardia,  fatigue,  weight  loss,  goiter,  tremor,  apathy,  atrial  fibrillation,  anorexia,  
How  do  you  diagnose   nervousness,  hyperactive  reflexes,  depression,  increased  sweating,  polydipsia,  heat  
and  treat   intolerance,  and  increased  appetite.    
hyperthyroidism?   •   Physical  exam  may  demonstrate  weight  loss,  elevated  systolic  blood  pressure,  tachycardia,  
  pretibial  edema,  tremor,  or  proximal  muscle  weakness.    
  Diagnostic  Testing  
  •   Primarily  decreased  TSH  
  •   TSH  is  either  normal  (Normal  free  T4  à  subclinical  hyperthyroidism,  resolving  
  hyperthyroidism,  medication,  pregnancy,  and  non-­‐‑thyroid  illness;  High  free  T4  à  
  exogenous  hormone,  thyroiditis,  iodide  exposure,  extraglandular  production,  Grave’s  
Disease,  toxic  multinodular  goiter,  toxic  adenoma)  or  TSH  is  elevated  (rarely)  à  secondary  
hyperthyroidism  (pituitary  tumor)  
Treatment  
•   ß-­‐‑blockers  resolve  the  adrenergic  symptoms  
•   Antithyroid  drugs  (Methimazole,  Propothiouracil)    
•   Persistent  hyperthyroidism  is  treated  with  antithyroid  drugs,  radioactive  iodine,  or  surgery  
Family  Medicine  Textbook  Notes     46  
 
  General  ways  to  describe  goiter:  
  1)   Endemic  vs  sporadic:  goiter  is  termed  endemic  if  it  occurs  in  10%  or  more  of  the  population  
  2)   Simple  vs  multinodular:  simple  means  the  gland  is  diffusely  enlarged;  multinodular  goiters  
  have  multiple  nodules  within  the  gland  
  3)   Nontoxic  vs  toxic:  nontoxic  goiter  exhibits  normal  thyroid  function,  whereas  toxic  goiter  
  refers  to  an  enlarged  gland  associated  with  either  hypo-­‐‑  or  hyperthyroidism  
  History  and  Physical  
Describe  the  evaluation   •   Most  patients  with  goiter  are  asymptomatic.  History  should  be  directed  towards  eliciting  
and  management  of   symptoms  of  hypo-­‐‑  or  hyperthyroidism  and  determining  the  presence  of  risk  factors  for  
goiter   thyroid  disease  or  malignancy  (ingestion  of  goitrogens,  pregnancy,  smoking,  family  history  
of  thyroid  pathology,  history  of  neck  radiation  therapy,  previous  thyroid  surgery,  and  
cervical  adenopathy)  
•   Physical  exam:  palpation  to  determine  size,  tenderness,  and  presence  of  nodules  
Labs  
•   Order  TSH  
•   Thyroid  ultrasound  should  be  considered  in  multinodular  goiter  to  rule  out  the  presence  of  
a  dominant  nodule  that  would  need  further  workup  
Treatment  
•   Surgery  is  the  treatment  of  choice  for  patients  with  large  symptomatic  goiters  or  those  with  
risk  factors  for  thyroid  cancer  à  total  thyroidectomy  is  preferred  over  subtotal  
thyroidectomy  b/c  it  results  in  no  risk  of  recurrence  and  has  the  same  risk  of  surgical  
complications  
Family  Medicine  Textbook  Notes     47  
 
  General  
  •   Solitary  thyroid  nodules  are  frequently  found  incidentally.  Occur  in  women  >  men.    
  Diagnosis  
  •   Most  common  type  of  nodule  is  a  colloid  nodule,  which  has  no  malignant  potential.  
  •   Cysts  and  thyroiditis  are  also  common.    
  •   Colloid  nodules,  cysts,  and  thyroiditis  make  up  80%  of  nodules.    
  •   Of  the  remaining  20%,  15%  are  benign  follicular  adenomas,  and  5%  are  cancerous.    
  History  and  Physical  Exam  
  •   Factors  Raising  Strong  Concern  
  -­‐‑   Family  history  of  medullary  thyroid  cancer  or  multiple  endocrine  neoplasia  
  -­‐‑   Rapid  growth  of  nodule  
Describe  the  evaluation   -­‐‑   Firm  or  hard  nodule  
and  management  of   -­‐‑   Nodule  fixed  to  adjacent  structures  
solitary  thyroid  nodules:   -­‐‑   Paralysis  of  vocal  cords  
-­‐‑   Regional  lymphadenopathy  
•   Factors  Raising  Moderate  Concern  
-­‐‑   Male  sex  
-­‐‑   Patient  age  younger  than  20  or  older  than  65  
-­‐‑   Previous  radiation  to  the  head  or  neck  
-­‐‑   Nodule  greater  than  4  cm  or  partially  cystic  
-­‐‑   Symptoms  suggesting  compression  (dysphagia,  hoarseness,  dyspnea)  
Labs  
•   TSH  
•   Free  T4,  if  indicated  
•   Ultrasound:  Doppler  criteria  that  may  indicate  an  increased  risk  of  malignancy  include  
irregular  margins,  intranodular  vascular  spots,  and  microcalcifications  
•   Fine  needle  biopsy  of  a  thyroid  nodule  is  the  most  accurate  and  cost-­‐‑effective  way  to  
differentiate  benign  from  malignant  nodules  
Family  Medicine  Textbook  Notes     48  
 
Treatment  
•   All  nodules  1  cm  or  greater,  any  nodule  in  a  patient  with  risk  factors,  or  any  nodule  with  
suspicious  ultrasound  characteristics  should  be  biopsied  by  an  experienced  clinician.    
•   If  biopsy  shows  benign  follicular  cells  à  no  further  treatment  
•   If  malignant  cells  found  à  near-­‐‑total  thyroidectomy,  except  for  medullary  carcinoma,  
which  requires  total  thyroidectomy  and  bilateral  regional  lymph  node  resection  

 
 
 
 
What  are  the  disease  risks  
associated  with  obesity?  

 
Family  Medicine  Textbook  Notes     49  
 
 
 
 
When  advising  patients  
about  exercise,  what  
types  of  everyday  
activities  can  you  give  as  
examples  of  “moderate  
physical  activity”?  

 
Family  Medicine  Textbook  Notes     50  
 
 
 
 
 
What  drugs  can  be  used  
for  weight  loss?  

 
  •   Surgical  procedures  for  weight  loss  should  be  reserved  for  patients  in  whom  medical  
  weight  loss  treatment  has  failed,  and  who  are  suffering  from  complications  of  extreme  
What  are  the  indications   obesity.    
for  surgery  in  order  to   •   Bariatric  surgery  can  be  considered  an  option  for  patients  with  a  BMI  ≥  40,  or  ≥  35  if  
help  patients  lose   cardiovascular  risk  factors  are  present.  
weight?   •   Gastric  bypass  has  been  shown  to  be  more  effective  than  gastric  banding  for  weight  loss  
and  requires  fewer  surgeries  for  revision,  but  has  more  side  effects.  
  •   High-­‐‑fat  foods  do  not  necessarily  help  an  underweight  person  gain  weight  because  they  
What  are  some   cause  the  person  to  feel  full  for  long  periods  and  therefore  can  act  as  an  appetite  
considerations  for   suppressant  
management  of   •   Snacks  or  small  frequent  meals  may  help  and  should  be  taken  at  least  two  hours  before  the  
underweight  patients?   next  meal  
•   Adding  calories  without  adding  volume  to  liquids  consumed  may  be  effective  (e.g.,  adding  
powdered  milk  to  regular  milk  to  increase  protein  calories)  
Family  Medicine  Textbook  Notes     51  
 
  •   Age  <  2  years  
  •   Male  
What  are  the  risk  factors   •   Genetic  predisposition  
for  the  development  of   •   Previous  episode(s)  of  otitis  media  
acute  otitis  media?   •   Cigarette  smoking  in  household  
•   Attendance  at  day  care  
•   Recent  upper  respiratory  infection  
   
  •   Decreased  hearing  
  •   Pain  that  is  unaffected  by  movement  of  the  outer  ear  (otitis  externa  causes  pain  with  
What  are  the  clinical   external  ear  movement)  
findings  in  a  patient  with   •   Associated  systemic  signs  of  infection  such  as  fever  or  malaise  
acute  otitis  media  vs   •   Otitis  media  generally  causes  a  dulling  of  sound,  but  hearing  is  still  present.  Complete  
acute  otitis  externa?   hearing  loss  is  more  common  with  auditory  canal  occlusion  (foreign  body,  cerumen)  and  
sometimes  with  otitis  externa  
•   A  mobile  TM  suggests  that  no  fluid  is  present  in  the  middle  ear  and  that  the  diagnosis  of  
otitis  media  cannot  be  made.    
•   With  examination  of  the  tympanic  membrane  for  acute  otitis  media,  the  most  useful  
positive  findings  include  a  bulging  or  cloudy  TM  (due  to  effusion),  bulging  of  the  TM,  and  
a  loss  of  TM  mobility  
 
 
 
Family  Medicine  Textbook  Notes     52  
 
  First  Line  
  •   Amoxicillin:  80  mg/kg  split  at  least  BID  X  at  least  5  days  
  •   SMX-­‐‑TMP:  40  mg/kg  SMX  and  8  mg/kg  TMP  divided  BID  X  10  days  à  avoid  in  patients  
What  is  the  treatment   with  Sulfa  allergy,  G6PD  deficiency;  light  sensitivity  possible  
strategy  for  acute  otitis   Second  Line  
media?   •   Ceftriaxone:  50  mg/kg  up  to  1  g  
•   Amoxicillin-­‐‑clavulanate:  20-­‐‑45  mg/day  of  amoxicillin  component  in  2  or  3  doses  
•   Azithromycin:  30  mg/kg  as  a  single  dose  OR  10  mg/kg  QD  x  3  days  OR  10  mg/kg  X  1  day  
then  5  mg/kg  on  days  2-­‐‑5  
Prophylaxis  for  recurrent  AOM  
•   Amoxicillin:  half  daily  dosage  at  bedtime  
•   SMX-­‐‑TMP:  40  mg/kg  SMX  and  8  mg/kg  TMP  QHS  
   
What  is  the  treatment   1)   Neomycin  Solutions:  3-­‐‑4  drops  QID  X  7  days  à  Adverse  effects  include  rupture  of  
strategy  for  acute  otitis   tympanic  membrane,  potential  ototoxity  with  ruptured  TM    
externa?   2)   Ofloxacin  Solutions:  for  children  1-­‐‑12  years,  use  5  drops  BID  X  10  days;  for  patients  ≥12  
years  old,  use  10  drops  BID  X  10  days  

  Swimmers  ear  is  a  form  of  recurrent  or  chronic  otitis  externa  caused  by  chronic  irritant  fluid  
  accumulation  in  the  acoustic  canal,  such  as  can  occur  in  competitive  swimmers.  Often,  this  is  more  
What  is  the  pathogenesis   of  inflammatory  etiology  than  infective    
of  “swimmers  ear”?  What    
is  the  treatment  strategy?   The  use  of  topical  astringent  drops,  such  as  acetic  acid,  sometimes  combined  with  topical  steroids  
such  as  hydrocortisone  are  effective  along  with  efforts  to  clear  water  from  the  canal  when  drying  
off.    
 
 
 
Family  Medicine  Textbook  Notes     53  
 
  Type  of   Primary   Clinical  Presentation   Key  Labs  and   Prognosis  
  Dementia   Anatomic   Radiological  
  Location   Findings  
  Normal  Aging   None   Occasionally  forgetful  of  names   Mild  generalized   Good  
      cortical  atrophy,  
nonspecific  white  
  matter  changes  
  Mild  Cognitive   Medial   Impaired  short-­‐‑term  memory  for  events   Variable  medial   Increased  risk  of  
  Impairment   Temporal   temporal  lobe   Alzheimer’s  
atrophy  
What  are  the  common   Delirium   Cortical/   Often  toxic/  metabolic/  infectious   EEG  slowing,   Depends  on  
diagnoses  among  older   Subcortical   etiology;  Impaired  attention,  may  be   evidence  of  drug  or   etiology  and  
persons  with  cognitive   fidgety  and  tremulous,  or   metabolic  toxicity,   severity  
apathetic/obtunded   signs  of  infection  
complaints?  What  are  the   Alzheimer   Cortical   Gradually  progressive  short-­‐‑term   Medial  temporal   Course  4-­‐‑20  years  
clinical  and  lab  findings?   Disease   (temporal   memory  and  other  cognitive  deficits;   and  parietal  lobe   (average  8  years)  
Prognosis?   and  parietal)   Generally  normal  neurological  exam   atrophy  
(may  be  apraxic)  
Vascular   Cortical   Often  a  history  of  multiple  stroke-­‐‑like  
Evidence  of   Course  static  or  
Dementia   and/or   events  and/or  vascular  risk  factors;   significant   progressive  (often  
subcortical   Variable  physical  exam   cerebrovascular   coexists  with  AD)  
disease    
Frontotemporal   Cortical   Typically  presents  with  change  in   Imaging  may  show   Variable,  
Dementia   (frontal  and   behavior/personality/  language;  variable   severe  atrophy  in   progressive,  
temporal)   physical  exam  findings   frontal/temporal   speech/swallowing  
difficulties  
Lewy  Body   Cortical  and   Fluctuating  attention,  visual   No  specific  brain   Variable,  may  see  
Dementia   subcortical   hallucinations,  parkinsonian  motor   imaging  features   rapid  functional  
signs,  sleep  disturbance;  limb  rigidity,   decline    
bradykinesia,  may  see  intention  tremor  
and  gait  disturbance  
Parkinson   Subcortical   Parkinson  disease  with  later-­‐‑onset   No  specific  brain   Variable,  have  
Dementia   cognitive  dysfunction;  limb  rigidity,   imaging  features   severe  motor  
bradykinesia,  resting  tremor,  gait   disability  when  
 
disturbance   dementia  occurs  
Family  Medicine  Textbook  Notes     54  
 
    •   The  earliest  symptoms  in  AD  are  difficulty  with  higher  cognitive  functions  such  as  
What  is  the  initial   memory,  language,  problem  solving,  and  reasoning  
presentation  for  a  patient   •   Memory  loss  is  usually  prominent  early  in  the  disease  and  typically  progresses  over  time  
with  Alzheimer’s   •   Also,  patients  with  early  AD  are  often  disproportionately  impaired  in  category  fluency  
Disease?   (e.g.,  naming  as  many  animals  as  they  can  in  a  minute)  compared  with  letter  fluency  (e.g.  
naming  words  that  begin  with  F)  
  •   MCI  is  present  when  cognitive  function  is  impaired  more  than  one  would  expect  based  on  
What  are  the  findings  in   the  individual’s  age  and  education  level,  but  is  not  severe  enough  to  interfere  with  activities  
a  patient  with  mild   of  daily  living  
cognitive  impairment?   •   Subtypes  
-­‐‑   Amnestic  MCI:  patient  has  isolated  memory  loss;  high  risk  of  progression  to  AD  
-­‐‑   Nonamnestic  MCI:  patient  has  impairment  in  other  areas  than  memory  
  Mini-­‐‑Cog  
  •   3-­‐‑minute  instrument  to  screen  for  cognitive  impairment  in  older  adults  
  •   Uses  a  3-­‐‑item  recall  test  for  memory  and  a  simply  scored  clock-­‐‑drawing  test  (which  serves  
  as  an  “informative  distractor”)  
Which  cognitive   •   Give  the  patient  3  unrelated  words  to  remember  and  make  sure  they  repeat  them  back  to  
screening  tests  are   you  à  Ask  the  patient  to  draw  a  clock  face  placing  all  of  the  numbers  and  hands  in  the  
recommended  for  use  in   correct  position  à  Ask  the  patient  to  recall  the  items  
primary  care?   •   Scoring:  1  point  for  each  word  remembered,  2  points  for  a  normal  clock  drawing  
•   Sensitivity  =  76%  (Mini  Mental  Status  Exam  has  Sn  =  79%)  
•   Specificity  =  89%  (MMSE  has  Sp  =  88%)  
AD-­‐‑8  
•   8-­‐‑question,  rapid  screening  test  for  cognitive  impairment  that  is  administered  to  a  family  
member.  Monitors  changes  in  mental  status  observed  by  family.  
•   Sensitivity  >  84%;  Specificity  >80%;  PPV  >  85%  
Family  Medicine  Textbook  Notes     55  
 
 
 
 
 
What  is  the  primary  care  
approach  to  the  detection  
and  subsequent  
evaluation  of  dementia?  

 
 
   
 
 
What  are  the  strategies  
for  the  prevention  of  
dementia?  

 
Family  Medicine  Textbook  Notes     56  
 
  Mild  Cognitive  Impairment  
  •   Cognitive  enhancers  (donepezil,  memantine)  (Level  B)  
  Dementia:  Cognitive  Symptoms  
  •   Cholinesterase  inhibitors  (donepezil,  rivastigmine,  galantamine,  memantine)  (Level  A)  
What  are  the  key   •   Cognitive  training  (Level  B)  
treatment  strategies  for   •   Physical  Activity  (Level  B)  
the  common  symptoms  in   Caregiver  stress,  burden,  and  depression  
patients  with  dementia   •   Caregiver  support  group  programs  and  educational  programs;  sue  of  respite  services  (level  
and  related  disorders?   A)  
Depression  in  people  with  dementia  
•   SSRIs  (sertraline,  citalopram)  (Level  A)  
•   Physical  activity/exercise  (Level  B)  
Agitation,  aggression,  and  delusions  in  people  with  dementia  
•   Caregiver  training  in  dementia  management  skills  (Level  A)  
•   Antipsychotic  drugs  (Level  A)  
•   Anticonvulsants  (Carbamazepine,  Valproic  Acid)  (Level  B)  
•   Cognitive  Enhancers  (donepezil,  memantine)  (Level  A)  
•   Environmental  modification  (music  reduces  agitation,  aggression,  and  mood  disturbance)  
(Level  B)  
Physical  Activity  (Level  A)  
   
What  is  the   •   Cyclic  mastalgia  (bilateral  pain  varying  in  intensity  throughout  the  menstrual  cycle  with  
pathophysiology  of  cyclic   the  premenstrual  time  often  the  most  painful)  is  thought  to  be  hormonally  mediated.  
and  non-­‐‑cyclic  breast   However,  studies  investigating  circulating  levels  of  progesterone,  estrogen,  prolactin,  and  
pain?   quantity/sensitivity  of  hormone  receptors  have  yielded  conflicting  results.      
•   Noncyclic  mastalgia  (usually  unilateral)  typically  occurs  in  women  over  the  age  of  40.  
Family  Medicine  Textbook  Notes     57  
 
  Cyclic  
  •   Patients  usually  <  40  YO  
  •   Most  severe  premenstrually  and  subsides  during  menses  
Compare  the  clinical   •   Usually  bilateral,  in  the  upper  outer  breast  quadrants,  and  a/w  nodularity  
presentations  for  cyclic   •   Described  as  dull,  aching,  or  heavy  
and  noncyclic  breast   Noncyclic  
pain:   •   Patients  usually  >  40  YO  
•   No  temporal  relationship  to  the  menstrual  cycle  
•   Usually  unilateral  
•   Often  described  as  sharp,  burning,  or  drawing  
•   More  commonly  located  in  the  subareolar  or  medial  portion  of  the  breast  
  Cyclic  
  •   Proper-­‐‑fitting  brassiere  
What  are  the  treatment   •   Evening  primrose  oil  (1  gram  every  8  hours)  (Level  B)  
strategies  for  cyclic  and   •   Topical  NSAID  (50  mg  diclofenac  gel  every  8  hours)  (Level  A)  
noncyclic  breast  pain?   •   Danazol  (50-­‐‑100  mg  every  12  hours)  (Level  A)  
•   Bromocriptine  (1.25-­‐‑2.5  mg  orally  at  bedtime)  (level  B)  
•   Tamoxifen  (short-­‐‑term  use  only)  (10  mg  orally  per  day)  (Level  A)  
•   Goserelin  (3.6  mg  subcutaneous  per  month)  (Level  B)  
Noncyclic  
•   Topical  NSAID  (50  mg  diclofenac  gel  every  8  hours)  (Level  A)  
•   Evening  primrose  oil  (1  gram  every  8  hours)  (Level  B)  
Family  Medicine  Textbook  Notes     58  
 
 
 
 
 
 
 
 
 
 
 
What  is  the  clinical  
algorithm  for  
management  of  nipple  
discharge?  

 
Family  Medicine  Textbook  Notes     59  
 
 
 
 
 
What  are  the  key  
elements  of  the  history  
and  physical  exam  for  
nipple  discharge?  

 
  Physiologic  breast  secretions  
  •   Related  to  hormonal  influences  (i.e.  prolactin)  on  breast  tissue  and  are  typically  bilateral,  involve  
multiple  ducts,  and  require  some  form  of  manipulation  to  be  expressed  
What  are  the  
•   Galactorrhea  (milky  discharge)  is  most  common  
pathophysiologic  causes  
•   Precipitators  include  nipple  stimulation,  sexual  orgasm,  sleep,  exercise,  and  food  ingestion  
of  nipple  discharge?   Pathologic  breast  secretions  
•   Pathologic  reasons  for  increases  in  prolactin  include  hypothalamic  lesions,  pituitary  tumors,  chest  wall  
trauma,  hypothyroidism,  renal  failure  (decreased  prolactin  clearance),  and  anovulatory  syndromes,  such  
as  polycystic  ovaries.  
•   Pathologic  discharge  can  originate  either  from  the  nipple  and  areola  region  or  from  a  breast  duct.  
•   Eczema,  nipple  adenoma,  and  Paget  disease  can  cause  erythema  and  ulceration  of  the  nipple  skin,  with  
an  associated  bloody  discharge  
•   Ductal  diseases  associated  with  nipple  discharge  are  duct  ectasia  (periductal  mastitis),  duct  papilloma,  
and  early  ductal  carcinoma  
•   Infection  and  abscess  can  lead  to  a  purulent  discharge  
Family  Medicine  Textbook  Notes     60  
 
 
 
 
 
 
What  are  the  red  flags  
suggestive  of  breast  
cancer?  

 
 
 
What  are  the  key  element  
of  the  history  and  
physical  exam  when  a  
woman  has  a  palpable  
breast  mass?  

 
Family  Medicine  Textbook  Notes     61  
 
 
 
 
 
 
 
 
 
 
 
What  is  the  algorithm  for  
the  management  of  a  
palpable  breast  mass?  

 
Family  Medicine  Textbook  Notes     62  
 
What  is  the  timeframe  for   •   The  ovum  is  only  able  to  be  fertilized  for  12-­‐‑24  hours  after  ovulation  
egg  fertilization   •   Sperm  remain  viable  for  3  days  after  intercourse  
following  ovulation?   •   The  most  fertile  period  for  women  is  the  several  days  before  ovulation  and  ends  24  hours  
How  long  can  sperm   after  ovulation  
remain  viable  in  a   •   After  the  egg  is  fertilized,  it  is  transported  to  the  uterine  cavity  in  about  2-­‐‑3  days  
woman’s  body?   •   Implantation  occurs  approximatly  6-­‐‑7  days  after  fertilization  following  cell  division  that  
forms  a  blastocyst  
  Description  
  •   Contains  two  hormones:  an  estrogen  and  a  progestin  
-­‐‑   In  most,  the  estrogen  is  ethinyl  estradiol  20-­‐‑35  mg  
 
-­‐‑   Progestins  vary  by  selectivity  à  there  are  9  progestins  used  
For  combined  oral  
•   OCs  are  described  as  monophasic  (same  dose  of  estrogen  and  progestin  in  each  pill)  or  multiphasic  
hormonal  contraceptives   (amounts  of  hormones  in  tablets  can  vary  across  the  cycle/month)  
(“The  Pill”),  describe  the   MOA  
components,  mechanism   •   Primary  MOA:  suppression  of  ovulation  
of  action,  efficacy,   •   Additional  contraceptive  actions  are  due  to  progestins  à  creation  of  a  thickened  cervical  mucus  that  
benefits,  and  side  effects:   blocks  sperm  from  the  uterus,  inhibition  of  capacitation  (ability  of  the  sperm  to  fertilize  an  egg),  
interference  with  transport  of  the  egg  and  sperm,  and  endometrial  changes  that  interfere  with  
 
implantation  
Efficacy  
•   All  OCs  are  equally  effective  (99%  effective  when  used  properly)  
Benefits  
•   Protection  against  ovarian,  endometrial,  and  colorectal  cancer,  benign  breast  tumors,  ovarian  cysts,  
dysmenorrhea,  and  blood  loss  
•   Reduces  acne  
•   Suppressed  endometriosis  
•   Treats  hot  flashes  
Side  Effects  
•   Amenorrhea,  spotting  and  breakthrough  bleeding,  nausea,  acne,  breast  pain/tenderness,  increased  
vaginal  discharge,  melisma,  decreased  libido,  Venous  thromboembolism,  risk  of  MI  or  stroke  (<2  
events/100,000  women  years),  Hyperkalemia,  Benign  hepatic  tumor  
Family  Medicine  Textbook  Notes     63  
 
 
 
 
 
 
What  are  the  
contraindications  and  
potential  risks  of  using  
combined  OCPs?  
 

Describe  the  mechanism,   •   Ortho  Evra  is  a  thin  transdermal  patch  that  contains  ethinyl  estradiol  and  norelgestromin  
use,  benefits,  and  side   •   Inhibition  of  ovulation  is  the  primary  mechanism  of  action  
effects  of  the  transdermal   •   It  is  believed  to  have  similar  efficacy,  health  benefits,  health  risks,  and  drug  interactions  as  
patch  for  contraception   the  OCP,  although  patches  may  be  less  effective  for  women  who  weigh  more  than  198  
(Ortho  Evra):   pounds  
•   Use:  the  patch  should  be  applied  once  a  week  for  3  weeks,  followed  by  a  patch-­‐‑free  week.  
The  patch  should  be  applied  to  clean  dry  skin  of  the  upper  outer  arms,  abdomen,  buttocks,  
or  the  torso  (except  the  breasts).  When  a  patch  is  removed,  the  new  patch  should  be  applied  
to  a  different  area  of  the  body.  
Family  Medicine  Textbook  Notes     64  
 
  •   NuvaRing  is  a  soft,  flexible  and  transparent  ring  that  releases  ethinyl  estradiol  and  
Describe  the  mechanism,   etonogestrel  (active  metabolite  of  desogestrel)  
use,  benefits,  and  side   •   The  efficacy,  drug  interactions,  side  effects,  and  risks/benefits  are  thought  to  be  similar  to  
effects  of  the  vaginal  ring   the  OCP  
for  contraception   •   Important  to  remember  that  the  vaginal  ring  should  be  kept  refrigdrated  until  use  
(NuvaRing)   •   Use:  the  vaginal  ring  should  be  compressed  and  inserted  high  into  the  vagina  (exact  
  placement  not  necessary).  One  ring  is  left  in  the  vagina  for  3  weeks.  It  is  NOT  removed  for  
sexual  intercourse.  At  the  end  of  3  weeks,  it  is  removed  and  there  is  a  ring-­‐‑free  week.  
•   Use  of  a  backup  method  of  contraception  is  necessary  for  the  first  7  days  the  ring  is  inserted  
unless  the  woman  has  switched  from  another  hormonal  method  of  contraception.  
  Description  
  •   There  are  3  progestin-­‐‑only  contraceptives:  the  progestin-­‐‑only  pill  (POP,  also  known  as  the  
  minipill),  DMPA  medroxyprogesterone  injections,  and  subdermal  implants  (Implanon)        
For  Progestin-­‐‑only   MOA  
Contraceptives,  describe   •   Must  take  POP  at  the  same  time  every  day  in  order  to  produced  a  thickened  cervical  mucus  
the  components,   •   DMPA  is  a  long-­‐‑acting  contraceptive  administered  via  injection  every  3  months  
mechanism  of  action,   •   Implanon  is  a  single  contraceptive  rod  that  is  inserted  subdermally  in  the  groove  b/w  the  biceps  
efficacy,  benefits,  and   and  triceps  in  the  non-­‐‑dominant  arm.  It  provides  continuous  contraception  for  3  years.    
Efficacy  
side  effects:  
•   >  99%  effective  if  used  properly  
 
Benefits  
•   The  progestin-­‐‑only  contraceptives  have  the  advantage  of  eliminating  exposure  to  estrogen  
making  these  methods  useful  for  women  who  have  contraindications  to  the  use  of  estrogen.  
They  are  also  more  suitable  for  women  who  want  to  breastfeed  as  they  do  not  impair  lactation.  
•   Protection  against  ovarian  and  endometrial  cancer  and  PID  
•   Reduced  risk  of  PID  and  VTE  vs  estrogen-­‐‑containing  OCPs  
Side  Effects  
•   Menstrual  changes  (unpredictable,  frequent  or  prolonged  bleeding)  and  amenorrhea  
•   Narrow  margin  for  error  for  contraceptive  efficacy  (late  or  missed  pills)  
Family  Medicine  Textbook  Notes     65  
 
 
 
 
What  are  the  
contraindications  and  
potential  risks  of  using  
progestin-­‐‑only  
contraceptives?  
 

 
  Description  
For  Hormonal  Emergency   •   Available  over  the  counter  for  women  aged  17+;  prescription  needed  for  ≤  16  YO  
Contraceptives  (Plan  B),   •   Used  after  intercourse  to  prevent  pregnancy  
describe  the  components,   •   There  are  two  commonly  used  ECs:  1)  levonorgestrel  2)  OCs  containing  ethinyl  estradiol  plus  
mechanism  of  action,   either  levonorgestrel  or  norgestrel  
efficacy,  benefits,  and   MOA  
side  effects:   •   Inhibits  the  mid-­‐‑cycle  LH  surge  and  thus  ovulation  
  •   If  given  at  least  2  days  prior  to  ovulation,  prevention  or  delay  of  ovulation  occurs  
•   If  ovulation  occurs,  the  contraceptive  action  is  prevention  of  fertilization  
•   The  EC  does  NOT  impair  the  development  of  an  embryo  or  disrupt  an  already  established  
pregnancy  
Efficacy  
•   Efficacy  is  highly  dependent  on  timing  
•   0.5%  pregnancy  rate  if  taken  within  12  hours  but  4.1%  if  taken  61-­‐‑72  hours  after  unprotected  sex  
SEs  
•   Nausea,  vomiting  
Family  Medicine  Textbook  Notes     66  
 
 
 
 
 
 
 
 
Compare  and  contrast  the  
progestin-­‐‑only  
contraceptive  options:  
 

 
  Description  
  •   There  are  two  types  of  IUDs:  the  Copper  T-­‐‑380A  and  the  levonorgestrel  intrauterine  system  
  MOA  
  •   Action  is  to  prevent  fertilization  of  the  egg  
For  Intrauterine  Devices   •   The  TCu380A  alters  tubal  and  uterine  fluids  thus  impairing  sperm  function  and  preventing  
(IUDs)  describe  the   fertilization  
components,  mechanism   •   The  LNG-­‐‑IUS  has  several  actions  including  thickening  of  cervical  mucus,  inhibiting  sperm  
of  action,  insertion   capacitation  and  survival,  and  suppression  of  the  endometrium  
techniques,  efficacy,   •   IUDs  are  NOT  abortifacients  
benefits,  and  side  effects:   Efficacy  
  •   Highly  effective  with  very  low  failure  rates  
   
   
Family  Medicine  Textbook  Notes     67  
 
  Insertion  
For  Intrauterine  Devices   •   IUDs  should  be  inserted  slowly  and  gently  
(IUDs)  describe  the   •   IUDs  can  be  placed  at  any  time  during  the  menstrual  cycle,  provided  the  woman  is  reasonably  
components,  mechanism   certain  she  is  not  pregnant,  as  well  as  immediately  following  childbirth  (within  10  minutes  of  
of  action,  insertion   expulsion  of  the  placenta),  4-­‐‑6  weeks  postpartum  in  a  woman  who  is  breastfeeding  and  has  no  
techniques,  efficacy,   menses,  and  immediately  after  or  up  to  3  weeks  after  a  first  trimester  abortion  
benefits,  and  side  effects,   Benefits  
continued   •   Protection  against  endometrial  cancer  
•   Decreased  menstrual  blood  loss  and  pain  
SEs  
•   Menstrual  cramping  and  increased  or  irregular  bleeding  
 
 
 
What  are  the  
contraindications  and  
potential  risks  of  using  
IUDs?  
 

 
Family  Medicine  Textbook  Notes     68  
 
  Description  
  •   The  diaphragm  is  a  dome-­‐‑shaped  reusable  barrier  contraceptive  made  of  latex  
  MOA  
  •   Physical  barrier  covering  the  cervix  with  or  without  spermicide  
  Efficacy  
For  contraceptive   •   94%  effective  if  used  correctly  
diaphragms  describe  the   Fitting  and  Insertion  
components,  mechanism   •   3  basic  types:  arcing,  coil,  and  flat  spring  à  most  use  the  arcing  rim  style  
of  action,  insertion   •   Spermicide  is  applied  to  the  diaphragm  so  that  the  cup  holding  the  spermicide  faces  the  
techniques,  efficacy,   cervix  
benefits,  and  side  effects:   •   Inserted  by  pinching  the  sides  together  and  gently  inserting  it  into  the  vagina  with  a  small  
  amount  of  lubricant  on  the  leading  edge.  Once  inserted,  it  should  cover  the  cervix  and  fit  
snugly  but  comfortably  in  place  
Benefits  
•   Protection  against  some  STIs  
•   Reduced  cervical  cancer  risk  
•   Reversible  and  inexpensive    
SEs  
•   Vaginal  and  UTI  risk  
•   Toxic  Shock  Syndrome  risk  
•   Latex  allergies  
•   Cervical  irritation  
Family  Medicine  Textbook  Notes     69  
 
 
What  are  the  
contraindications  and  
potential  risks  of  using  
diaphragms?  
   

  Description  
  •   The  contraceptive  sponge  is  a  soft,  one-­‐‑size  polyurethane  foam  device  containing  the  
For  contraceptive   spermicide  nonoxynol-­‐‑9  à  single  use  
sponges,  describe  the   MOA  
components,  mechanism   •   Spermicide  +  sponge  absorption  of  sperm  +  physical  blocking  of  sperm  
of  action,  efficacy,   Efficacy  
benefits,  and  side  effects:   •   More  effective  for  women  who  have  never  had  a  child  (16%  failure  rate  for  nulliparous  
  women  vs  32%  for  parous)  
Benefits  
•   Possible  protection  against  some  STI  
•   Low-­‐‑cost  
•   Non-­‐‑prescription  
SEs  
•   Difficulty  with  removal  
•   Increased  vaginal  infection  and  UTI  and  toxic  shock  risk  (if  used  while  menstruating)  
Family  Medicine  Textbook  Notes     70  
 
  MOA  
For  spermicide,  describe   •   Contains  the  agent  nonoxynol-­‐‑9,  which  is  a  surfactant  that  destroys  the  sperm  cell  
the  components,   membrane  
mechanism  of  action,   Efficacy  
efficacy,  benefits,  and   •   Although  not  highly  effective,  efficacy  is  dependent  on  use  of  an  adequate  amount  of  
side  effects:   spermicide  
  Benefits  
•   Ease  of  use  
•   Non-­‐‑prescription  
•   Low  cost  
•   Lubrication  
SEs  
•   Skin,  vaginal,  or  penile  allergy  or  sensitivity  
•   Messy  
•   May  cause  skin  irritation  or  tiny  abrasions  (increases  risk  of  STI)  
What  are  the    
contraindications  and  
potential  risks  of  using  
spermicide  and  latex  
male  condoms?  
   
Family  Medicine  Textbook  Notes     71  
 
   
What  labs  should  be   •   Recommended  lab  tests  are  hemoglobin  or  hematocrit  and  blood  typing  
ordered  prior  to  a  medical   •   Anti-­‐‑D  immune  globulin  should  be  administered  if  indicated  
abortion?   •   Confirmation  of  pregnancy  by  ultrasound  or  pregnancy  testing  is  necessary  
•   Follow  up  in  1-­‐‑2  weeks    
Family  Medicine  Textbook  Notes     72  
 
 
 
 
 
 
 
Compare  and  contrast  the  
different  medical  
abortion  regimens:  

 
 
Family  Medicine  Textbook  Notes     73  
 
  •   Most  UTIs  are  caused  by  bacteria  that  normally  inhabit  the  colon  à  80  to  90%  of  
  community-­‐‑acquired  UTIs  in  adults  and  children  are  from  E.  coli  
What  is  the  most  common   •   Other  gram-­‐‑negative  organisms  (Proteus  species,  Klebsiella  pneumoniae,  and  Pseudomonas  
pathophysiology  of  a   aeruginosa)  cause  infections  but  are  much  less  common  except  among  people  who  are  
UTI?   hospitalized,  live  in  long-­‐‑term  care  facilities,  are  immunocompromised,  or  have  undergone  
recent  genitourinary  catheterization  or  instrumentation  
•   The  usual  route  of  infection  involves  bacteria  invading  the  bladder  by  ascending  from  the  
perineum  and  passing  through  the  urethra  
 
 
 
What  is  the  differential  
diagnosis  for  dysuria  in  
an  otherwise  healthy  
woman  of  reproductive  
age  and  in  older  women?    

 
Family  Medicine  Textbook  Notes     74  
 
 
 
 
What  are  the  risk  factors  
for  acute  pyelonephritis  
in  healthy  women?  

 
 
 
 
What  are  the  red  flags  for  
a  complicated  urinary  
tract  infection?  

 
Family  Medicine  Textbook  Notes     75  
 
  History  
  •   Patients  with  UTIs  typically  have  urinary  frequency,  nocturia,  pain  on  urination,  and  
Describe  the  history  and   suprapubic  discomfort  
physical  exam  for  a   •   Malodorous  urine  and  cloudy  urine  color  both  increase  the  likelihood  of  a  UTI  
patient  presenting  with   •   Chills,  rigors,  fever,  nausea,  vomiting,  and  flank  pain  are  all  specific  for  pyelonephritis  
dysuria:   Physical  Exam  
•   Need  to  obtain  vital  signs,  palpate  the  mid  and  lower  abdomen,  and  percuss  the  flanks  of  
the  patient  
•   Tenderness  over  a  flank  or  in  the  mid  abdomen  suggest  upper  tract  disease  
•   In  men,  the  penis  should  be  gently  milked  to  elicit  a  urethral  discharge,  and  a  rectal  exam  
performed  to  feel  for  a  tender  or  boggy  prostate,  which  suggest  urethritis  and  prostatitis,  
respectively  
•   In  women  who  report  vaginal  discharge  or  irritation,  perform  a  vaginal  exam  
  Dry  Reagent  Test  Strip  (Dipstick)  
  •   Detects  blood,  nitrite,  and  leukocyte  esterase  
Describe  urinalysis  for  a   -­‐‑   Blood  is  detected  using  the  peroxidase-­‐‑like  activity  of  hemoglobin  in  th  eurine  
patient  presenting  with   -­‐‑   The  leukocyte  esterase  test  detects  the  presence  of  an  esterase  from  WBCs  and  is  
dysuria:   positive  in  75%  of  UTIs  à  sensitivity  (0.87)  >  specificity  (0.36)  
-­‐‑   Nitrite  is  found  in  the  urine  when  dietary  nitrates  are  excreted  in  the  urine  and  
converted  to  nitrite  by  bacteria  à  this  test  is  not  very  sensitive  (0.53),  but  it  is  very  
specific  (0.88)  
Sediment  Microscopy  
•   Used  to  look  for  white  cells,  red  cells,  bacteria,  and  white  cell  casts  
-­‐‑   There  is  disagreement  about  how  many  WBCs  need  to  be  identified  in  order  to  
diagnose  UTI  
-­‐‑   However,  the  presence  of  bacteria  is  highly  suggestive  of  UTI  à  Sn  =  0.95,  Sp  =  0.85  
Family  Medicine  Textbook  Notes     76  
 
 
 
 
 
 
 
 
 
 
 
 
Algorithm  for  the  
management  of  dysuria  

 
Family  Medicine  Textbook  Notes     77  
 
 
 
 
 
 
 
What  interventions  are  
recommended  for  
management  of  
uncomplicated  lower  
UTIs,  uncomplicated  
upper  UTIs,  and  in  the  
prevention  of  UTIs?  

 
Family  Medicine  Textbook  Notes     78  
 
 
 
 
 
 
 
What  is  the  initial  drug  
therapy  strategy  for  lower  
UTIs?  

 
Family  Medicine  Textbook  Notes     79  
 
 
 
 
 
 
 
 
What  is  the  initial  drug  
therapy  strategy  for  
upper  UTIs?  

 
  •   A  more  extensive  evaluation  is  warranted  in  men  with  dysuria  because  they  are  more  likely  to  have  
  a  complicated  infection  
What  is  the  treatment   •   As  with  women,  men  should  be  identified  as  having  either  an  upper  tract  or  a  lower  tract  infection  
strategy  when  an  adult   •   Other  causes  of  dysuria,  including  prostatitis  and  urethritis  need  to  be  excluded  
•   Initial  treatment  for  men  with  suspected  lower  UTI  =  fluoroquinolone  
man  presents  with  
•   Treatment  for  men  with  pyelonephritis  =  fluoroquinolone  for  14-­‐‑21  days  
dysuria?  
•   After  a  2nd  lower  UTI  or  a  1st  episode  of  pyelonephritis,  adult  male  patients  should  undergo  imaging  
to  identify  an  anatomic  abnormality  or  nephrolithiasis  à  ultrasonography  and  plain  abdominal  
radiograph  appear  comparable  to  IV  pyelogram  as  the  initial  imaging  study.  
Family  Medicine  Textbook  Notes     80  
 
  Epidemiology  
  •   Seen  more  often  in  minority  populations  and  in  women,  especially  young  women  (<  24  YO)  
  Clinical  Evaluation  
  •   Women  can  prevent  with  mucoid  vaginal  discharge  without  odor,  abnormal  menstrual  bleeding,  
and  lower  abdominal  pain.  
 
-­‐‑   Physical  findings  include  cervicitis  with  yellow  or  cloudy  mucoid  discharge  from  the  cervical  
 
os.  
  -­‐‑   The  cervix  is  often  friable  (tends  to  bleed  easily)  when  touched  with  a  swab  or  spatula  
  •   Men  most  commonly  present  with  urethritis  with  associated  dysuria,  perimeatal  tingling,  and  a  
  clear-­‐‑to-­‐‑white  urethral  discharge  
Chlamydia   Diagnostic  Testing  
  •   Wet  mount  and  the  amine  test  (significant  odor  release  with  addition  of  potassium  hydroxide  to  
  vaginal  secretions)  on  vaginal  discharge  helps  differentiate  chlamydia  infection  from  other  
  infections  such  as  UTI,  bacterial  vaginosis,  and  trichomoiasis  
  -­‐‑   In  the  wet  mount,  the  presence  of  >  10  WBCs  per  high-­‐‑power  field  is  a  predictor  of  
  endocervical  infection  
•   C.  trachomatis  infections  can  be  detected  using  culture  of  epithelial  cells  (b/c  it  is  obligate  
 
intracellular)  and  non-­‐‑culture  techniques.    
 
•   Culture  is  the  gold  standard.  The  Nucleic  Acid  Amplification  Tests  (NAAT)  have  replaced  the  
  chlamydia  culture  recently.  
  Treatment  
  •   For  men  and  non-­‐‑pregnant  women:  
  -­‐‑   Azithromycin  1  g  orally  single  dose  
  -­‐‑   Doxycycline  100  mg  orally  2x/day  for  7  days  
  -­‐‑   Alternatively,  could  use  Erythromycin  (500  mg  orally  4x/day  for  7  days),  Erythromycin  
  Ethylsuccinate  (800  mg  orally  4x/day  for  7  days),  Ofloxacin  (300  mg  orally  2x/day  for  7  days),  
  or  Levofloxacin  (500  mg  orally  once/day  for  7  days)  
  •   For  pregnant  women:  
-­‐‑   Azithromycin  1  g  orally  single  dose  
 
 
Family  Medicine  Textbook  Notes     81  
 
  -­‐‑   Alternatively,  could  use  Amoxicillin  (500  mg  3x/day  for  7  days),  Erythromycin  (500  mg  
  4x/day  for  7  days  or  250  mg  4x/day  for  14  days),  or  EES  (800  mg  4x/day  for  7  days  or  400  mg  
  4x/day  for  14  days)  
  Follow-­‐‑Up  
•   CDC  does  NOT  recommend  testing  for  cure  of  chlamydia  after  completion  of  the  antibiotics,  except:  
 
-­‐‑   In  those  with  persistent  or  recurring  symptoms  
 
-­‐‑   In  non-­‐‑adherent  patients  
Chlamydia,  continued   -­‐‑   In  patients  treated  with  an  alternative  regiment  
-­‐‑   In  pregnant  women  
•   The  majority  of  post-­‐‑treatment  infections  result  from  reinfection  because  the  patient’s  sex  partners  
were  not  treated  or  the  patient  resumed  sex  with  a  new  infected  partner.  
•   Recommended  to  rescreen  patients  3-­‐‑4  months  after  completion  of  antibiotics  
Screening  and  Prevention  
•   Behavioral  changes  recommended:  delaying  the  age  of  first  intercourse,  reducing  the  number  of  
sexual  partners,  using  barrier  contraception  
•   Annual  screening  of  sexually  active  women  ≤  25  YO  and  sexually  active  older  women  with  risk  
factors  
•   All  pregnant  women  at  risk  (including  those  ≤  25  YO)  should  be  screened  at  the  first  prenatal  visit  
and  again  during  the  3rd  trimester    
•   Routine  screening  of  men  is  appropriate  in  settings  of  high  prevalence,  such  as  correctional  facilities  
and  STD  clinics  
Chlamydia  Infections  in  Children  
•   Consider  chlamydia  in  all  children  ≤  30  days  old  who  have  conjunctivitis  à  present  within  5-­‐‑12  
days  of  birth  with  swelling  and  mucopurulent  drainage  of  one  or  both  eyes  
•   Treatment  for  ophthalmia  neonatorum:  Erythromycin  base  or  ethylsuccinate  50  mg/kg/day  orally,  
divided  into  4  doses  per  day,  for  14  days  
•   Chlamydia  pneuomia  can  develop:  staccato  cough  usually  with  no  wheezing  and  no  temperature  
elevation  
Complications  
•   PID  in  20-­‐‑40%  of  untreated  women  
•   Chronic  prostatitis    &  Reiter  Syndrome  (reactive  arthritis,  urethritis,  conjunctivitis)    
Family  Medicine  Textbook  Notes     82  
 
  Epidemiology  
  •   2nd  most  common  reportable  bacterial  STI  in  the  US  
  •   Gram-­‐‑negative,  intracellular  diplococcus  
  •   Highest  reported  rates  are  seen  among  adolescents  and  young  adults  (women  aged  15-­‐‑19  
  and  men  aged  20-­‐‑24),  minorities  (20  times  greater  frequency  in  blacks  than  whites),  MSM,  
  and  people  in  southeastern  USA  
  •   Acquired  both  sexually  and  vertically  
  Clinical  Symptoms  
  •   Incubation  period  =  2-­‐‑6  days  
  •   Often  asymptomatic  
  •   Symptomatic  infection  presents  as  vaginal  pruritus,  postcoital  bleeding,  deep  dyspareunia,  
Gonorrhea   and/or  odorless  mucopurulent  discharge.  
  •   One  exam,  the  cervix  may  be  friable  with  mucuopurulent  d/c  from  the  cervical  os  
  •   Men:  urethritis  with  dysuria  and  yellowish  penile  d/c  
  •   Penis  may  be  erythematous  with  a  purulent  d/c  at  the  meatus  
  Diagnostic  Testing  
  •   As  for  chlamydia,  NAAT  testing  has  replaced  culture  
  •   NAAT  is  not  approved  for  use  of  testing  female  urine,  because  female  urine  has  high  levels  
  of  inhibitory  substances  that  can  impair  test  performance  
  Treatment  
  •   Cephalosporins:  single  dose  125  mg  ceftriaxone,  or  single  dose  400  mg  cefixime  
  •   Because  dual  infection  with  chlamydia  is  common  (10-­‐‑30%),  you  should  treat  for  both  
  chlamydia  and  gonorrhea    
   
   
   
   
   
Family  Medicine  Textbook  Notes     83  
 
  Follow-­‐‑Up  
  •   Patients  treated  for  uncomplicated  gonococcal  infection  do  not  require  a  test  of  cure  
  •   If  symptoms  persist  or  recur  shortly  after  treatment,  a  test  of  cure  is  indicated  by  culture  to  
  verify  susceptibility  and  direct  retreatment  
  •   Pregnant  women:  test  for  cure  after  3  weeks  following  treatment  
  •   Rescreening  should  be  performed  within  3  months  of  treatment  for  all  patients  
Gonorrhea,  continued   Screening  
•   Routine  annual  screening  is  recommended  for  all  sexually  active  women  ≤  25  YO  or  for  
older  women  at  increased  risk  of  infection  
•   All  pregnant  women  should  be  routinely  screened  at  the  first  prenatal  visit.  If  there  are  risk  
factors,  rescreening  is  advised  in  the  3rd  trimester  
Gonorrhea  in  Children  
•   Presents  within  3-­‐‑5  days  of  delivery  with  ophthalmia  neonatorum  and  sepsis.  Other,  less  
severe,  manifestations  include  rhinitis,  vaginitis,  urethritis,  and  infection  at  the  site  of  fetal  
monitoring  
Complications  
•   If  untreated  in  pregnancy,  risk  of  pre-­‐‑term  rupture  of  membranes,  preterm  labor,  
chorioamnionitis,  and  postpartum  endomyometritis  
•   PID  in  10-­‐‑40%  
•   Disseminated  gonococcal  infection  is  a  rare  complication  à  septic  emboli  à  polyarticular  
tenosynovitis  and  dermatitis  
 
Family  Medicine  Textbook  Notes     84  
 
  Definition  
  •   An  acute  infection  in  woman  of  any  or  all  of  her  upper  genital  tract  structures  (i.e.,  uterus,  
  fallopian  tubes,  and  ovaries)  
  •   Initiated  by  the  ascent  of  a  sexually  transmitted  agent  from  the  endocervix  to  the  upper  genital  
  structures  
  •   MCC:  N  gonorrhoeae  and  C  trachomatis  
  •   However,  up  to  70%  are  nongonococcal  and  non-­‐‑chlamydial  à  other  microorganisms  include  
  Mycoplasma  hominis,  Streptococcus  sp.,  Staphylococcus  sp.,  Haemophilus  sp.,  Escherichia  coli,  
Bacteroides  sp.,  Peptostreptococcus  sp.,  Peptococcus  sp.,  Clostridium  sp.,  and  Actinomyces  sp.  
 
Clinical  Evaluation  
Pelvic  Inflammatory  
•   No  set  of  signs  and  symptoms  is  pathognomonic  
Disease  
•   May  present  with  recent  onset  of  lower  abdominal  pain  that  worsens  during  coitus  or  with  
jarring  movement  
•   Other  symptoms  include  fever,  malaise,  vaginal  discharge,  irregular  bleeding,  nausea,  and  
vomiting  
•   Symptoms  usually  develop  during  menses  or  during  first  2  weeks  of  menstrual  cycle  
Diagnostic  Testing  
•   PID  is  a  clinical  diagnosis.  Most  lab  tests  are  nonspecific  
•   Begin  evaluation  for  women  with  suspected  PID  with  a  pregnancy  test  to  rule  out  ectopic  
pregnancy  or  intrauterine  pregnancy  complications.    
•   Other  recommended  tests  include:  
-­‐‑   Gonorrhea  and  chlamydia  
-­‐‑   Microscopic  exam  of  vaginal  discharge  
-­‐‑   Complete  blood  counts  (<  ½  will  have  leukocytosis)  
-­‐‑   Urinalysis  
-­‐‑   ESR  or  CRP  
Treatment  
•   If  patients  with  PID  meet  hospitalization  criteria  and  parenteral  antibiotic  therapy  is  indicated  
à  use  cefotetan  or  cefoxitin  plus  doxyclycline  
Family  Medicine  Textbook  Notes     85  
 
  Background  
  •   Most  common  viral  STI  in  the  United  States  
  •   Types  6  and  11  (low  risk)  are  associated  with  common  venereal  warts  and  recurrent  respiratory  
  papillomatosis  
  •   Types  16,  18,  31,  32,  and  35  (high  risk)  cause  low-­‐‑  and  high-­‐‑grade  cervical  dysplasias  and  
  anogenital  cancers  
  Clinical  Evaluation  
  •   Majority  are  asymptomatic  
•   Most  common  clinical  manifestation  is  genital  warts,  which  may  appear  as  single  or  multiple  
 
papules  on  the  vulva,  cervix,  vagina,  perineum,  penis,  scrotum,  or  perianal  region  
 
•   May  also  manifest  as  recurrent  respiratory  papillomatosis  and  cancers  
 
Diagnostic  Testing  
 
•   Definitive  diagnosis  is  based  on  detection  of  viral  nucleic  acid  (DNA  or  RNA)  or  capsid  protein  
Human  Papillomavirus  
•   Cervical  HPV  can  be  diagnosed  by  Pap  smear,  which  correlates  well  with  the  presence  of  HPV  
DNA  by  polymerase  chain  reaction  
•   DNA  probe  can  be  used  for  viral  typing  in  order  to  determine  if  it  is  a  high-­‐‑risk  or  low-­‐‑risk  
subtype  
Treatment  for  external  HPV  
•   Treatment  is  not  recommended  for  subclinical  genital  HPV  b/c  there  is  no  cure  
•   Patient  applied  treatment  
-­‐‑   Podofilox  0.5%  solution  or  gel  applied  twice  daily  for  3  days  followed  by  4  days  of  no  
therapy.  May  repeat  this  cycle  4  total  times  
-­‐‑   Imiquimod  5%  cream  applied  once  daily  at  bedtime,  3  times  per  week,  for  up  to  16  weeks  
•   Provider  administrated  treatment  
-­‐‑   Cryotherapy  with  liquid  nitrogen  or  cryotherapy  
-­‐‑   Podophyllin  resin  10%-­‐‑25%  in  a  compound  tincture  of  benzoin  
-­‐‑   Trichloracetic  acid  or  bichloracetic  acid  80-­‐‑90%  
-­‐‑   Surgical  removal  or  laser  surgery  
-­‐‑   Intralesional  interferon    
Family  Medicine  Textbook  Notes     86  
 
  Background  
  •   Caused  by  Herpes  Simplex  Virus  (HSV-­‐‑1  and  HSV-­‐‑2_  
  Clinical  Evaluation  
  •   First-­‐‑episode  primary  infection  (new  infection)  
  -­‐‑   Usually  causes  significant  symptoms  including  a  prodrome  of  fever,  malaise,  
  headache,  myalgia,  and  genital  paresthesias  before  the  breakout  of  cutaneous  lesions  
  -­‐‑   Multiple,  painful  vesicles  develop  1-­‐‑3  days  after  the  prodrome  which  later  ulcerate  
  -­‐‑   Often  there  is  painful  inguinal  lymphadenopathy  
  -­‐‑   Patients  are  most  infectious  during  this  early  phase  of  prodromal  symptoms  or  open  
  ulcers  
  •   First-­‐‑episode  non-­‐‑primary  infection  (unrecognized  HSV)  
Genital  Herpes   -­‐‑   Tends  to  be  less  severe  with  fewer  lesions,  faster  healing,  and  a  shorter  period  of  viral  
  shedding  
  •   Recurrent  Episodes  
  -­‐‑   Usually  a  prodrome  of  tingling,  pruritis,  or  dysethesias  before  the  outbreak  of  genital  
  lesions  
  -­‐‑   Lesions  normally  erupt  in  the  same  site  as  the  primary  episode  and  crust  over  in  4-­‐‑5  
  days  
  Diagnostic  Testing  
  •   Isolation  of  HSV  in  cell  culture  is  the  preferred  virologic  test  for  patients  who  seek  medical  
  treatment  for  genital  ulcers  or  other  mucocutaneous  lesions  à  culture  needs  to  be  
  performed  within  the  first  few  days  of  a  breakout  
  •   The  presence  of  serum  antibodies  to  HSV-­‐‑2  is  usually  indicative  of  genital  herpes,  whereas  
  HSV-­‐‑1  antibodies  do  not  differentiate  b/w  genital  and  oropharyngeal  infection  
   
   
   
   
Family  Medicine  Textbook  Notes     87  
 
  Treatment  
  •   First  episode  of  genital  herpes  
  -­‐‑   Acyclovir  400  mg  orally  3x/day  for  7-­‐‑10  days  OR  
  -­‐‑   Acyclovir  200  mg  orally  5x/day  for  7-­‐‑10  days  OR  
  -­‐‑   Famciclovir  250  mg  orally  3x/day  for  7-­‐‑10  days  OR  
  -­‐‑   Valacyclovir  1  g  orally  2x/day  for  7-­‐‑10  days  
Genital  Herpes,   •   Suppressive  therapy  for  recurrent  genital  herpes  
continued   -­‐‑   Acyclovir  400  mg  orally  2x/day  OR  
-­‐‑   Famciclovir  250  mg  orally  2x/day  OR  
-­‐‑   Valacyclovir  500  mg  orally  1x/day  OR  
-­‐‑   Valacyclovir  1  g  orally  1x/day  
•   Episodic  therapy  for  recurrent  genital  herpes  
-­‐‑   Acyclovir  400  mg  orally  3x/day  for  5  days  OR  
-­‐‑   Acyclovir  800  mg  orally  2x/day  for  5  days  OR  
-­‐‑   Acyclovir  800  mg  orally  3x/day  for  2  days  OR  
-­‐‑   Famciclovir  125  mg  orally  2x/day  for  5  days  OR  
-­‐‑   Famciclovir  1000  mg  orally  2x/day  for  1  day  OR  
-­‐‑   Valacyclovir  500  mg  orally  2x/day  for  3  days  OR  
-­‐‑   Valacyclovir  1  g  orally  1x/day  for  5  days  
Family  Medicine  Textbook  Notes     88  
 
 
 
 
 
 
 
Stages  and  Treatment  
Regimens  for  Syphilis  

 
Family  Medicine  Textbook  Notes     89  
 

 
  •   T  pallidum  cannot  be  detected  by  culture  
  •   Serological  tests  are  non-­‐‑treponemal  (Venereal  Disease  Research  Laboratory  Test  (VDRL)  
Diagnostic  Testing  for   and  Rapid  Plasma  Reagin  (RPR)  tests)  or  treponemal  (fluorescent  treponemal  antibody  
Syphilis   absorb  (FTA-­‐‑ABS))  
•   A  spinal  tap  for  CSF  analysis  to  rule  out  neurosyphilis  is  recommended  for  all  children  with  
syphilis,  patients  with  treatment  failure,  patients  with  nervous  system  or  eye  involvement,  
those  with  evidence  of  tertiary  syphilis,  and  those  infected  with  both  HIV  and  late  latent  
syphilis/syphilis  of  unknown  duration  
Family  Medicine  Textbook  Notes     90  
 
  Background  
  •   HIV  is  a  retrovirus  that  produces  a  broad  spectrum  of  disease  from  asymptomatic  to  
  acquired  immunodeficiency  syndrome  (AIDS)  
  •   After  infection,  the  virus  enters,  replicates,  and  then  destroys  the  CD4  Helper  T  Cells  
  •   The  transition  from  initial  HIV  infection  to  AIDS  takes  a  median  of  10  years  
  •   Transmission  can  be  sexual,  vertical,  through  the  use  of  shared/contaminated  needles,  
HIV   through  exposure  to  contaminated  blood,  and  via  breast  milk  
Clinical  Evaluation  
•   Most  individuals  with  HIV  infection  are  asymptomatic  
•   A  portion  of  newly  infected  individuals  will  develop  an  acute  syndrome  with  fever,  
pharyngitis,  weight  loss,  adenopathy,  and  nausea/vomiting  
Diagnostic  Testing  
•   It  takes  3  weeks  to  6  months  for  a  patient  to  develop  detectable  HIV  antibodies  
•   Initial  testing  uses  an  enzyme  immunoassay  for  antibodies  to  HIV  
•   If  the  initial  test  is  positive,  confirmatory  testing  using  the  Western  blot  assay  is  performed  
•   Rapid  testing  using  a  drop  of  blood  or  a  buccal  swab  is  also  available    
•   Initial  lab  testing  for  patients  with  newly  diagnosed  HIV  include  CD4  count,  HIV  RNA  
viral  load,  CBC,  liver  and  renal  functions,  and  urinalysis    
Family  Medicine  Textbook  Notes     91  
 
 
 
 
 
 
 
 
Cancers,  Opportunistic  
Infections,  and  
Syndromes  Associated  
with  HIV  

 
Family  Medicine  Textbook  Notes     92  
 
  •   The  ankle  joint  is  a  hinge  joint,  which  
  can  also  invert  and  evert  in  response  to  
  walking  on  irregular  surfaces  
  •   It  includes  the  distal  fibula,  talus,  and  
  distal  tibia  
  •   The  ankle  mortise  (combined  joint  
  structure)  is  stabilized  on  the  lateral  
  side  by  –  from  anterior  to  posterior  –  
  the  anterior  talofibular  ligament,  the  
Functional  Anatomy  of   calcaneal  fibular  ligament,  the  posterior  
the  Ankle   talofibular  ligament,  and  the  peroneus    
longus  and  brevis  tendons   •   The  Achilles  tendon  attaches  to  the  
•   In  the  frontal  plane,  the  anterior   posterior  portion  of  the  calcaneus  à  
tibiofibular  ligament,  posterior   plantar  flexion  
tibiofibular  ligament  and  syndesmosis,   •   Peroneus  brevis  and  longus  tendons  à  
a  thickened  sheet  of  interosseous   eversion  
membrane,  stabilize  the  mortise  and   •   Posterior  tibialis  tendon  runs  posterior  and  
allow  minimal  motion  b/w  the  distal   inferior  to  the  medial  malleolus  and  
tibia  and  fibular  à  more  importantly,   attaches  to  the  navicular  bone  à  inversion  
they  prevent  separation  of  the  tibia  and   +  arch  support  
fibula  as  forces  from  the  calcaneous  are  
transferred  up  the  leg  
•   On  the  medial  side,  the  deltoid  
ligament  has  a  superficial  and  deep  
layer  à  provides  resistance  to  eversion  
stress  
Family  Medicine  Textbook  Notes     93  
 
 
 
 
 
 
 
 
What  are  red  flags  in  
patients  with  ankle  or  
knee  pain?  

 
Family  Medicine  Textbook  Notes     94  
 
  •   The  physical  exam  should  include  evaluation  of  gait,  range  of  motion,  strength  testing,  
  palpation,  and  finally,  ought  to  include  an  assessment  of  stability  
Physical  exam  for  a   •   To  help  distinguish  an  ankle  sprain  from  a  fracture,  you  should  palpate  the  posterior  edge  
patient  with  ankle  pain   and  the  tip  of  each  malleolus  and  the  base  of  the  5th  metatarsal  
•   You  should  also  have  the  patient  take  4  steps  

  Anterior  Drawer  Test  


  •   Small  arrow  =  positive  test  
  •   Used  to  evaluate  the  intactness  of  the  
  anterior  talofibular  ligament  
  •   A  3-­‐‑mm  difference  between  the  ankles  
  suggests  disruption  of  the  anterior  
  Talofibular  ligament  
 
 
What  are  the  physical                            
Talar  Tilt  Test  
exam  maneuvers  that  can  
•   Used  to  evaluate  the  stability  of  the  
be  used  to  evaluate  ankle  
anterior  talofibular  and  calcaneofibular  
pain?  
ligaments  
•   The  ankle  is  unstable  if  the  anterior  
talofibular  and  calcaneofibular  
ligaments  are  torn  

 
Family  Medicine  Textbook  Notes     95  
 
Squeeze  Test  
•   Used  for  anterior/lateral  ankle  pain  
•   Compress  the  tibia  and  fibula  together  
above  the  midpoint  of  the  calf  
•   Pain  indicates  a  syndesmosis  sprain  à  
“high-­‐‑ankle  sprain”  

 
Cotton  Test  
•   “Rocker  Test”  
•   Used  for  syndesmosis  sprains  
•   Performed  like  the  talar  tilt  test,  except  
mediolateral  force  is  applied  

 
Thompson  Test  
•   Used  to  assess  the  Achilles  Tendon  
•   Midcalf  compression  test  
•   With  the  patient  lying  prone  on  the  
exam  table  and  the  feet  extended  over  
the  edge  of  the  table,  the  gastrocnemius  
and  soleus  are  compressed  by  squeezing    
the  calf  
•   If  the  foot  plantar  flexes,  the  test  is  
negative  or  normal  
•   If  the  foot  does  not  move  à  indicates  
complete  or  near  complete  rupture  of  
the  tendon  
Family  Medicine  Textbook  Notes     96  
 
 
 
 
 
 
 
 
 
 
 
 
Ottawa  Ankle  Rules  

 
Family  Medicine  Textbook  Notes     97  
 
 
 
 
 
 
Management  of  Ankle  
Sprains  

 
  Grade  I  
  •   Partial  rupture  of  the  anterior  talofibular  ligament  (ATFL)  
How  do  you  grade  ankle   Grade  II  
sprains?   •   Complete  rupture  of  the  ATFL  and  partial  rupture  of  the  calcaneofibular  ligament  (CFL)  
Grade  III  
•   Complete  rupture  of  both  ATFL  and  CFL  

  Grade  I  Sprain  
  •   Rest,  Ice,  Compression,  Elevation  (RICE)  
  •   NSAIDs  
Describe  the   •   Do  NOT  require  immobilization.  Early  mobilization  improves  function,  reduces  pain  and  
management  of  different   swelling,  and  speeds  return  to  work  and  sports  
types/grades  of  ankle   Grade  II  Sprain  
injuries   •   RICE  
  •   Immobilization  in  a  lace-­‐‑up  splint  or  an  air-­‐‑stirrup  splint  for  2-­‐‑7  days  
  •   Crutches  are  helpful  
Family  Medicine  Textbook  Notes     98  
 
  Grade  III  Sprain  
  •   RICE  
Describe  the   •   Air-­‐‑stirrup  splint  or  a  below-­‐‑knee  cast  for  up  to  3  months  followed  by  formal  physical  
management  of  different   therapy  
types/grades  of  ankle   Syndesmosis  Sprains  
injuries,  continued   •   RICE  
•   NSAIDs  
•   Removable  splints  (i.e.,  posterior  splint,  pneumatic  plent,  or  a  Bledsoe  brace)  or  casting  
facilitates  progressive  weight  bearing  
•   Passive  range-­‐‑of-­‐‑motion  exercises  (e.g.,  tracing  the  alphabet,  drawing  circles,  etc  with  the  
foot),  especially  dorsiflexion,  should  begin  within  a  week  of  injury  
  •   Many  fractures  can  be  managed  in  the  office  without  referral  
  •   Avulsion  or  chip  fractures  are  the  most  common  and  can  occur  at  the  distal  fibula  below  the  
  level  of  the  mortise,  the  distal  portion  of  the  tibial  plafond,  and  the  anterior  surface  of  the  
  talus  
Management  of  Ankle   •   Regardless  of  the  avulsion  fracture  size,  these  usually  can  be  treated  based  upon  the  
Fractures   severity  of  the  associated  ankle  sprain  
•   If  the  fragments  are  displaced  more  than  2  mm,  immobilize  the  patient  in  a  cast  or  posterior  
splint  and  refer  to  an  orthopedic  surgeon  
•   Avulsion  fractures  of  the  peroneus  brevis  insertion  from  the  5th  metatarsal  head  will  heal  
without  treatment  and  should  be  immobilized  
•   You  should  refer  patients  with  fractures  of  the  base  of  the  5th  metatarsal  (Jones’  Fracture),  
the  proximal  second,  third  or  fourth  metatarsals  (known  as  Lis  Franc  Fractures)  and  of  the  
growth  plate  (Salter-­‐‑Harris  Fractures)  to  an  orthopedic  surgeon  
Family  Medicine  Textbook  Notes     99  
 
  •   Relative  rest,  rehabilitation  of  the  gastroc  and  soleus  muscles,  ice,  heel  lifts,  and  analgesics  
  •   Rehabilitation  of  the  calf  muscles  calf  muscles  begins  with  progressive  stretching  and  ROM  
Management  of  Achilles   exercises  followed  by  strength  training  
Tendinosis   •   NSAIDs  for  pain  and  inflammation  
•   Surgery  is  recommended  only  for  patients  who  fail  conservative  therapy  and  desire  to  
continue  activities  that  exacerbate  tendinosis  ,  and  in  those  patients  with  Achilles  tendon  
ruptures  
  •   The  knee  joint  consists  of  the  patella,  
  tibia,  and  femur  
  •   The  primary  stabilizers  of  the  knee  are  
  the  ACL  and  the  PCL,  the  MCL  and  
  LCL,  the  menisci,  and  joint  capsule,  and  
  the  medial  and  lateral  retinacula  that  
  attach  to  the  patella  
Functional  Anatomy  of   •   Secondary  stabilizers  of  the  knee  include  
the  Knee   the  iliotibial  band  (AKA  the  tensor  fascia  
lata)  and  the  quadriceps,  hamstrings,  
and  popliteus  mm  
•   The  ACL  prevents  anterior  movement  of  
the  tibia  on  the  femur  
•   The  PCL  resists  posterior  movement  of  
the  tibia  on  the  femur  
•   The  medial  and  lateral  menisci  primarily    
act  as  shock  absorbers  but  also  stabilize  
the  knee  during  movements  such  as  
pivoting  
Family  Medicine  Textbook  Notes     100  
 
  •   Hearing  or  feeling  a  “pop”  suggests  an  ACL  
  •   Locking  of  the  knee  (where  the  knee  gets  “stuck”  at  some  point  in  its  ROM)  is  typically  
Which  findings  are   associated  with  meniscus  injuries  or  a  loos  joint  body  (usually  cartilage)  
associated  with  which   •   Inability  to  bear  weight  and  persistent  “giving  way”  of  the  knee  indicate  internal  
knee  injuries?   derangement  but  are  NOT  specific  for  any  one  injury  
•   Patients  with  degenerative  joint  disease  typically  complain  of  stiffness  with  inactivity  and  
pain  with  weight  bearing  activity  
  Lachman  Test  
  •   Very  good  at  ruling  in  or  out  ACL  tears  
  •   Performed  with  the  knee  flexed  to  20-­‐‑30  
  degrees  and  with  the  top  hand  
  stabilizing  the  femur  while  the  lower  
  hand,  with  the  thumb  on  top,  wraps  
  around  the  inside  of  the  proximal  tibia  
What  are  the  physical   •   Using  the  lower  hand,  smoothly  and  
exam  maneuvers  that  can   gently  try  to  slide  the  tibia  forward    
be  used  to  evaluate  knee   •   3-­‐‑mm  side-­‐‑to-­‐‑side  difference  or  the  
pain?   absence  of  a  distinct  stop  indicates  an  
  ACL  tear  
  Posterior  Drawer  Test  
  •   Do  this  test  before  the  Lachman  Test  
  •   Flex  the  knee  to  90  degrees  and  push  the  
  tibia  in  a  posterior  direction  
  •   Used  to  ensure  PCL  integrity  
  •   If  the  PCL  is  torn,  the  Lachman  test  is  
  still  accurate  but  must  be  done  more  
  carefully    
Family  Medicine  Textbook  Notes     101  
 
  McMurray  Test  
  •   Used  to  detect  meniscal  damage  
  •   Place  the  knee  in  full  flexion  and  place  
  your  fingers  along  the  lateral  joint  line.  
What  are  the  physical   Use  your  other  hand  to  cup  the  heel  
exam  maneuvers  that  can   •   While  applying  a  combination  of  
be  used  to  evaluate  knee   external  rotation  and  medially  directed  
pain,  continued   stress,  bring  the  knee  slowly  into  
  extension  
  •   A  palpable  click  suggests  a  lateral  
  meniscus  tear  
  •   It  is  very  likely  that,  in  the  presence  of  a  
  tear,  this  will  also  cause  an  increase  in  
  pain  
  •   Repeat  the  maneuvers  with  your  fingers  
  along  the  medial  joint  line  and  apply    
  internal  rotation  and  laterally-­‐‑directed  
  stress  to  the  knee  to  detect  medial  
  meniscus  tears  
Family  Medicine  Textbook  Notes     102  
 
  Patellar  Apprehension  Test  
  •   The  examiner  applies  inferiorly  directed  
  pressure  at  the  superior  portion  of  the  
  patella  
  •   Test  is  positive  if  the  patient  winces,  
What  are  the  physical   grabs  the  knee,  or  otherwise  voices  
exam  maneuvers  that  can   displeasure  
be  used  to  evaluate  knee  
pain,  continued  

 
Patellar  Compression  Test  
•   The  examiner  firmly  holds  the  patella  in  
place  while  the  patient  contracts  the  
quadriceps  muscle  
•   Test  is  positive  if  the  patient  winces,  
grabs  the  knee,  or  otherwise  voices  
displeasure    
Family  Medicine  Textbook  Notes     103  
 
 
 
 
 
Ottawa  Knee  Rules  

 
What  are  the  major   •   Hemarthrosis  or  rapid  fluid  accumulation  
reasons  to  refer  a  patient   •   ACL  or  meniscus  tear  
with  knee  pain  for  an   •   Third-­‐‑degree  collateral  ligament  injuries  
additional  evaluation  or   •   Severe  functional  impairment  
for  surgery?   •   Uncertainty  about  the  diagnosis  (referral  for  second  opinion)  
•   Poor  response  to  conservative  treatment  (referral  for  second  opinion)  
Family  Medicine  Textbook  Notes     104  
 
 
 
 
 
Management  of  the  most  
common  causes  of  knee  
pain  

 
Family  Medicine  Textbook  Notes     105  
 
 
 
 
 
 
 
 
Anatomy  of  the  Spine  

 
Family  Medicine  Textbook  Notes     106  
 
  •   Reflects  soft  tissue  inflammation  
Lumbar  Sprain/Strain   •   Most  common  cause  of  back  pain  
•   Presents  with  pain  in  the  lower  back  that  can  radiate  into  the  buttocks  or  proximal  lower  
extremities  
   
  •   Syndrome  of  herniated  disk  or  herniated  
  nucleus  pulposus  with  impingement  of  
  the  nerve  root  (radiculopathy)  
Sciatica   •   The  radicular  pain  typically  radiates  in  a  
dermatomal  pattern  down  the  leg  and  
below  the  knee  
•   Whether  or  not  the  symptoms  radiate  
past  the  knee  is  a  key  distinguishing  
feature  of  sciatica  since  non-­‐‑radicular  
causes  of  lower  back  pain  
   
   
  •   Diagnosed  when  acute  neurological  impairment  occurs  in  those  structures  supplied  by  the  
Cauda  Equina  Syndrome   sacral  nerve  roots,  notably  causing  bowel  or  bladder  dysfunction  or  perineal  (“saddle”)  
anesthesia  
•   This  is  a  surgical  emergency  that  requires  urgent  diagnosis  and  treatment  
•   MCC:  large  paracentral  disk  herniations  and  tumors  

  •   Occurs  primarily  in  older  individuals  and  is  related  to  degenerative  changes  in  the  spine  
Spinal  Stenosis   with  resulting  hypertrophy  of  the  facet  joints  and  ligamentum  flavum  
•   Diameter  of  spinal  canal  and  neural  outlets  is  compromised,  often  at  multiple  levels  
Family  Medicine  Textbook  Notes     107  
 
  •   Typically  occur  in  older  individuals  with  osteoporosis  
Vertebral  Compression   •   Risk  factors:  female  sex,  early  menopause,  Northern  European  or  Asian  ethnicity,  cigarette  
Fractures   smoking,  sedentary  lifestyle,  and  chronic  steroid  use  
 
 
 
 
 
 
 
Red  flags  for  patients  
with  low  back  pain  

 
Family  Medicine  Textbook  Notes     108  
 
  •   The  SLR  is  performed  with  the  patient  supine  to  see  if  radicular  pain  occurs  when  the  leg  is  
Straight  Leg  Raising  Test   elevated  between  30  and  60  degrees  
(SLR)   •   The  SLR  test  is  fairly  sensitive  for  a  herniated  disk,  but  NOT  specific  
•   It  is  considered  positive  when  the  SLR  causes  or  increases  lower  back  pain  that  radiates  
down  the  leg  
•   The  crossed  SLR  test  (pain  radiating  down  the  leg  on  lifting  the  opposite  leg)  is  more  
specific  for  a  herniated  disk,  but  less  sensitive  
 
 
 
 
 
 
 
Algorithm  for  
management  of  low  back  
pain  

 
Family  Medicine  Textbook  Notes     109  
 
 
 
Findings  for  disk  
herniation  at  different  
levels  
 

  Plain  Film  
  •   Use  for  cancer  
  -­‐‑   Sn  0.6,  Sp  0.95-­‐‑0.995  
  -­‐‑   Costs  ~  <  $150  
Imaging  used  for   CT  
different  causes  of  low   •   Use  for  herniated  nucleus  pulposus  
back  pain   -­‐‑   Sn  0.62-­‐‑0.9,  Sp  0.7-­‐‑0.87  
-­‐‑   Costs  ~  $400-­‐‑1,000  
•   Use  for  Spinal  Stenosis  
-­‐‑   Sn  0.9,  Sp  0.8-­‐‑0.96  
MRI  
•   Use  for  cancer  
-­‐‑   Sn  0.83-­‐‑0.93,  Sp  0.90-­‐‑0.97  
-­‐‑   Costs  ~  $750-­‐‑1,500  
•   Use  for  Infection  
-­‐‑   Sn  0.96,  Sp  0.92  
•   Use  for  herniated  nucleus  pulposus  
-­‐‑   Sn  0.6-­‐‑1.0,  Sp  0.43-­‐‑0.97  
Family  Medicine  Textbook  Notes     110  
 
  •   Unfortunately,  the  multifactorial  nature  of  back  pain  makes  any  single  preventative  
  strategy  unlikely  to  account  for  much  benefit  
  •   The  most  consistently  identified  risk  factor  for  LBP  is  a  history  of  back  pain  
  •   Other  risk  factors  include  physical  determinants  such  as  heavy  lifting,  frequent  bending,  
Prevention  of  lower  back   twisting  and  lifting,  repetitive  work  with  exposure  to  vibration,  and  psychosocial  issues  
pain   such  as  depressed  mood,  psychosocial  distress,  poor  coping  strategies,  fear  avoidance,  
somatization,  workers  compensations  claims  and  litigation.  
•   The  most  effective  prevention  strategy  seems  to  by  physical  exercise  
•   Education  that  assists  in  coping  with  back  pain  and  encourages  activity  has  a  small  benefit  
in  preventing  chronic  or  recurrent  back  pain  
•   There  is  strong  evidence  that  back  belts  and  lumbar  supports  are  NOT  effective  in  
preventing  back  pain  in  workers  and  should  not  be  recommended  
 
 
 
 
 
 
Pain  receptor  sites  of  the  
lower  cervical  spine  

 
Family  Medicine  Textbook  Notes     111  
 
  Grade  I  
  •   No  signs  or  symptoms  of  significant  structural  abnormalities;  minimal  interference  with  
  normal  activities;  responds  to  minimal  interventions;  requires  no  diagnostic  evaluation  
Grading  for  neck  pain   Grade  II  
•   No  signs  or  symptoms  of  significant  structural  problems;  significant  interference  with  daily  
activities;  requires  urgent  attention  and  treatment  to  minimize  prolonged  disability  
Grade  III  
•   No  signs  or  symptoms  of  significant  structural  problems;  neurologic  signs  present;  may  
require  diagnostic  evaluation  and  more  aggressive  interventions  
Grade  IV  
•   Signs  of  structural  problems;  requires  immediate  diagnostic  and  treatment  interventions  
  •   Refers  to  degenerative  change  in  the  intervertebral  unit  of  the  cervical  spine  
Cervical  Spondylosis   •   This  term  is  used  synonymously  with  degenerative  disc  disease  or  degenerative  
spondylosis  
•   Degenerative  changes  include  disk  space  narrowing,  osteophytes  in  the  disc  margins,  and  
arthritic  changes  in  the  facet  joints  
Cervical  Spondylotic   •   A  condition  in  which  arthritic  changes,  primarily  the  development  of  osteophytes  and  
Myelopathy   thickening  of  the  ligamentum  flavum,  cause  direct  compression  of  the  spinal  cord,  resulting  
in  myelopathy  
•   Symptoms  develop  when  the  cord  has  been  impinged  by  >  30%  
•   Motion  can  aggravate  spinal  cord  damage  by  stretching  the  cord  over  protruding  
osteophytes  in  flexion  or  by  pressure  from  a  thickened  bulging  ligamentum  flavum  in  
extension  
Family  Medicine  Textbook  Notes     112  
 
   
   
   
  •   Typically  defined  as  an  acute  injury  to  the  
  cervical  spine  resulting  from  sudden  
  acceleration  (hyperextension)  and  
  subsequent  deceleration  (hyperflexion)  of  
Acceleration/Deceleration   the  head,  often  resulting  from  a  rear  
Injury  (Whiplash-­‐‑ impact  motor  vehicle  accident    
Associated  Disorder)   •   Injuries  can  occur  to  the  paracervical  
muscles,  anterior  longitudinal  ligament,  
nerve  roots,  disk  annulus,  and,  most  
importantly,  the  facet  joints  can  be  forced  
to  impact  posteriorly  on  each  other  
•   The  symptoms  often  do  not  occur  until  12-­‐‑
24  hours  after  injury,  which  may  be  
because  it  takes  time  for  edema  to  develop  
after  microscopic  hemorrhage  and  injury  
 
 
Family  Medicine  Textbook  Notes     113  
 
  Spurling  Test/Maneuver  
  •   Performed  by  having  the  patient  side  bend  
  (bring  ear  close  to  shoulder)  to  the  side  of  
  the  radicular  pain  and  extend  his  or  her  
  head  
  •   Positive  test:  pressure  exerted  downward  
  by  the  examiner  on  the  patient’s  head  will  
  create  or  intensify  radicular  symptoms  
 
  •   Suggests  nerve  impingement  
  Axial  Manual  Traction  Test  
Clinical  tests  that  can  be   •   Examiner  pulls  up  on  the  head  to  
used  to  aid  the  diagnosis   momentarily  theoretically  decrease  the  
of  cervical  disk   pressure  on  the  cervical  root  
herniation   •   Relief  of  pain  is  considered  a  positive  
result  for  cervical  root  compression  

 
Arm  Abduction  Test  
•   Performed  by  elevating  (full  abduction  of  
the  shoulder)  the  affected  arm  over  the  
head  of  the  seated  patient  
•   Theoretically  decreases  the  traction  on  the  
cervical  root  
•   Relief  of  pain  is  considered  a  positive  
result  for  cervical  root  compression  

 
Family  Medicine  Textbook  Notes     114  
 
 
 
 
 
 
 
Canadian  C-­‐‑Spine  Rule  
 
 
Note:  a  “dangerous  
mechanism”  is  considered  
to  be  a  fall  from  an  
elevation  of  ≥  3  feet  or  5  
stairs;  an  axial  load  to  the  
head  (e.g.  diving);  a  motor  
vehicle  collision  at  high  
speed  (>  100  km/hr)  or  
with  rollover  or  ejection;  a  
collision  involving  a  
motorized  recreational  
vehicle  or  a  bicycle  
 
collision  
Family  Medicine  Textbook  Notes     115  
 
 
 
 
 
 
 
Red  flags  for  patients  
with  neck  pain  

 
Family  Medicine  Textbook  Notes     116  
 
  Spinal  Level:  C4/C5  
  Motor  Weakness:  deltoid,  shoulder,  biceps  
  Sensory  Loss:  lateral  upper  arm  
C5  Nerve  Root  Injury   Paresthesia:  None  in  digits  
Referred  Pain:  Shoulder  and  upper  lateral  arm  
Reflex  Loss:  Biceps  
Subjective  Pain:  Shoulder  (but  relatively  pain  free)  
  Spinal  Level:  C5/C6  
  Motor  Weakness:  Biceps,  brachioradialis  
  Sensory  Loss:  thumb  and  forearm  proximal  to  thumb  
C6  Nerve  Root  Injury   Paresthesia:  Thumb  
Referred  Pain:  Radial  aspect  of  forearm  
Reflex  Loss:  Brachioradialis  and  biceps  
Subjective  Pain:  deltoid,  rhomboid  muscle  areas  
  Spinal  Level:  C6/C7  
  Motor  Weakness:  Triceps  
  Sensory  Loss:  Middle  ring  finger  
C7  Nerve  Root  Injury   Paresthesia:  Middle  Finger  
Referred  Pain:  Dorsal  aspect  of  forearm  
Reflex  Loss:  Triceps  
Subjective  Pain:  Dorsolateral  upper  arm,  superomedial  angle  of  scapula  
  Spinal  Level:  C7/T1  
  Motor  Weakness:  Finger  intrinsic  
  Sensory  Loss:  Inner  forearm,  little  finger  
C8  Nerve  Root  Injury   Paresthesia:  Ring  and  little  finger  
Referred  Pain:  Ulnar  aspect  of  forearm  and  little  finger  
Reflex  Loss:  triceps  or  none  
Subjective  Pain:  Scapula,  ulnar  side  of  upper  arm  
Family  Medicine  Textbook  Notes     117  
 
 
 
 
 
 
 
Therapeutic  options  for  
management  of  neck  pain  

 
  •   A  “stinger”  or  “burner”  (transient  neurapraxia)  is  a  form  of  injury  unique  to  collision  sports  
  •   The  “stinger”  is  where  the  athlete  complains  of  burning  or  tingling  in  an  upper  extremity  
“Stingers”  in  contact   after  a  collision  à  distributed  in  the  upper  arm  or  in  the  entire  arm;  peri-­‐‑scapular  pain  also  
sports   possible  
•   Nerve  roots  most  common  affected  are  C5-­‐‑C6  
•   Injury  is  likely  due  to  a  traction  injury  to  the  brachial  plexus  or  compression  injury  to  the  
dorsal  nerve  root  ganglion  as  it  exits  the  neural  foramen  à  symptoms  last  minutes  to  hours  
•   Can  return  to  play  when  they  have  full,  pain  free  ROM  and  symmetric  strength  with  
uninjured  side  
Family  Medicine  Textbook  Notes     118  
 
 
 
 
 
 
 
 
 
Shoulder  Anatomy  

 
•   The  bony  anatomy  of  the  shoulder  is  composed  of:  the  humerus,  the  clavicle  and  the  
scapula,  which  is  further  divided  into  the  acromion,  the  coronoid,  and  the  glenoid.  
•   Shoulder  joint  complex  has  4  different  articulations:  Sternoclavicular,  Acromioclavicular,  
Glenohumeral,  and  Scapulothoracic  joints  
•   The  soft-­‐‑tissue  support  can  be  divided  into  static  and  dynamic  stabilizers  
-­‐‑   Static  stabilizers:  bony  support,  joint  capsule,  glenohumeral  ligaments,  and  glenoid  
labrum  
-­‐‑   Dynamic  Stabilizers:  rotator  cuff  muscles,  long  head  of  biceps  tendon  à  function  is  to  
maintain  the  humeral  head  centered  within  the  glenoid  of  the  scapula  during  
movement  
Family  Medicine  Textbook  Notes     119  
 
  •   The  majority  of  OA  in  the  shoulder  occurs  at  the  AC  Joint,  particularly  with  repetitive  
  overhead  activities  or  a  history  of  heavy  weight  lifting  
Shoulder  osteoarthritis   •   OA  of  the  glenohumeral  joint  is  much  more  rare  and  generally  occurs  with  a  history  of  
distant  trauma/dislocation  or  a  history  of  auto-­‐‑immune  arthritis  involving  the  shoulder  
joint  
•   Both  conditions  involve  the  degeneration  of  articular  cartilage  at  the  joint  with  associated  
synovitis,  effusion,  and  osteophyte  formation  
 
 
 
 
 
 
 
Key  elements  in  the  
history  for  common  
shoulder  disorders  

 
Family  Medicine  Textbook  Notes     120  
 
 
 
 
 
 
 
 
 
 
Differential  Diagnosis  of  
the  patient  with  shoulder  
pain  

 
Family  Medicine  Textbook  Notes     121  
 
 
 
 
 
 
Red  Flags  for  patients  
with  shoulder  pain  
indicating  more  serious  
disease  

 
  Abduction  
  •   Tests  the  supraspinatus  
Muscle  Strength  Testing   Internal  Rotation  
for  Shoulders   •   Tests  the  subscapularis  
External  Rotation  
•   Tests  the  infraspinatus  and  teres  minor  
Family  Medicine  Textbook  Notes     122  
 
  Hawkins’  Impingement  Test  
  •   Used  to  diagnose  impingement  of  the  
  rotator  cuff  muscles  
  •   A  positive  result  is  when  pain  results  
  reflecting  rotator  cuff  injury  
  •   The  test  is  performed  by  the  examiner  
  passively  forward  flexing  the  patient’s  arm  
  to  90  degrees  followed  by  passively  
Physical  Exam   internally  rotating  the  arm  to  its  end  point  
Maneuvers  for  the   (approximately  90  degrees)  
Shoulder  
 
 
 
 
 
 
 
 
 
 
 
 
 
Family  Medicine  Textbook  Notes     123  
 
  Empty  Can  Test  
  •   Isolates  the  supraspinatus  more  effectively  
  than  abduction  
  •   A  painful  result  is  often  associated  with  
  rotator  cuff  disease  
  •   Performed  by  having  the  patient  abduct  
  their  arm  to  90  degrees,  followed  by  
  forward  flexing  the  arm  30  degrees  to  lie  in  
  the  plane  of  the  scapula.  The  patient  is  
  then  instructed  to  “empty  the  imaginary  
  can.”  The  examiner  then  places  a  
  downward  force  at  the  wrist,  which  the  
  patient  should  resist  
Physical  Exam   •   Pain  and/or  weakness  may  signify  a    
Maneuvers  for  the   rotator  cuff  injury,  particularly  the  
Shoulder,  continued   supraspinatus  
  Subscapularis  Lift  Off  Test  
  •   Designed  to  best  isolate  the  subscapularis  
  •   Performed  by  having  the  patient  place  the  
  dorsum  of  their  hand  against  their  low  
  back  while  sitting  on  the  exam  table.  The  
  patient  then  manually  lifts  the  hand  away  
  (mainly  by  forcible  extension  of  the  
  shoulder)  from  the  low  back  while  the  
  examiner  resists    
  •   Pain  and/or  weakness  suggests  injury  to  
  subscapularis  
Family  Medicine  Textbook  Notes     124  
 
  Drop  Arm  Test  
  •   Performed  by  passively  abducting  the  
  patient’s  arm  to  160  degrees  and  then  
  asking  the  patient  to  slowly  lower  the  arm  
  to  their  side  
  •   A  positive  test  is  when  the  arm  cannot  be  
  lowered  in  a  controlled  fashion  and  
  “drops”  to  their  side  
  •   When  positive,  it  is  highly  specific  for  a  
  large  or  massive  rotator  cuff  tear,  often  
  necessitating  a  surgical  consultation  
Physical  Exam  
Maneuvers  for  the    
Shoulder,  continued  
 
 
 
 
 
 
 
 
 
 
 
 
Family  Medicine  Textbook  Notes     125  
 
  Cross-­‐‑Body  Adduction  Test  
  •   Stresses  the  AC  joint  à  assists  in  the  
  diagnosis  of  AC  joint  osteoarthritis  
  •   Performed  by  passively  bringing  the  
  patient’s  arm  across  the  body  towards  the  
  contralateral  shoulder  
Physical  Exam   •   Considered  positive  if  the  patient  has  pain  
Maneuvers  for  the   localized  to  the  AC  joint  
Shoulder,  continued  
 
 
   
Apprehension  Test  
 
•   Used  to  diagnose  chronic  anterior  
 
shoulder  instability  
 
•   Performed  by  having  the  patient  lie  supine  
 
and  in  a  relaxed  state.  The  patient’s  arm  is  
 
brought  into  90  degrees  of  abduction  with  
 
the  elbow  flexed  to  90  degrees.  The  
 
examiner  then  applied  gentle  external  
 
rotation  with  the  arm  still  abducted  at  90  
 
degrees.    
 
•   If  the  patient  becomes  apprehensive,  this  is  
   
a  positive  sign  of  anterior  instability  
Family  Medicine  Textbook  Notes     126  
 
  Sulcus  Sign  
Physical  Exam   •   Diagnostic  of  multidirectional  instability  
Maneuvers  for  the   •   Performed  with  the  patient’s  arm  dangling  
Shoulder,  continued   at  their  side.  The  examiner  then  exerts  an  
inferiorly  directed  force  on  the  arm  by  
pulling  down  on  the  patient’s  wrist  
•   The  test  is  positive  if  a  sulcus  or  
indentation  is  created  between  the  
acromion  and  proximal  humeral  head    
  •   The  majority  of  shoulder  disorders  can  be  diagnosed  with  history,  physical  exam,  and  plain  
  radiographs  
Diagnostic  Imaging  for   •   Generally,  the  role  of  MRI  is  when  a  patient  has  failed  6-­‐‑8  weeks  of  conservative  
Shoulder  Injuries   management  and  the  diagnosis  remains  uncertain  
•   Plain  radiographs  can  assess  whether  there  is  a  hook  or  spur  of  the  acromion,  significant  
AC  joint  osteoarthritis,  as  well  as  a  potential  accessory  os  acromiale  that  might  predispose  
patients  to  rotator  cuff  injury  and  tendonitis  
•   Diagnostic  Accuracy  of  MRI  
-­‐‑   Any  rotator  cuff  tear:  Sn  0.83  Sp  0.86  
-­‐‑   Partial-­‐‑thickness  rotator  cuff  tear:  Sn  0.44  Sp  0.90  
-­‐‑   Full-­‐‑thickness  rotator  cuff  tear:  Sn  0.89  Sp  0.93  
•   Diagnostic  Accuracy  of  Ultrasound  
-­‐‑   Any  rotator  cuff  tear:  Sn  0.80  Sp  0.85  
-­‐‑   Partial-­‐‑thickness  rotator  cuff  tear:  Sn  0.67  Sp  0.94  
-­‐‑   Full-­‐‑thickness  rotator  cuff  tear:  Sn  0.97  Sp  0.96  
Family  Medicine  Textbook  Notes     127  
 
 
 
 
 
 
 
 
 
 
Algorithm  for  patient  
presenting  with  acute  
shoulder  pain  

 
Family  Medicine  Textbook  Notes     128  
 
 
 
 
 
 
 
 
 
 
Algorithm  for  patient  
presenting  with  chronic  
shoulder  pain  

 
Family  Medicine  Textbook  Notes     129  
 
  •   Span  the  spectrum  from  tendinopathy  to  partial  or  complete  tears  
  •   The  majority  of  rotator  cuff  pathology  can  be  managed  with  nonsurgical  treatment  options  
  •   The  least  invasive  treatment  options  include:    
Rotator  Cuff  Disorders:   -­‐‑   Relative  rest  with  avoidance  of  overhead  activity  
Management   -­‐‑   Anti-­‐‑inflammatory  medication  (NSAIDs)  
-­‐‑   Physical  Therapy  
-­‐‑   Subacromial  injections  with  corticosteroids  are  often  used  in  the  short-­‐‑term  reduction  
of  pain  and  improvement  of  function.  Also  shown  to  be  beneficial  for  rotator  cuff  
tendonitis  
  •   Adhesive  capsulitis  (“frozen  shoulder”)  is  challenging  to  treat  
  •   Majority  of  cases  will  resolve  spontaneously  over  the  course  of  12-­‐‑18  months  
  •   Treatment  is  directed  at  decreasing  the  duration  of  symptoms  and  improving  function  
  -­‐‑   Activity  modification  to  decrease  pain    
Adhesive  Capsulitis:   -­‐‑   Anti-­‐‑inflammatory  or  analgesic  medication  
Management   -­‐‑   Physical  Therapy  
-­‐‑   Systemic  oral  steroids  have  been  found  to  provide  significant  short-­‐‑term  relief  as  well  
as  improved  range  of  motion  
-­‐‑   Intra-­‐‑articular  steroid  injection  has  been  shown  to  provide  short-­‐‑term  benefit  
-­‐‑   Arthrographic  distention  of  the  joint  with  saline  and  stoid  has  shown  short-­‐‑term  
benefit  
 
 
 
Family  Medicine  Textbook  Notes     130  
 
  •   Most  often  due  to  acute  dislocation  or  subluxation  
  •   Initial  treatment  (after  ruling  out  acute  fracture  or  significant  rotator  cuff  tear):  
Glenohumeral  Joint   -­‐‑   Activity  modification  
Instability:  Management   -­‐‑   Pain  medication  
-­‐‑   Short-­‐‑term  use  of  a  sling  for  pain  control  
•   After  initial  symptoms  have  subsided:  
-­‐‑   Strengthening  of  the  rotator  cuff  and  scapular  stabilizers  
•   If  no  improvement  with  conservative  treatment  over  6-­‐‑12  weeks,  or  if  there  is  recurrent  
episodes  of  instability,  or  if  the  patient  is  a  young  athlete  (<  20  YO)  who  with  acute  anterior  
dislocation:  
-­‐‑   Surgery  
  •   AC  arthritis  is  associated  with  hypertrophy  of  the  AC  joint  
  •   Hypertrophied  AC  joint  can  be  associated  with  impingement  and  wear  of  the  rotator  cuff  
AC  Joint  Arthritis:   between  the  hypertrophied  AC  joint  and  the  proximal  humerus  during  abduction  
Management   •   Treatment:  
-­‐‑   Pain  control  with  NSAIDs  
-­‐‑   Activity  modification  
-­‐‑   Corticosteroid  injections  into  the  AC  joint  may  be  effective  short-­‐‑term  in  severe  cases  
-­‐‑   Resection  of  distal  clavicle  is  often  ultimately  effective  in  resistant  AC  arthritis  
  •   Uncommon,  but  causes  significant  pain  and  disability  when  present  
  •   Treatment:  
Glenohumeral  Joint   -­‐‑   Focus  is  to  maintain  overall  function  with  adequate  pain  control  (NSAIDs)  
Arthritis:  Management   -­‐‑   If  pain  in  inadequately  controlled,  intra-­‐‑articular  steroid  injection  may  be  considered  
-­‐‑   Physical  therapy  can  be  helpful,  but  must  be  undertaken  with  caution  
-­‐‑   Need  to  control  comorbid  conditions  (such  as  rheumatoid  arthritis)  
-­‐‑   Surgical  referral  if  conservative  treatment  fails:  capsular  release  and  arthroscopic  
debridement,  hemiarthroplasty,  and  total  shoulder  arthroplasty  are  options  
Family  Medicine  Textbook  Notes     131  
 
  •   Fair  skinned  people  older  than  65  YO  
  •   Patients  with  many  atypical  moles  
Risk  Factors  for   •   Patients  with  ≥  50  moles  
Melanoma   •   Family  history    
•   Substantial  history  of  sun  exposure  and  sunburns  
•   When  a  person  has  had  basal  cell  carcinoma  à  risk  of  2nd  BCC  is  ≥  40%  in  the  next  3  years  

 
 
 
 
 
 
 
 
Primary  Skin  Lesions  

 
Family  Medicine  Textbook  Notes     132  
 
 
 
 
 
 
 
 
 
Secondary  Skin  Lesions  

 
Family  Medicine  Textbook  Notes     133  
 
  Microscopy  
  •   Used  for  suspected  fungal  infection  
  •   Scrape  some  of  the  scale  from  the  skin  lesion  onto  a  microscopic  slide  à  add  potassium  
Diagnostic  Testing  for   hydroxide  (KOH)  à  look  for  hyphae  of  dermatophytes  or  pseudohyphae  of  Candida  or  
assessing  skin  disorders   Pityrosporum  
Wood’s  Light  Exam  
•   Used  for  tinea  capitis  and  erythrasma  
•   Tinea  capitis  caused  by  the  Microsporum  species  produce  green  fluorescence  
•   Erythrasma  has  a  coral  red  fluorescence  
Surgical  Biopsy  
•   Shave,  punch,  or  elliptical  biopsy  can  be  useful  
  Skin  Disorders  
  •   As  severity  or  chronicity  of  disorder  increases  à  use  higher  potency  steroids  
What  factors  must  be   •   Thicker  lesions  (e.g.,  psoriatic  plaques)  à  use  higher  potency  steroids  
considered  when  you  are   Site  
choosing  a  topical  steroid   •   Face,  genitals,  skin  folds  (thin,  moist  areas)  à  use  weakest  potency  steroids  
in  order  to  maximize   •   Palms  à  use  higher  potency  steroids  
benefit  and  minimize   Age  
adverse  effects?   •   Infants,  Children  (high  surface  area  per  body  mass)  à  use  weak  potency  steroids  
 
Family  Medicine  Textbook  Notes     134  
 
 
 
 
 
 
 
 
 
Choosing  Topical  
Corticosteroids  

 
Family  Medicine  Textbook  Notes     135  
 
 
 
 
 
Strength  of  
recommendation  for  
treatment  of  bacterial  
skin  infections  of  the  
skin  

 
*Oral  antibiotics  must  cover  Group  A  Beta-­‐‑hemolytic  streptococci  and  S  aureus  (1  line:  dicloxacillin,  
st

cephalexin;  2nd  line:  clindamycin)  


  •   Superficial  skin  infection  often  characterized  by  “honey”  crusts  
  •   It  can  also  be  vesicular  or  bullous  
  •   Variations:  
Impetigo   -­‐‑   Ecthyma:  has  an  ulcerated  punched-­‐‑out  base    
-­‐‑   Bullous  
•   Staphylococcal-­‐‑scalded  skin  syndrome  (SSSS)  is  a  life-­‐‑threatening,  more  severe  variation  of  
bullous  impetigo  à  bullae  are  caused  by  exfoliating  toxin;  patient  is  systematically  ill  
-­‐‑   Patients  need  IV  antibiotics,  fluids,  and  supportive  care  
Family  Medicine  Textbook  Notes     136  
 
  •   Cellulitis  is  an  acute  infection  of  the  skin  that  involves  the  dermis  and  subcutaneous  tissues  
  à  often  caused  by  ß-­‐‑hemolytic  streptococci  and  S  aureus  
Cellulitis  and  Erysipelas   •   Erysipelas  is  a  specific  type  of  superficial  cellulitis  with  prominent  lymphatic  involvement  
causing  lesions  that  are  raised  above  the  level  of  the  surrounding  skin  with  a  clear  line  of  
demarcation  between  involved  and  uninvolved  tissue  
  •   Infection  or  inflammation  of  the  superficial  portion  of  the  hair  follicle  
  •   Usually  caused  by  S  aureus,  or  it  can  be  caused  by  other  bacteria,  yeast,  or  occlusion  
  •   Presentation  can  include  perifollicular  erythema,  papules,  or  pustules  
Folliculitis   •   Lesions  are  associated  with  hair  follicles  
•   Hot  tub  folliculitis  is  due  to  Pseudomonas  
•   Also  can  be  caused  by  Pityrosporum  yeast  or  occlusion  of  hair  follicles  with  tight-­‐‑fitting  
clothing  
  •   Localized  collection  of  pus  that  occurs  in  or  directly  below  the  skin  à  includes  furuncles,  
  carbuncles,  and  abscesses  around  fingernails  
Abscess   •   A  furuncle/boil  is  an  abscess  that  starts  in  a  hair  follicle  or  sweat  gland  
•   Carbuncle  occurs  when  the  furuncle  extends  into  the  subcutaneous  tissue  
•   Most  are  caused  by  S  aureus  
•   Treatment:  Incision  and  Drainage  
  •   Deep  infection  of  the  subcutaneous  tissues  and  fascia  
  •   Often  presents  with  diffuse  swelling  of  the  arm  or  leg,  followed  by  the  appearance  of  bullae  
  with  clear  fluid  that  may  become  violaceous  in  color  
Necrotizing  Fasciitis   •   Patient  has  marked  systemic  symptoms  such  as  pain  out  of  proportion  to  the  apparent  skin  
lesion  with  gas  in  the  soft  tissues,  edema  that  extends  beyond  the  margin  of  erythema,  
cutaneous  anesthesia,  and  systemic  toxicity  marked  by  fever,  tachycardia,  delirium,  and  
renal  failure  
•   Almost  always  due  to  S  pyogenes  
•   Can  lead  to  cutaneous  gangrene,  myonecrosis,  shock,  and  death  
Family  Medicine  Textbook  Notes     137  
 
  •   Caused  by  >  100  subtypes  of  HPV  
  •   Commonly  occur  on  the  hands,  feet,  and  genitals  
  •   Warts  on  the  hands  (verruca  vulgaris)  are  usually  raised  and  hyperkeratotic  
  -­‐‑   Tx:  salicylic  acid  or  cryosurgery  
  •   Warts  on  the  soles  of  the  feet  (plantar  warts)  are  flat,  disrupt  skin  lines,  have  dark  dots  
Warts   visible  in  them,  and  may  be  quite  painful  
-­‐‑   Tx:  salicylic  acid  or  cryosurgery  
•   Flat  warts  (verruca  plana)  are  usually  seen  in  groups  on  the  face  or  legs  
-­‐‑   Tx:  salicylic  acid,  topical  tretinoin  (Retin-­‐‑A),  cryosurgery,  or  imiquimod  (Aldara)  
•   Genital  warts  (condylomata  acuminate)  often  have  a  cauliflower  appearance  and  are  
transmitted  sexually  (90%  are  caused  by  HPV  6)  
-­‐‑   Tx:  Podophyllin  resin,  podofilox  (Condylox),  trichloroacetic  acid,  cryosurgery,  or  
imiquimod  (Aldara)  
  •   A  major  characteristic  of  herpes  infection  is  that  the  virus  lies  dormant  in  dorsal  root  ganglia,  
  leading  to  recurrences  
  •   All  herpetic  skin  infections  are  characterized  by  vesicular  eruptions  with  surrounding  erythema,  
  which  progress  to  ulcers  and/or  crusts  and  then  reepithelialize  over  the  course  of  days  or  weeks  
•   Herpes  Gingivostomatitis/Labialis  (cold  sore)  
 
-­‐‑   MCC:  HSV-­‐‑1  
 
-­‐‑   Initial  infections  are  often  the  most  severe  
Herpes  Virus   •   Genital  Herpes  
-­‐‑   MCC:  HSV-­‐‑2  
•   Chickenpox  
-­‐‑   MCC:  VZV  
-­‐‑   Consists  of  a  few  days  of  fever  and  respiratory  symptoms  with  the  characteristic  vesicles  on  a  
red  base  that  begin  on  the  trunk  and  spreads  to  the  extremities  
•   Herpes  Zoster  (shingles)  
-­‐‑   Reactivation  of  VZV  along  a  skin  dermatome  
Family  Medicine  Textbook  Notes     138  
 
  •   Tinea  of  the  head      
  •   Annular  appearance  with  central  clearing,  redness,  and  scale  on  the  perimeter  
Tinea  Capitis   •   Causes  patchy  alopecia  (hair  loss)  with  broken  hairs  and  scaling  
•   Because  the  hair  shaft  and  follicle  are  involved,  topical  antifungals  are  not  effective    
•   Oral  antifungals:  Griseofulvin,  Itraconazole,  Terbinafine  
  •   Tinea  of  the  body  à  can  occur  anywhere  
  •   Annular  appearance  with  central  clearing,  redness,  and  scale  on  the  perimeter  
Tinea  Corporis   •   Tx:  topical  OTC  antifungals  à  miconazole,  clotrimazole  
•   Large  areas  require  oral  antifungals  à  griseofulvin,  terbinafine,  itraconazole  

  •   Tinea  of  the  groin  


  •   May  be  red  and  scaling  without  the  central  clearing  seen  in  tinea  corporis  
Tinea  cruris   •   Topical  or  systemic  antifungals  may  be  used  
•   Topical  antifungals  that  cover  all  superficial  fungal  skin  infections  include  the  azoles  and  
terbinafine  (Lamisil)  
  •   Tinea  of  the  feet  
  •   May  be  seen  as  macerated  white  areas  between  the  toes  or  as  dry  red  scaling  on  the  soles  or  
Tinea  pedis   sides  of  the  feet  (moccasin  distribution)  
•   Treat  with  same  topical  or  oral  antifungals  used  for  tinea  corporis  or  cruris  

   
  •   Tinea  unguium  à  fungal  infection  of  the  nails  
  •   It  is  important  to  establish  a  definitive  diagnosis  of  onychomycosis  before  starting  
Onychomycosis     treatment  with  oral  antifungals,  because  there  are  other  causes  for  dystrophic  nails  such  as  
psoriasis,  lichen  planus,  and  trauma  
•   Terbinafine  is  most  effective  treatment  
Family  Medicine  Textbook  Notes     139  
 
  •   Type  of  eczematous  eruption  that  is  itchy,  recurrent,  and  symmetric  and  often  found  on  
  flexural  surfaces  
Atopic  Dermatitis   •   Often  have  personal  or  family  history  of  asthma  and  allergic  rhinitis  
  •   Most  cases  are  not  caused  by  specific  allergies,  but  develops  from  a  number  of  trigger  
factors  in  patients  who  have  a  strong  genetic  predisposition  to  develop  eczematous  
eruptions  
•   Tx:  avoid  skin  irritants  (e.g.  drying  soaps),  use  emollients  or  moisturizers  to  add  needed  
moisture  to  the  skin,  treat  the  inflammation  with  a  topic  steroid  or  another  anti-­‐‑
inflammatory  agents  such  as  topical  immunomodulators  (tacrolimus  and  pimecrolimus)  
  •   Allergic  response  to  an  allegen  such  as  the  chemical  found  in  the  poison  ivy  or  poison  oak  
  plant  (rhus  dermatitis)  
Allergic  Contact   •   These  lesions  are  often  linear  and  vesicular  
Dermatitis   •   Other  contact  allergens:  nickel  in  jewelry  and  belt  buckles  and  chemicals  in  deodorants  and  
cosmetics)  
•   Tx:  avoid  skin  irritants  (e.g.  drying  soaps),  use  emollients  or  moisturizers  to  add  needed  
moisture  to  the  skin,  treat  the  inflammation  with  a  topic  steroid  or  another  anti-­‐‑
inflammatory  agents  such  as  topical  immunomodulators  (tacrolimus  and  pimecrolimus)  
  •   Seen  on  the  hands  and/or  feet  
  •   Tapioca-­‐‑like  vesicles  occur  between  the  fingers  or  toes  along  with  scaling  
Dyshidrotic  Eczema   •   The  scaling  inflamed  skin  can  proceed  to  develop  painful  cracks  and  fissures  
•   Tx:  avoid  skin  irritants  (e.g.  drying  soaps),  use  emollients  or  moisturizers  to  add  needed  
moisture  to  the  skin,  treat  the  inflammation  with  a  topic  steroid  or  another  anti-­‐‑
inflammatory  agents  such  as  topical  immunomodulators  (tacrolimus  and  pimecrolimus)  
Family  Medicine  Textbook  Notes     140  
 
  •   Coin-­‐‑shaped  with  erythema  and  scale  
  •   Most  often  found  on  the  lower  legs  
Nummular  Eczema   •   Tx:  avoid  skin  irritants  (e.g.  drying  soaps),  use  emollients  or  moisturizers  to  add  needed  
moisture  to  the  skin,  treat  the  inflammation  with  a  topic  steroid  or  another  anti-­‐‑
inflammatory  agents  such  as  topical  immunomodulators  (tacrolimus  and  pimecrolimus)  
  •   Dfn:  a  superficial  inflammatory  dermatitis  that  is  characterized  by  erythema  and  scaling  on  
  the  scalp  and  face  
  •   Typical  distribution  includes  the  scalp  (dandruff);  eyebrows  and  eyelids;  cheeks;  nasolabial  
  creases;  behind  the  ears;  forehead;  cheeks;  around  the  nose;  udner  the  beard  or  mustache;  
  over  the  sternum;  and  in  the  axillae,  submammary  folds,  umbilicus,  groin,  and  the  gluteal  
Seborrhea   creases  à  these  regions  have  the  greatest  number  of  pilosebaceous  units  producing  sebums  
•   Characterized  by  remissions  and  exacerbations  à  MC  precipitating  factors  are  stress  and  
cold  weather  
•   People  with  seborrhea  have  a  profusion  of  Pityrosporum  (Malassezia)  on  the  skin  
•   Treatment  should  be  directed  at  the  inflammation  and  the  Pityrosporum  
-­‐‑   Low-­‐‑dose  steroids  (1%  hydrocortisone  cream)  
-­‐‑   Antifungals:  ketoconazole  cream  for  skin,  antifungal  shampoo  (selenium  sulfide,  zinc  
pyrithione,  ketoconazole,  or  coal  tar  derivatives)  
 
 
 
 
 
Family  Medicine  Textbook  Notes     141  
 
  •   Dfn:  chronic  condition  characterized  by  alterations  in  the  immune  system  that  lead  to  
  epidermal  proliferation  and  inflammation    
  •   Lesions  are  well-­‐‑circumscribed,  red,  scaling  plaques,  with  white  thickened  scales  
  •   Areas  affected  include  the  scalp,  nails,  and  extensor  surfaces  of  the  limbs  
  •   Treatment:  
  -­‐‑   Emollients  
Psoriasis   -­‐‑   Topical  steroids  
-­‐‑   Vitamin  D  (calcipotriene  or  calcipotriol)  
-­‐‑   Topical  tar  and  tar  shampoo  
-­‐‑   Intralesional  steroids  
-­‐‑   UV  light  
-­‐‑   Topical  retinoids  
-­‐‑   Systemic  treatment  with  methotrexate,  acitretin,  cyclosporine,  and  injectable  biologics  
-­‐‑   Tarazotene  (topical)  is  effective  for  plaque  psoriasis  
 
 
 
 
 
 
 
 
 
Family  Medicine  Textbook  Notes     142  
 
  •   Dfn:  inflammatory  disease  of  the  pilosebacous  unit  which  involves  blockage  of  the  unit  
  with  sebum  and  desquamated  cells,  accompanied  by  the  overgrowth  of  P  acnes  in  the  
  follicle  
  •   Treatment:  
 
 
 
 
 
 
 
 
Acne  

 
Family  Medicine  Textbook  Notes     143  
 
  •   Most  potent  medication  for  acne  
  •   Has  many  potential  side  effects,  including  birth  defects  
Isotretinoin  (Accutane)   •   Should  NOT  be  prescribed  or  dispenses  to  any  woman  without  two  negative  pregnancy  
tests  and  two  forms  of  birth  control  

 
 
 
 
 
 
 
 
ABCDE  Guidelines  for  
Melanoma  

 
Family  Medicine  Textbook  Notes     144  
 
  •   Most  common  skin  cancer  
  •   85%  are  on  the  head  and  neck  
Basal  Cell  Carcinoma   •   3  major  morphologic  types:  
(Skin)   -­‐‑   Nodular:  pearly  and  raised  with  telangiectasias  à  expands  à  ulcerates,  bleeds,  and  
becomes  crusted  
-­‐‑   Superficial:  red  or  pink,  flat,  scaling  plaques  that  may  have  erosions  or  crusts  
-­‐‑   Sclerosing:  flat  and  scarlike;  very  rare  
Squamous  Cell   •   Can  look  like  superficial  BCC  or  can  be  more  elevated  and  nodular  
Carcinoma  (Skin)   •   SCCs  are  frequently  hyperkeratotic  and  bleed  easily  
  •   Premalignant  lesions  
Actinic  Keratoses   •   Treated  with  cryotherapy  for  a  small  number  of  lesions  or  topical  field  treatment  for  areas  
with  many  lesions  (5-­‐‑fluorouracil,  imiquimod,  and  diclofenac)  
Sebaceous  hyperplasia   •   May  look  like  BCC  
•   Benign  lesions  that  are  raised  and  can  have  pearly  borders  and  telangiectasias  
   
  •   Can  mimic  melanoma  
Seborrheic  Keratosis   •   These  develop  with  age  and  are  often  large  and  pigmented  with  irregular  borders  
•   Have  a  stuck-­‐‑on  appearance,  but  they  can  be  flat  and  irregular  
Family  Medicine  Textbook  Notes     145  
 
  •   A  chronic,  genetically  linked  primary  headache  that  usually  begin  in  late  childhood  or  early  
  adulthood  
  •   Neurologic  symptoms  may  either  precede  (“aura”)  or  accompany  the  headache.  Patients  
  often  report  pain  in  the  face  or  around  (or  behind)  one  eye  
Migraine  Headaches   •   Typical  features:  intense  facial  pain,  photophobia  
•   The  frequency,  severity,  and  associated  symptoms  can  vary  between  patients  and  even  
within  a  given  person’s  lifetime  
•   Known  triggers  include  estrogen  changes  (menstrual  cycle),  caffeine,  sleep  deprivation,  
psychosocial  stressors,  or  changes  in  weather  or  barometric  pressure  
 
 
 
 
 
Diagnostic  criteria  for  
migraines  with  and  
without  typical  aura  

 
Family  Medicine  Textbook  Notes     146  
 
 
 
 
Treatment  for  acute  
migraines  and  for  
prevention  of  migraines  

 
  1)   Headaches  >  2  days/week,  on  average  
  2)   Recurring  migraines  that,  in  the  patient’s  opinion,  significantly  interfere  with  his  or  her  
Clinical  criteria  for   daily  routine  
initiating  migraine   3)   Failure  of,  or  contraindications  to,  acute  therapies  
prophylaxis:   4)   Patient  preference  
5)   Significant  cost  of  acute  therapies  
6)   Presence  of  uncommon  headache  conditions  including  hemiplegic  migraine,  basilar  
migraine,  migraine  with  prolonged  aura,  or  migrainous  infarction  
Family  Medicine  Textbook  Notes     147  
 
 
 
 
 
 
Common  Headache  
Triggers  

 
  •   Most  common  type  of  headache  overall  
  •   Has  are  usually  mild  or  moderate  in  severity  and  are  often  self-­‐‑treated  
  •   Pain  is  bilateral  in  a  “headband-­‐‑like”  pattern  
Tension-­‐‑Type  Headaches   •   They  are  commonly  episodic  but  can  develop  into  daily  or  near-­‐‑daily  headaches  
Treatment  

 
Family  Medicine  Textbook  Notes     148  
 
  •   More  prevalent  in  males;  very  rare  (0.3-­‐‑0.4%)  
  •   Classic  presentation  is  described  as  a  series  of  headaches  occurring  close  together  over  6-­‐‑12  
  weeks  à  severe,  intense,  unilateral  pain  lasting  from  several  seconds  to  many  minutes  
  •   Concurrent  symptoms  include  ipsilateral  lacrimation,  rhinorrhea,  and  ptosis  
Cluster  Headaches   •   The  headache  is  always  on  the  same  side,  no  matter  how  many  months  lapse  between  
episodes  
Treatment  

 
   
  •   Have  symptoms  suggestive  of  nasal  or  sinus  etiology  (rhinosinusitis)  including  purulence  
Sinus  Headache   in  the  nasal  cavity,  nasal  obstruction,  altered  smell  (hyposmia  or  anosmia),  and/or  fever  
•   Often  have  incomplete  resolution  of  pain  after  taking  decongestants  
•   Patients  may  actually  have  migraines  rather  than  sinusitis  

  •   Headaches  that  occur  daily  or  nearly  daily  


  •   Paradoxically,  the  very  medications  commonly  used  to  treat  episodic  headaches  (including  
  OTC  analgesics,  especially  acetaminophen,  and  migraine-­‐‑specific  medications  such  as  
Chronic  Daily  Headache   triptans)  are  implicated  in  the  transformation  of  episodic  to  chronic  headaches,  especially  if  
consumed  more  often  than  2  days  per  week  over  several  months  
Treatment  

 
Family  Medicine  Textbook  Notes     149  
 
 
Signs  and  symptoms  that  
suggest  neuroimaging  
may  be  indicated  in  
patients  with  headaches  

 
 
 
Red  flags  suggesting  that  
a  headache  may  indicate  
a  progressive  or  life-­‐‑
threatening  disease  

 
  •   Commonly  presents  with  severe  headache  reaching  maximal  intensity  within  minutes  and  
Subarachnoid   lasting  an  hour  or  more  
Hemorrhage   •   SAH  may  be  accompanied  by  focal  neurologic  signs  or  other  symptoms  such  as  nausea,  
vomiting,  photophobia,  neck  stiffness,  seizures,  or  altered  level  of  consciousness  
 
 
 
Family  Medicine  Textbook  Notes     150  
 
  Presentation  
  •   Child  maltreatment  often  presents  with  symptoms  of  inattention,  school  failure,  disruptive  
  symptoms,  anxiety,  depression,  failure  to  thrive,  and  a  broad  range  of  somatic  symptoms  
  Physical  Abuse  
  •   Suspect  in  cases  of  childhood  injury  that  are  (i)  unexplained,  (ii)  not  plausible  by  the  
  explanation  offered,  (iii)  in  a  pattern  suspicious  for  inflicted  injury,  (iv)  developmentally  
  inconsistent,  or  (v)  from  punishment  with  excessive  force  
  •   Physical  markings  that  last  ≥  24  hours  are  regarded  as  significant  injuries    
  Sexual  Abuse  
  •   Usually  presents  with  child  disclosure.  However,  may  present  with  acute  sexual  trauma,  
Child  Maltreatment   STDs,  pregnancy,  extremes  of  sexualized  behavior,  and  somatic  symptoms  such  as  dysuria  
and  enuresis  
Neglect  
•   Neglect  can  be  thought  of  as  failure  to  meet  the  basic  needs  of  a  child,  such  as  adequate  
supervision,  food,  clothing,  shelter,  medical  care,  education,  and  love.  
•   Often  manifests  as  a  pattern  of  chronic  unmet  needs  
•   The  cause  of  neglect  may  or  may  not  be  malevolent,  but  the  child  is  still  at  risk  à  for  
example,  a  poor  single  father  may  leave  his  2-­‐‑year-­‐‑old  child  alone  at  night  so  that  he  can  
work  a  2nd  job  
Psychological  Abuse  
•   Common,  but  very  difficult  to  substantiate  because  of  social  norms  
•   Some  examples  include  (i)  threatening  to  leave  or  abandon  a  child,  (ii)  threatening  to  kick  a  
child  out  of  the  home,  (iii)  locking  a  child  out  of  the  house,  or  (iv)  calling  a  name  like  
stupid,  ugly,  or  useless  
•   Often  only  made  through  long-­‐‑term  observation  of  parent-­‐‑child  interaction  
•   Symptoms  include  aggressiveness,  impulsivity,  depression,  hyperactivity,  school  failure,  
inattention,  disturbances  of  conduct,  anxiety,  eating  disorders,  and  somatic  symptoms  
•   Consider  a  referral  to  family  therapy  
Family  Medicine  Textbook  Notes     151  
 
Management  
•   Physicians  are  mandated  by  law  to  report  suspected  child  abuse  and  neglect  à  laws  
include  immunity  from  lawsuits  for  reports  made  in  good  faith  
•   It  is  not  required  by  law  that  the  parent  be  informed  of  the  report    

 
 
 
 
 
Suspicious  injuries  
indicative  of  child  abuse  

 
Family  Medicine  Textbook  Notes     152  
 
 
 
 
 
 
Situations  that  should  
raise  suspicion  for  
intimate  partner  violence  

 
  Physical  Violence  
  •   Intentional  use  of  physical  force  with  the  potential  for  causing  death,  disability,  injury,  or  
  harm  
  Sexual  Violence  
  •   (i)  use  of  physical  force  to  compel  a  person  to  engage  in  a  sexual  act  against  his  or  her  will,  
Intimate  Partner  Violence   whether  or  not  that  act  is  completed  
  •   (ii)  Attempted  or  completed  sex  act  involving  a  person  who  is  unable  to  understand  the  
  nature  or  condition  of  the  act,  to  decline  participation,  or  to  communicate  unwillingness  to  
  engage  in  the  sexual  act  (e.g.  illness,  disability,  influence  of  alcohol/drugs,  or  because  of  
  intimidation)  
  •   (iii)  abusive  sexual  contact  
  Psychological/Emotional  Violence  
  •   Trauma  to  the  victim  caused  by  acts,  threats  of  acts,  or  coercive  tactics  
   
Family  Medicine  Textbook  Notes     153  
 
  Common  Presentations  
  •   Injuries  
  -­‐‑   Partner  violence  causing  physical  injury  is  the  most  direct  health  effect  of  IPV  
  -­‐‑   Certain  patterns  of  injury,  such  as  injuries  to  the  head,  neck,  breast,  or  abdomen  
  should  raise  suspicion  
  •   Other  Physical  Health  Effects  
  -­‐‑   Concerns  related  to  sexual  health,  such  as  STIs,  cervical  dysplasia,  and  unplanned  
  pregnancy  are  common  
Intimate  Partner   -­‐‑   Victims  are  at  increased  risk  for  CV  disease  and  for  stroke  
Violence,  continued   -­‐‑   Could  present  with  GI  disorders  such  as  irritable  bowel  syndrome  
  -­‐‑   Could  present  with  a  variety  of  chronic  pain  complaints,  such  as  arthritis,  migraine,  
  fibromyalgia,  chronic  fatigue  syndrome,  and  temporomandibular  joint  syndrome  
  -­‐‑   May  present  with  somatic  complaints,  such  as  stomach  pain,  back  pain,  menstrual  
  problems,  headaches,  chest  pain,  dizziness,  fainting  spells,  palpitations,  shortness  of  
  breath,  constipation,  generalized  fatigue,  and  insomnia  
  •   IPV  and  Pregnancy  
  -­‐‑   IPV  often  continues  throughout  pregnancy  
  -­‐‑   Victims  are  often  delayed  in  seeking  prenatal  care  
  -­‐‑   Infants  are  at  risk  for  low  birth  weight,  prematurity,  and  perinatal  death  
  •   Mental  Health  
  -­‐‑   Victims  commonly  experience  depression,  suicidal  thoughts  and  attempts,  and  PTSD  
  -­‐‑   Tobacco,  alcohol,  and  illicit  drug  abuse  are  common  
  -­‐‑   Abused  women  are  more  likely  to  have  disordered  eating  patterns  
   
   
   
   
   
Family  Medicine  Textbook  Notes     154  
 
  Assessment  
  •   All  clinicians  examining  children  and  adults  should  be  alert  to  physical  and  behavioral  
  signs  and  symptoms  associated  with  abuse  or  neglect.  Patients  in  whom  abuse  is  suspected  
  should  receive  proper  documentation  of  the  incident  and  physical  findings,  treatment  for  
  physical  injuries,  arrangement  for  skilled  counseling  by  a  mental  health  professional,  and  
  the  telephone  number  of  local  crisis  centers,  shelters,  and  protective  service  agencies  
  •   Physicians  should  ensure  a  private  setting,  without  friends  or  family  present.  They  should  
  assure  patients  of  confidentiality,  but  notify  them  if  any  reporting  requirements  apply.  
Intimate  Partner   Language  should  be  direct  and  nonjudgmental  
Violence,  continued   Management  
•   Key  components  of  an  initial  interaction  should  include  validation  of  the  patient’s  concerns,  
education  regarding  the  dynamics  and  consequences  of  IPV,  safety  assessment,  and  referral  
to  local  resources  
•   An  initial  response  to  a  disclosure  of  IPV  should  include  listening  to  the  patient  
empathically  and  nonjudgmentally,  expressing  a  concern  for  her  health  and  safety,  and  
affirming  a  commitment  to  help  her  address  the  problem  
•   Abused  women  often  have  very  low  self  esteem,  so  it  is  important  to  physicians  to  help  
counter  this  belief,  reassuring  that  although  partner  violence  is  common,  it  is  unacceptable  
and  not  the  fault  of  the  victim  
 
 
WAST-­‐‑Short  screening  
for  domestic  violence  

 
Family  Medicine  Textbook  Notes     155  
 
 
 
 
Intimate  partner  violence  
red  flags  indicating  
increased  risk  for  serious  
injury  or  homicide  

 
  Background  
  •   Elder  mistreatment  is  defined  as  intentional  actions  that  cause  harm  or  create  a  serious  risk  of  harm  
  (whether  or  not  harm  is  intended)  to  a  vulnerable  elder  by  a  caregiver  or  other  person  who  stands  in  
  a  trust  relationship  to  the  elder,  or  failure  by  a  caregiver  to  satisfy  the  elder’s  basic  needs  or  to  
protect  the  elder  from  harm  
 
•   Includes  physical  abuse,  psychological  abuse,  sexual  abuse,  financial  exploitation,  and  neglect  
 
•   Elders  who  live  with  their  caregivers  are  more  likely  to  be  victims  of  mistreatment.  Social  isolation  
Elder  Abuse   of  both  elders  and  their  caregivers  also  appears  to  increase  risk  for  mistreatment.  Patients  with  
dementia,  in  particular  patients  who  have  disruptive  behavior  or  aggression,  are  at  increased  risk  
Assessment  
•   Insufficient  evidence  to  recommend  for  or  against  routine  screening  
•   There  is  no  clear  constellation  of  symptoms  that  is  suggestive  of  elder  mistreatment  à  remain  alert  
to  bruises  or  burns  in  unusual  locations  or  injuries  that  are  not  consistent  with  the  explanation  
offered.  Genital  or  breast  injuries  should  raise  suspicion  of  sexual  abuse.  Findings  that  should  raise  
suspicion  for  neglect  include  dehydration  or  malnutrition,  pressure  ulcers,  poor  hygiene,  or  medical  
nonadherence  
Management  
•   In  most  states,  reporting  of  elder  abuse  and  neglect  is  legally  mandated  
Family  Medicine  Textbook  Notes     156  
 
 
 
 
 
 
 
Criteria  for  an  episode  of  
Major  Depression  

 
Family  Medicine  Textbook  Notes     157  
 
 
Criteria  for  a  single  
episode  of  MDD  

 
 
Criteria  for  recurrent  
MDD  

 
  •   Complaints  that  involve  multiple  organ  systems  or  are  physiologically  unrelated  
  •   Emotional  flatness,  or  worry  that  is  not  consistent  with  the  severity  of  the  presenting  
What  are  some  clues  that   problem  
may  indicate  a  patient   •   Sleep  disturbance  that  is  persistent  or  unrelated  to  obvious  stressors  
has  depression?   •   Frequent  office  visits  for  unclear  or  seemingly  minor  complaints  
•   Frequent  emergency  room  visits  for  unexplained  physical  symptoms  
•   Patients  who  are  “difficult”  for  unclear  reasons  
•   Patients  who  express  thoughts  or  emotions  that  are  inappropriate  to  the  context  
•   Patients  with  a  previous  history  of  emotional  disturbances  or  “nervous  breakdowns”  
Family  Medicine  Textbook  Notes     158  
 
 
 
 
 
 
 
 
 
Diagnostic  criteria  for  
dysthymic  disorder  

 
Family  Medicine  Textbook  Notes     159  
 
 
 
 
 
 
 
 
Diagnostic  criteria  for  
mood  disorders  
secondary  to  a  medical  
condition  

 
Family  Medicine  Textbook  Notes     160  
 
 
 
 
 
 
Criteria  for  a  hypomanic  
episode  

 
 
 
Criteria  for  Bipolar  II  
Disorder  

 
Family  Medicine  Textbook  Notes     161  
 
 
 
 
 
 
 
Criteria  for  seasonal  
pattern  specifiers  for  
depressive  disorders  

 
Family  Medicine  Textbook  Notes     162  
 
 
 
 
 
 
 
 
 
 Medications  that  often  
cause  or  worsen  
depression  

 
Family  Medicine  Textbook  Notes     163  
 
 
 
 
 
 
 
 
Red  flags  suggesting  
more  serious  or  complex  
disease  in  patients  
presenting  with  
depression  

 
Family  Medicine  Textbook  Notes     164  
 
 
 
 
 
 
 
 
Factors  that  increase  the  
risk  of  suicide  in  
depressed  patients  

 
  •   Patients  who  have  considered  a  specific  plan  for  suicide  or  have  made  a  suicidal  gesture  
How  do  you  manage   should  be  referred  to  a  mental  health  specialist  for  rapid  evaluation  
depressed  patients  who   •   Patients  with  suicidal  thoughts  may  require  escorting  to  an  emergency  room  with  mental  
are  at  risk  of  attempting   health  specialists  on  staff.  
suicide?   •   Telephone  contact  between  the  family  physician  and  receiving  specialist  is  essential  to  
convey  the  specific  statements  of  the  patient  that  are  eliciting  concern.  
Family  Medicine  Textbook  Notes     165  
 
  Medication  should  be  initiated  when:  
When  do  you  initiate   •   Symptoms  have  been  present  for  more  than  1  month  
medications  in  patients   •   Symptoms  result  in  significant  interference  with  ability  to  function  at  work  or  home,  or  
with  MDD?   •   Score  on  the  PHQ-­‐‑9  is  ≥  15  

 
 
 
 
 
 
Recommended  
Treatments  for  MDD  

 
Family  Medicine  Textbook  Notes     166  
 
  •   History  of  good  response  to  previous  use  
  •   Successful  use  of  an  agent  in  a  close  relative  (use  by  a  parent  or  sibling  may  enhance  
What  considerations   compliance)  
should  be  made  when   •   Presence  of  chronic  pain  or  severe  sleep  disturbance  (if  so,  consider  using  a  TCA)  
you  are  choosing  an  anti-­‐‑ •   Coexisting  medical  conditions  (e.g.,  avoid  TCAs  in  patients  with  known  cardiac  conduction  
depressant  medication  to   disturbances)  
prescribe?   •   Hypersomnia  (if  so,  consider  an  SSRI)  
•   Cost  
  •   Allow  12-­‐‑14  weeks  of  treatment  at  maximum  effective  dose  before  abandoning  the  drug  
For  how  long  should  you   being  used.  If  there  is  no  response  by  14  weeks,  augment  or  switch  medication  and/or  
keep  a  patient  on  an   consider  referral  
antidepressant  drug  that   •   It  is  important  to  emphasize  to  the  patient  that  failure  of  one  medication  does  not  mean  that  
has  not  begun  to  work?   the  condition  is  untreatable.  

  •   Asthma  is  defined  as  a  chronic  inflammatory  disease  involving  mast  cells,  neutrophils,  
  eosinophils,  T  lymphocytes,  macrophages,  and  epithelial  cells  à  this  inflammatory  process  
Asthma  definition   may  lead  to  diffuse,  variable  airflow  obstruction,  manifested  as  recurrent  episodes  of  
coughing,  wheezing,  breathlessness,  and  chest  tightness  
•   Airway  obstruction  in  asthma  worsens  in  response  to  a  number  of  stimuli  (such  as  
infection,  allergies,  and  irritants),  and  improves  either  spontaneously,  after  withdrawal  of  
the  offending  stimulus,  or  with  treatment.  
  •   BMI  
  •   Smoking  or  exposure  to  secondhand  smoke  
Modifiable  risk  factors   •   Maternal  use  of  tobacco  during  pregnancy    
for  asthma?   •   Exposure  to  allergens  from  pollen  types,  dust  mites,  cockroaches,  cats,  dogs,  rodents,  and  
some  fungi  
•   Exposure  to  outdoor  air  pollution,  such  as  ozone,  nitrogen  oxides,  sulfur  dioxide,  and  diesel  
exhaust  particles  
Family  Medicine  Textbook  Notes     167  
 
  •   Allergic  rhinitis  
  •   Postnasal  drip  
What  is  the  differential   •   Sinusitis  
diagnosis  for  chronic   •   GERD  
cough  similar  to  asthma?   •   CHF  
•   COPD  
•   Airway  obstruction  

 
 
 
 
 
Key  elements  in  the  
history  and  physical  
examination  for  asthma  

 
Family  Medicine  Textbook  Notes     168  
 
  •   A  chronic  cough  lasting  several  months  primarily  lasting  several  months  primarily  
  occurring  at  night  may  be  the  only  symptom  
  •   More  typical  symptoms  of  asthma  include  difficulty  breathing,  wheezing  and  cough,  either  
  in  combination  or  alone  
  •   Adults  with  asthma  may  describe  difficulty  in  getting  enough  air,  tightness  in  the  neck  or  
  chest,  or  having  to  work  harder  or  concentrate  more  in  order  to  breathe  
  •   Exercise  may  precipitate  asthma  symptoms  by  increased  ventilation,  resulting  in  the  
  irritation  of  airways  secondary  to  loss  of  heat  and  moisture  à  exercise-­‐‑induced  asthma  
Typical  symptoms  of   (compared  to  poor  physical  conditioning)  usually  starts  5-­‐‑15  minutes  after  starting  exercise  
asthma   and  continues  more  than  10  minutes  after  stopping  
•   During  an  acute  episode,  the  patient  may  look  anxious,  have  rapid  or  labored  breathing,  
and  be  sitting  upright  or  standing  to  maximize  efficiency  of  thoracic  muscles  
•   Vital  signs:  
-­‐‑   Elevated  respiratory  rate  and  heart  rate  
-­‐‑   Fever  present  if  infection  is  the  trigger  
-­‐‑   Pulsus  paradoxus  (when  the  difference  between  systolic  blood  pressure  in  inspiration  
and  expiration  is  increased  >  10  mm  Hg)  may  be  present  with  significant  airway  
obstruction  
•   Auscultation  
-­‐‑   Most  useful  technique  
-­‐‑   Decreased  intensity  of  breath  sounds  is  most  common  finding  
-­‐‑   Wheezes  during  expiration  are  characteristic  of  obstructive  airway  disease  
-­‐‑   Rhonchi  (low-­‐‑pitched  breath  sounds  similar  to  snoring)  are  often  present  during  
exacerbations  
Family  Medicine  Textbook  Notes     169  
 
 
 
 
 
 
 
 
 
Physical  examination  
findings  in  status  
asthmaticus    

 
Family  Medicine  Textbook  Notes     170  
 
  •   Spirometry  is  recommended  to  confirm  diagnosis  and  define  the  severity  of  asthma  
  •   Basic  tests  are  forced  vital  capacity  (FVC),  forced  expiratory  volume  in  1  second  (FEV1),  
  and  the  ratio  of  the  two  (FEV1/FVC)  
Pulmonary  Function   •   Obstructive  disease  =    
Tests  for  Patients  with   -­‐‑   FEV1/FVC  ratio  <  70%  in  adults  (or  <  80%  in  children)  
Asthma   -­‐‑   In  asthma,  both  the  FEV1  and  FEV1/FVC  ratios  are  decreased  
-­‐‑   In  asthma,  FEV1  improves  at  least  12  %  (and  at  least  200  mL)  with  bronchodilators  
such  as  inhaled  albuterol  à  if  it  doesn’t  improve  by  12%,  diagnosis  is  more  likely  to  
be  COPD  
•   Methacholine  Challenge  Test  
-­‐‑   Methacholine  is  given  in  progressively  higher  concentrations  to  determine  the  dose  
causing  a  20%  reduction  in  FEV1  
-­‐‑   Reaction  to  a  concentration  <  10  mg/mL  is  diagnostic  of  asthma  
  •   Control  factors  that  contribute  to  asthma  severity  (such  as  allergens  and  irritants)  to  
  improve  baseline  respiratory  status  and  decrease  the  frequency  of  exacerbations  
  •   Monitor  respiratory  status  with  objective  measures  of  lung  function  for  diagnosis,  for  
Asthma  Management   classification  of  severity,  and  to  assess  response  to  treatment  
•   Use  pharmacologic  therapy  to  address  the  inflammatory  nature  of  asthma  
•   Provide  education  for  a  partnership  in  asthma  care,  so  that  the  patient  and  family  
understand  the  disease  well  enough  to  be  motivated  to  make  changes,  use  medications  
wisely,  and  work  collaboratively  with  the  physician.  
Family  Medicine  Textbook  Notes     171  
 
 
 
 
 
 
Classification  of  asthma  
in  adults  

 
Family  Medicine  Textbook  Notes     172  
 
 
 
 
Long-­‐‑term  control  
medications  for  
managing  asthma  in  
adults  

 
  •   Pregnant  women  with  asthma  can  be  treated  accordingly  to  the  same  protocols  as  other  
  patients  
Treating  pregnant   •   All  of  the  inhaled  asthma  medications  may  be  used  in  pregnancy.  Montelukast  and  
women  with  asthma   zafirlukast  both  have  Category  B  safety  indications  for  pregnancy.  Because  very  little  
prednisone  crosses  the  placenta,  it  is  also  considered  safe  in  pregnancy  
Family  Medicine  Textbook  Notes     173  
 
 
 
 
 
 
 
 
 
Red  flags  for  increased  
risk  of  death  from  asthma  

 
Family  Medicine  Textbook  Notes     174  
 
   
Make  sure  you  go    
through  the  USPSTF  A   https://www.uspreventiveservicestaskforce.org/Page/Name/uspstf-­‐‑a-­‐‑and-­‐‑b-­‐‑recommendations/  
and  B  Recommendations  

 
 
 

You might also like